You are on page 1of 1100

R

SI
PHYSICAL WORLD
IT
1.1 WHAT IS SCIENCE? Scientific attitude. The tremendous growth of
1. What is science ? science has taken place only due to the use of scientific
attitude and scientific method. The scientific attitude
Science. It is a systematised and organised knowledge
requires a flexible, open-minded approach towards solving
about the various natural phenomena which is obtained by
problems in which other important points of view are not
careful experimentation, keen observation and accurate rea-
H
neglected without any reason. First a solution. is
soning. The Sanskrit word Shastra and Arabic word Ilm
suggested for a problem. This -solution is tried. If it
also have a similar meaning i.e., organised knowledge.
works satisfactorily, it is adopted and otherwise it is
2. What are physical and biological sciences. ? replaced by a better solution to the same problem.
Two types of sciences. The knowledge of science 4. What is scientific method ? Mention the various
O

can be divided into two broad categories: steps involved in it. .


1. Biological sciences. The sciences which deal with the Scientific method. The step by step approach used by
behaviour of living things are called biological sciences. a scientist in studying natural phenomena and establishing
These sciences include Botany, Zoology, Orinthology,
laws which govern these phenomena is called scientific
Anthropology, Entomology, Forensic Science, etc.
method.
M

2. Physical sciences. The two main physical Generally, it involves the following steps:
sciences are physics and chemistry. Chemistry is the
1. Taking a large number of systematic observations
study of every substance, its structure, its composition and
by doing controlled experiments.
changes in which it takes part. Physics is the study of the
natural world which deals with the concepts of space, time, 2. Studying these observations and making quali-
motion, matter, energy, radiation, etc. Other physical tative and quantitative reasoning.
sciences include Geology, Geography, Astronomy, 3. Suggesting mathematical models to account for
Astrology, Oceanology, etc. the observed behaviour.
4. Predicting new phenomena on the basis of
1.2 SCIENTIFIC ATTITUDE AND suggested model.
SCIENTIFIC METHOD 5. Modifying the theory, if necessary, in the light
3. What is scientific attitude ? of the fresh evidences.

- --------
1.2 PHYSICS-XI

1.3 SCIENTIFIC THEORY 2. Sometimes a theoretical advancement predicts a


5. What is scientific theory ? new experiment. For example, the concept of
antiparticle was introduced theoretically by
Scientific theory. Theory is the name given to a set of Paul Dirac in 1930 and was confirmed two years
limited number of laws in terms of which the behaviour of a later by the experimental discovery of positron
physical system can be explained. A good theory should (anti-electron) by Carl Anderson. .
not only explain the already existing phenomenon but
it should be able to predict and explain the new 1.4 WHAT IS PHYSICS?
phenomenon.
8. What is physics ? What is the origin of the word
6. When does a scientific theory need modification or physics ? What is its Sanskrit equivalent?

R
replacement by a new theory ? Briefly explain giving Physics. The word physics originates from a Greek
suitable examples. word which means nature. This word was introduced
Need for modification of a scientific theory. In by ancient scientist Aristotle in the year 350 B.c. The
science, no theory is final. A theory must be able to Sanskrit equivalent of physics is bhauitaka which refers
explain what may be observed from time to time. to the physical world.

SI
Following two types of problems may ari~e : Physics is the branch of science that deals with the study
1. When any new observation shows some devia- of basic laws of nature and their manifestation in various
tion from the existing theory, the theory has to be duly natural phenomena. It is concerned with the interaction
modified. For example, when Johann Kepler examined of matter with matter or energy. It deals with the
the extensive data on planetary motion collected by various features of the natural world such as space,
Tycho Brahe, the planetary circular orbits in helio- time, matter, motion, energy, radiation, etc. Physics is
centric theory (sun at the centre of the sol at: system) the most fundamental of all sciences as it is concerned
imagined by Nicholas Coperanicus had to be replaced with the study of various natural phenomena.
IT
by elliptical orbits to fit the data better.
1.5 TWO BASIC QUESTS IN PHYSICS
2. When even the modification of a theory fails to
account for the new observations, the theory has to be
9. What are the two principal thrusts in physics ?
Give suitable examples for each.
replaced by a new theory. For example, it was realised
in the beginning of twentieth century that Newtonian Two basic quests in physics. The two principal
mechanics could not explain some basic features of thrusts in physics are unification and reductionism.
H
atomic phenomena. Also, the wave theory of light 1. Unification. In physics, attempt is made to
failed to explain photoelectric effect. This resulted in explain various physical phenomena in terms of just
the development of an entirely new theory, called Quan- few concepts and laws. We try to see the physical
tum Mechanics, to deal with microscopic phenomena. world as manifestation of some universal laws. For
7. Theory and experiments go hand in hand in example, the same (Newton's) law of gravitation can
be used to describe the motion of a body falling
O

physics and help each 'other's progress. Give two suitable


examples in support of your answer. towards the earth, motion of the moon around the
earth and motion of planets around the sun. Attempts
Or
are being made to unify fundamental forces of nature
With the help of suitable examples, briefly explain the in the persuit of unification.
interplay between theoretical models and experiments.
2. Reductionism. Another attempt made in physics
M

Interplay between theoretical models and is to explain a macroscopic system in terms of its
experiments. The interplay of theory and experiment microscopic constituents. This persuit is called red-
(or observation) is important for the progress of uctionism. For example, thermodynamics was developed
science. to explain the macroscopic properties such tempe-
This is obvious from the following examples: rature, internal energy, entropy, etc., of the bulk systems.
1. Sometimes a new experiment suggests an alter- Later on, these properties were explained in terms of
native theoretical model. For example, alpha molecules in kinetic theory and statistical mechanics.
particle scattering experiments in 1911 by Ernest
1.6 SCOPE OF PHYSICS
Rutherford established the nuclear model of the
atom, which then became the basis of quantum Introduction. The scope of physics is very wide.
theory of hydrogen atom given by Niels Bohr Every event, which occurs around us in our daily life,
in 1913. is governed by one or the other principle of physics.

~'------
PHYSICAL WORLD 1.3

One way of getting the idea of the scope of physics is to covers wide ranges of mass, length, time, energy, etc.
look at its various sub-disciplines. Another way is look The mass of an object can vary from 10- 30 kg (mass of
at the vast range of magnitude of the physical an electron) to 1055 kg (mass of the universe). The
quantities it deals with. length of an object can vary from 10- 14 m (size of a
/ 10. Briefly explain the main disciplines and sub- nucleus) to 1026 m (size of the universe). Time interval
. disciplines in physics. can vary from 10- 22 s (time taken by light to cross a'
nuclear distance) to 1018 s (life of the sun). The
~.,

Branches of physics. Physics has two main


everyday phenomena lie in the middle ranges.

.*
domains of study-macroscopic and microscopic.
Classical physics deals with macroscopic phenomena
which may be at the laboratory, terrestrial and astro-
For Your Knowledge

R
nomical scales. It includes branches like mechanics, .\. The scope of physics (or science in general) is very
optics, thermodynamics and electrodynamics. Quantum wide. It has been quite aptly described by some
mechanics deals with microscopic phenomena at the scientists as follows :
/ minute scales of atoms, molecules and nuclei. ~ Science is a method for describing, creating and
understanding human experience.
Main sub-disciplines in physics:

SI
R. Bruce Lindsay
1. Mechanics. It deals with the equilibrium or motion of
material bodies at low speeds. It is based on laws of ~ Science is the ever unfinished quest to discover all
gravitation. The propulsion of rocket, equilibrium of facts, the relationship between things and the
rod bent under a load, propagation of water waves or laws by which the world runs. Gerold Holton
sound waves in air, etc., are studied in mechanics. ~ The two processes that of Science and Art are not
very different. Both Science and Art, formed in the
2. Optics. It deals with the nature and propagation of
course of centuries a human language by which we
light. It deals with the formation of images by mirrors
can speak about the more real part of reality.
and lenses, colours in thin films, etc.
Heisenberg
IT
3. Thermodynamics. It deals with a macroscopic
system in equilibrium and is concerned with the changes ~ The task of science is both to extend the range of
in internal energy, temperature, entropy, etc., of the
our experience and to reduce it to order.
system through external work and heat. Here we study Niels Bohr
the efficiency of heat engines and refrigerators. ~ The most incomprehensible thing about the world
4. Electrodynamics. It deals with electric and magnetic is that it is comprehensible. Albert Einstein
phenomena associated with charged and magnetic bodies. It ~ Nature is pleased with simplicity and effects, not
H
is based on laws given by Coulomb, Oersted, Ampere the pump of superfluous causes. Isaac Newton
and Faraday, which were later on unified by Maxwell. ~ We know very little and yet it is astonishing that
It deals with problems like motion of current-carrying we know so much and still more astonishing that so
conductor in a magnetic field, propagation of radiowaves little knowledge can give us so much power.
through the atmosphere, etc. Bertrand Russel
O

5. Quantum mechanics. It deals with the mechanical


behaviour of sub-microscopic particles like atoms and 1.7 EXCITEMENT OF PHYSICS
nuclei and their interaction with projectiles like 12. Physics is a science of excitement. How?
electrons, photons and other elementary particles.
Excitement of physics. The study of physics is not
6. Relativity. It is theory of invariance in nature. It
only educative but also exciting in many ways as follows:
M

deals with the motion of the particles having speeds


comparable to the speed of light. 1. Inspite of the wide range and complexity of the

.;f
.\. An exciting field of research these days is a new
physical phenomena, it is quite exciting that these
phenomena can be analysed and understood in
terms of few universal laws and principles.
domain, called mesoscopic physics, which is 2. Some other people get excitement in carrying
intermediate between the microscopic and macro-
out new challenging experiments to unfold the
scopic domains and deals with a few tens or hundreds
mysteries of nature and in verifying or falsi-
of atoms.
fying the existing theories. .
11. The scope of physics is truly vast. Elaborate. 3. Applied physicists get great satisfaction when
The scope of physics is truly vast. We can see in they develop technologies for the welfare of the
another way. that the scope of physics is truly vast. It human beings just by using basic laws of physics.
1.4 PHYSICS-XI

13. What are the reasons behind the large scale planets and other heavenly bodies in the sky.
progress of physics in the last few centuries ? (ii) Radiotelescopes have helped to discover quasers,
Reasons behind the phenomenal growth of pulsars, etc., and can be used to see upto the farthest
physics in the last few centuries : limits of the universe. (iii) Dopler's effect predicted the
1. Precise and accurate measurements are central big-bang theory of the universe.
to the growth of physics because most of laws 5. Physics in relation to geology. Diffraction
of nature can be expressed in mathematical techniques help to study the crystal structure of various
forms. rocks. Radioactivity is used to estimate the age of rocks
2. The basic laws of physics are universal. The and fossils.

R
same laws can be applied in widely different 6. Physics in relation to seismology. The move-
domains and conditions. ment of the earth crust and types of waves so
3. Because of ability of the scientists to separate generated help a lot in the study of earthquake and its
the important and essential features from the effect.
less important ones, it is easier to understand
7. Physics in relation to meteorology. By studying

SI
even a highly complex phenomenon.
variation of pressure with temperature, we can forecast
1.8 PHYSICS IN RELATION TO weather.
OTHER SCIENCES
1.9 PHYSICS IN RELATION TO SOCIETY
14. How is the study of physics useful to the study of
other sciences ? 15. The fate of a society is linked to the develop-
Physics in relation to other sciences. Physics is ments in physics. Explain.
the most fundamental discipline of all sciences. It has Physics in relation to society. The fate of a
played a key role in the development of all other society is closely linked to physics. Whatever is dis-
IT
sciences. covered in physics, it immediately affects the society.
1. Physics in relation to mathematics. Physics is a For example:
quantitative science. Mathematics provides the necessary
1. The developments of telephone, telegraph, telex
signs and tools which the physicists use. It has played
enable us to quickly exchange messages between
an important role in the development of theoretical
far off places.
physics. Had Newton not invented calculus, he would
H
2. The discoveries of radio and television have
not have been able to discover the universal law of
made possible the instantaneous commu-
gravitation.
nication with other parts of the world.
2. Physics in relation to chemistry. In physics, we
3. The launching of satellites into space has
study the structure of atom, radioactivity, X-ray
revolutionised the concept of communication.
diffraction, etc. Such studies have enabled chemists to
O

arrange elements in the periodic table on the basis of 4. The development of alternative sources of
their atomic numbers. This has further helped to know energy is of great importance to the mankind.
the nature of valency and chemical bonding and to 5. Microelectronics, lasers, computers, super-
understand the complex chemical structures. conductivity and nuclear energy have entirely
3. Physics in relation to biological sciences. The changed the thinking and the living style of the
M

developments in life sciences owe a great deal to human beings.


physics. (i) Optical microscopes are extensively used in The modem science in general and physics in
the study of biology. (ii) With the help of an electron
particular has the potentiality to eradicate poverty and
microscope, one can study the structure of cell. (iii) The hunger from the surface of the earth and to usher in an
X-rays and neutron diffraction techniques have helped
age of prosperity: On the other hand, a wrong use of
in understanding the structure of nucleic acids, which
discoveries of physics may bring about a disaster. The
helped to control vital life processes. (iv) Radioisotopes
knowledge of physics can be applied alike, to the
are used in radiation therapy for the cure of deadly creation of heaven on the earth or to the' total
diseases like cancer. annihilation. Our wiseness lies in applying physics to
4. Physics in relation to Astronomy. (i) Astrono- solve the pressing problems the society faces and not
mical telescopes are used to study the motion of to annihilate it.
PHYSICAL WORLD 1.5

t,able 1.1 Some great phYSiCists and their discoveries

S.
No.
I Name of
Scientist
Country Discovery S.
No.
I Name of
Scientist
. Country Discovery
l. Abdus Salam America Unification of 25. Einstein, Albert Germany Theory of relati-
(Pakistan
born)
weak and
electromagnetic
I I vity, mass-energy
equivalence,
.

interactions I photoelectric
2. Alfred Noble Sweden Dynamite effect
Anderson C.D. America Positron 26. Edison, A America ' Electric bulb,
3.
telegraphy

R
4. Antony Hewert England Pulsars
27. Faraday, England Laws of electro-
5. Appleton, E. England Exploration of Michael magnetic
ionosphere induction, laws of
6. Ampere France Magnetism is due electrolysis,
to electric current designed electric
7. Archimedes Greece Principle of motor

SI
buoyancy, 28. Fermi, Enrico Italy Controlled
Principle of the nuclear fission
lever
29. Gabor, D. America Holography
8. Bardeen, John America Transistor,
superconductivity 30. Galileo, Galilei Italy Law of inertia

9. Bequerrel, France Discovery of 3l. Grahm Bel America Telephone


Henry A spontaneous 32. Hertz, Heinrich Germany ; Electromagnetic
radioactivity Rudolf waves
10. Bhabha, Homi India Cosmic ray 33. Hess, V.F. Austria Cosmic rays
Jehangir showers
34. Huygens, Holland Wave theory of
IT
11. Bose J.c. India Ultrashort Christiaan light
radiowaves
35. Hubble, Edwin America Expanding
12. Bose, S.N. India Bose-Einstein universe
statistics . Uncertainty
36. Heisenberg, Germany
13. Bohr, Niels Denmark Quantum model Wemes principle, Quantum
of hydrogen atom mechanics
14. Bragg, W.H. England Crystal structure 37. Kamerling Dutch Low temperature
H
by X-rays Onnes and liquid helium
15. Bragg, W.L. England Crystal structure 38. Kelvin (William England I Second law of
by X-rays Thomson) thermodynamics,
16. Barkla, England Number of thermodynamic
Charles G. electrons in an scale of
atom temperature
O

17. Cavendish England Experimental deter- 39. Landau, Lev Russia Theory of cond-
mination of 'G' Davidovich i ensed matter,
j Liquid helium
18. Cerenkov, P.A. Russia Cerenkov
40. Lawrence, E.O. America ' Cyclotron
radiations
4l. Lippman, G. France Colour
19. Chadwick, England Neutron
James photography
I
M

Compton, AH. America , Compton effect 42. Maxwell, James England Electromagnetic
20.
Cleric theory, Light as an
2l. Coulomb, France Coulomb's law electromagnetic
Charles A de wave
22. Curie, Marie Poland Studies on natural 43. Michelson, AA America Precision optical
Sklodowaska radioactivity, I instruments
Discovery of
, radium and
44. Marconi Italy I Wireless
telegraphy
polonium
45. Millikan, R.A America ' Measurement of.
23. de-Broglie, France Wave nature of charge on an
Louis Victor matter electron
24. Dirac, Paul England Relativistic theory 46. Newton, Isaac England I Law of gravitation,
of electron, r I Laws of motion,
Quantum statics Reflecting telescope
1.6 PHYSICS-XI

S. Name of Discovery
1. Electromagnetic waves are used in radio,
Country
No.
47. I Scientist
Oersted France I Magnetic effect of
current
television, radar and wireless communication.
2. Newton's concept of gravitation is used in
geostationary satellites which help us in fore-
48. Planck, Max Germany Quantum theory
casting weather and in geophysical survey.
49. I Pauli, W. America
I
of radiation
Exclusion
principle for
3. X-rays are used in radiotherapy, in detecting
fractures or dislocations in bones, in studying
filling electrons in crystal systems, in engineering and industry.
orbitals
4. The study of thermodynamics has helped to
SO. Raman, C.V. India Inelastic scattering

R
I of light (Raman design heat engines which have revolutionised
I I
I
51. Ramachandran, j India
G.N.
I
I cffect)
Triple helical
structure of
the industry.
5. The study of electricity has led to the develop-
ment of electric appliances like electric motors
proteins
and generators which are the backbones of
'1 molecules

SI
industry.
52. Robert Boyle 1 England Boyle's.law
53. Robert Hooke England Elasticity of 6. Nuclear power stations based on nuclear fission
solids, balance constitute one of the major sources of energy.
wheel of a watch
54. II Rutherford,
Ernest
England I Nuclear model of
atom
7. The concepts of modem electronics find exten-
sive use in telephone exchanges, robots, etc.
8. The discovery of silicon chips has brought a
55. Roentgen, W.K. Germany I X-rays
revolution in computer industry.
56. S. America Structure and
Chandershekhar (India evolution of stars, 9. Geothermal energy, i.e., the heat in the depth of
born) I Chandrashekhar
IT
the earth is being used these days. The tidal
limit energy in the oceans and solar energy too can be
57. SchrOdinger, E. Germany Wave mechanics converted into other forms of energy and used.
58. I Saha, M.N. India Thermal ionisation
10. Radioactive isotopes are now being widely used
59. Thomson, J.J. England I Electron in medicine, agriculture and industry.
60. Thomson, G.P. England I Diffraction of
electrons by These technologies have made our lives comfor-
H
I crystals table and materially prosperous.
61.
62.
Townes, C.H.
Van der Walls,
I
America
Dutch
I Maser, Laser
Expansion of .
Sometimes physics generates new technology. At
other times technology gives rise to new physics. Both
J.D. I gases and liquids have a direct impact on society.
63. Von Laue, Max Germany Diffraction of
X-rays in crystals
l:.,able 1.2 Some important technologies and
O

64. Volta Italy Discovered first


battery . their links with physics
65. Well, W. Germany Laws of radiation
of heat Technology Scientific Principle(s)
66. Watt, James England I Steam Engine Steam engine Laws o( thermodyna.~cs __
67. Yukawa, I Japan I Meson theory of Nuclear reactor
- - Controlled nuclear fission
M

-
Hedeki nuclear forces
Radio and Generation, propagation and detec-
Television tion of ~~ctromagn~tic waves
-
1.10 .•. PHYSICS IN RELATION TO TECHNOLOGY Wireless telegraphy Propagation of electromagnetic
waves
16. Technological advancements owe a great deal to
S?~uters Digital logic ~ electronic circuits _
the developments in physics. Briefly explain.
Lasers Light amplification by stimulated
Physics in relation to technology. The applica- emission of radiation (population
tions of physics have played a key role in the develop- inversion)
ment of technology. Today we see the applications of Production of ultra Superconductivity
physics in every walk of life. Some of the major high magnetic
fields
technologies based on the applications of physics are
as follows: Rocket propulsion Newton's laws of motion
PHYSICAL WORLD 1.7

Technology Scientific Principle(s) law of gravitation, the gravitational attraction between


Electric generator Faraday's laws of electromagnetic two bodies of masses 11"J. and "2 and separated by
induction distance r is given by
Hydroelectric Conversion of gravitational
power potential energy into electrical
energy
Aeroplane Bernoulli's principle in fluid where G is the universal gravitational constant.
dynamics __ Important properties of gravitational force:
Particle accelerator / Motion of charged particles in
1. It is a universal attractive force.
Cyclotron electromagnetic fields
2. It is directly proportional to the product of the

R
Sonar Reflection of ultrasonic waves
masses of the two bodies.
Optical fibres _ Total internal reflection of light
3. It obeys inverse square law.
Non-reflecting Thin film optical interference
coatings _ _ _ _ _ 4. It is a long range force and does not need any
Electron microscope ':\':ave nature of electrons
intervening medium for its operation.

SI
Photocell Photoelectric effect S. Gravitational force between two bodies does
not depend upon the presence of other bodies.
Fusion test reactor Magnetic confinement of plasma
(Tokamak) 6. It is the weakest force known in nature.
Giant Metrewave Detection of cosmic radio waves 7. It is a central force (i.e., it acts along the line
Radio Telescope joining the centres of the two bodies).
(GMRT)
8. It is a conservative force (i.e., work done in
Bose-Einstein Trapping and cooling of atoms by
laser beams and magnetic fields
moving a body against the gravitational force is
condensate
path independent).
IT
1.11 FUNDAMENTAL FORCES IN NATURE 9. Gravitational force between two bodies is
thought to be caused by an exchange of a
17. Name the four basic forces in nature. Arrange
particle called graviton.
them in the order of their increasing strengths.
Examples of gravitational force:
Fundamental forces in nature. In the macroscopic
1. All bodies fall because of the gravitational force
world, we observe several kinds of forces : muscular
of attraction exerted on them by the earth.
H
force, contact forces of support and friction, forces
exerted by springs and strings, viscous forces, electric 2. Gravitational force governs the motion of the
forces, magnetic forces, etc. All these forces between moon and the artificial satellites around the earth ;
macroscopic objects arise from two fundamental forces: and the motion of the planets around the sun.

1. Gravitational force 2. Electromagnetic force 3. Gravitation plays a key role in the formation and
evolution of stars, galaxies and galactic clusters.
O

In the microscopic world, in addition to the above


two forces, two more basic forces are required to 1.13 THE ELECTROMAGNETIC FORCE
account for the various atomic and nuclear processes.
19. What is electromagnetic force ? Mention its
These are
important properties. Give some examples of the electro-
1. Strong nuclear force 2. Weak nuclear force magnetic force.
M

The ratio of the strength of the four fundamental


Electromagnetic force. The force acting between two
forces in nature is
electric charges at rest is called electrostatic force. According
Fe : Fw : FE : Fs = 1: 10 25 : 10 36 : 10 38 to Coulomb's law, the magnitude of the electrostatic
force.F between two point charges ql and q2 separated
1.12 THE GRAVITATIONAL FORCE by distance r in vacuum is given by
18. What is gravitational force? Mention its important F __ 1_ qlq2
properties. Give some examples of gravitational force. - 41t EO' ,z
Gravitational force. It is the force of mutual where EO is the permittivity of vacuum. The force acting
attraction between two bodies by virtue of their masses. It is between two magnetic poles is called magnetic force. In fact,
a universal force. Every body attracts every other body electrostatic and magnetic forces are closely inter-
of the universe with this force. According to Newton's related. For example, a moving charge produces a
1.8 PHYSICS-XI

magnetic field. Also, a magnetic field exerts a force on Generally, these forces act normal to the surface of
a moving charge. This force depends both on the contact and are of pushing or repelling nature. For
magnitude and direction of the velocity of the electric example, a book lying on a table pushes it downwards
charge. Thus the electrostatic and magnetic forces are while the table pushes the book upwards.
inseparable and are considered as the two facets of a (ii) Force of friction. Sometimes, the electro-
general force known as electromagnetic jorce. magnetic contact force between two bodies may have a
Important properties of electromagnetic force: component acting parallel to the surface of contact. This
,'1. Electromagnetic force may be attractive or component is called friction. When bodies are placed
with their smooth surfaces in contact, they provide
repulsive. Like charges repel each other and
only a small parallel component of contact force and
unlike charges attract each other. hence friction between them is negligibly small. For

R
2. It obeys inverse square law. example, when we climb up a tree, its rough trunk
3. It is a long range jorce and does not require any provides sufficient frictional force parallel to the surface
intervening medium for its operation. of the tree which helps us cling on to the trunk. It is
4. It is a central jorce. difficult to climb up a smooth metallic lamp post because
it does not provide enough parallel frictional force.

SI
5. It is a conservative jorce.
6. It is 10 36 times stronger than the 'gravitational I (iii) Eldas(tiCforce in da)~pring. Wh en a fSPlrin.g is
e ongate or compresse ,It exerts a orcef 0 e asticity
force.
- which arises due to the net attraction (or repulsion)
7. It is caused by the exchange of photons (y) between the neighbouring atoms of the spring. This
between two charged particles. net attraction (or repulsion) is the (unbalanced) sum of
Examples of electromagnetic force: the electrostatic forces between the electrons and nuclei
1. When a spring is compressed/elongated, it exerts of these atoms.
a force of elasticity due to the net repulsion / (iv) Van der Walls' force. The Van der Walls' force
IT
attraction between its neighbouring atoms. This between two neutral molecules of a gas is not a funda-
net repulsion or attraction is the sum of the mental force but a derived force. It is the net residual
electrostatic forces between the electrons and force obtained by summing up the fundamental
nuclei of the atoms. electrostatic forces between the various electrons and
nuclei of the two molecules.
2. The Van der Walls' force between two neutral
molecules of a gas is net sum of the electrostatic (v) Force in a taut string/rope. Consider a block
forces between the electrons and nuclei of the tied to the lower end of a string suspended from a rigid
H
two molecules. support. The string is in a state of tension. The electrons
and protons of the lower end of the string exert
1.14 V ELECTROMAGNETIC NATURE OF electrostatic forces on the electrons and protons of the
SOME MACROSCOPIC FORCES block. The resultant of these forces balances the weight
of the block. Generally, a string under tension exerts an
Introduction. The macroscopic forces of our daily electromagnetic force of pulling nature on the two
O

life such as tension, friction, contact force, spring force, bodies attached to its two ends.
etc., are derived from the fundamental electrostatic
21. Electromagneticjorce is enormously stronger than
force only. All matter consists of charged particles like
the gravitational jorce. Give an example from daily life
electrons and protons. The strong electromagnetic
to illustrate it.
force between these particles is responsible for the
M

structure of atoms and molecules, rate of chemical Electromagnetic force is much stronger than the
reactions, and the mechanical, thermal and electrical gravitational force. When we hold a book in our
hand, the earth exerts a very large gravitational force
properties of materials.
on the book due to its huge mass. This force is balanced
20. Discuss the electromagnetic nature of (i) contact by the normal force exerted by our hand. But the latter
force between two bodies (ii) force of friction (iii) elastic force is the net electromagnetic force between the
force in a spring (iv) Van der'Walls' force between gas charged constituents of our hand and the book at the
molecules and (v) force in a taut string/rope. surface of contact. Clearly, electromagnetic force is
(i) Contact force between two bodies. When we much stronger than the gravitational force. .
place two bodies in contact with each other, their 22. Although gravitational jorce is incomparably
atoms come close to each other at the surface of weaker than the electromagnetic jorce, yet it governs the
contact. Large electromagnetic forces begin to act large scale motion both on terrestrial and astronomical
between the charged constituents of these atoms. scales. How ?
PHYSICAL WORLD 1.9

Gravitational force governs the large scale 2. Radioactivity occurs in heavier nuclei because---'
motion. Mass is only of one type i.e., there are no of insufficient nuclear force between their
positive and negative masses. So the gravitational protons and neutrons.
force is always attractive. But electric charges may be 3. The concept of nuclear force is useful in
positive and negative. Consequently, the electro- obtaining nuclear energy via the processes of
magnetic forces may be both attractive and repulsive. nuclear fission and fusion.
Between two neutral objects, the gravitational force
• Electrons do not experience the strong
goes on adding over all pairs of particles of the two
nuclear force.
objects while the electromagnetic forces being equally
attractive and repulsive, add up to zero. That is why 1.16 THE WEAK NUCLEAR FORCE

R
the large scale motion in the universe is controlled by 24. What is weak nuclear force ? Explain with the
the weakest gravitational force. help of an example. Give important properties of weak
nuclear force.
1.15 THE STRONG NUCLEAR FORCE
Weak nuclear force. It is the force that appears only
23. What is strong nuclear force ? Mention its between elemeniaru particles involved in a nuclear process

SI
important properties. Give some examples of this force. such as the I3-decayof a nucleus. In a I3-decay, the nucleus
Strong nuclear force.The strong attractive force emits an electron and an uncharged particle called
which binds together the protons 'and neutrons in a nucleus neutrino. The electron and neutrino interact with each
is called strong nuclear force. This force cannot be other through the weak nuclear force. The weak
electrostatic force because positively charged protons nuclear force is much stronger than the gravitational
strongly repel each other at such small separations of force, but much weaker than strong 'nuclear and
the order of 10 -15 m. Also the gravitational attraction electromagnetic forces. This is obvious from the fact
between two protons being much weaker, cannot that the decay of an elementary particle caused by
overcome this electrostatic repulsion. So a new attractive weak nuclear force (e.g., the decay of a pion to a muon
IT
force must be acting between the nucleons (protons and a neutrino) is much slower than the decays caused
and neutrons). This strong nuclear force is strongest of by strong nuclear or electromagnetic forces.
all the fundamental forces, about 100 times stronger Important properties of weak nuclear force:
than the electromagnetic force. 1. Any process involving neutrino and antineu-
Important properties of strong nuclear force: trino is governed by weak nuclear force beacuase
these particles can experience only weak interac-
1. It is the strongest interaction known in nature,
H
tion and not the strong nuclear interaction.
which is about 10 38 times stronger than the
2. Weak nuclear force is 1025 times stronger than
gravitational force.
the gravitational interaction.
2. It is a short range force that operates only over
3. It operates only through a range of nuclear size
the size of the nucleus (= 10- 15 m).
("" 10-15 m).
3. It is basically an attractive force, but becomes
O

4. The messenger particles that transmit the weak


repulsive when the distance between the
force between elementa7 particles are the
nucleons becomes less than 0.5 fermi (1 fermi
massive vector bosons (W , Z).
=10-15 m).
4. It varies inversely with some higher power (> 2)
of distance. ..., Neutrino is an elementary particle with zero rest mass
M

5. It is a non-central and non-conservative force. and zero charge.


6. It has charge independent character i.e., nuclear ..., It will be more correct to say that /3- decay ejects an
forces between proton-proton, proton-neutron electron and an antineutrino while /3+decay ejects a
positron (antielectron) and' a neutrino.
and neutron-neutron are almost equally strong.
.•.., Elementary particles are grouped as baryons,
7. It is caused by the exchange of particles, called
mesons, leptons. Nucleons (protons and neutrons)
Il-mesons, and their higher counterparts are called baryons. The
Examples of nuclear force: pions, kaons, etc., are mesons. Baryons and mesons
1. Nuclear forces bind together the protons and are together called hadrons which can interact
through strong nuclear force. Electrons, neutrinos
neutrons in the nuclei, So they are responsible
and their higher mass counterparts are called leptons.
for stability of nuclei and hence of the atoms and
Leptons do not experience the strong force, only the
ultimately of all matter that exists in the universe.
weak force is dominant between them.
1.10 PHYSICS-XI

1.17 .•. BASIC MECHANISM OF THE l:,able 1.3 Some of the important aspeds of
FUNDAMENTAL FORCES fundamental forces in nature
25. Briefly explain the mechanism that gives rise to
lunda- Particles
the various fundamental forces between elementary mental
Relative
Range on which Messenger
particles. Give the names of the exchange particles for strength particle
force force acts
each of these forces. ' Gravi- 1 Infinite All Gravitons
Fundamental forces arise due to exchange of particles. tational particles
force
Each fundamental force between two elementary particles --
arises from the exchange of its characteristic particles called Weak 1025 Very short, Elemen- Vector
nuclear within tary bosons
the quanta of that force or the messenger particles. To

R
force nuclear size particles
'-understand this mechanism, consider interaction bet-
- l::10-15m -
ween two persons A and R Suppose A throws a ball Electro- 1036 Infinite Charged Photons
and B catches it. By momentum conservation, A suffers magnetic particles
a recoil when he throws the ball. Similarly, B suffers a force
- --38 - - - - --
recoil when he catches the ball. Also the recoil force Strong 10 Very short, Nucleons Mesons

SI
nuclear within
changes the momentum of each person.
force nuclear size
The exchange particles for various fundamental (""to-15m)
forces are as follows:
1. Electromagnetic force between two charged For lour Knowledge
- particles (like electrons) arises from the exchange of
.\. The strong nuclear force is not a true' fundamental
photons (y) between them, as shown in Fig. l.1(a).
.force. The nucleons themselves are builtof subunits
These photons are emitted by one electron and called quarks. The quark-quark force; which binds
absorbed by the other within a very short time. As the nucleons together is now regarded as fundamental
IT
these, photons cannot be detected, so they are called force. It is caused by the exchange of massless
virtual photons. particles called gIuons.
2. Weak nuclear force during the ~ -decay of a
nucleus arises from the exchange of massive' particles, 1.18 .•. UNIFICATION OF FORCES
called vector bosons (W ± , Z) between the elementary
26. What is the basic quest of modern physicists ?
particles. As shown in Fig. 1.1(b), the charge of vector Mention the various significant attempts made towards
H
boson (W -) is exchanged in the ~ -decay of a neutron. the unification of forces in a chronical order.
3. Strong nuclear force between two nucleons arises Basic quest of modern physicists is the
from the exchange of mesons between them. For unification of forces. The great advacements in
example, negatively charged pior. (1t-) is exchanged in ' physics are the result of unification of different
the nuclear force inyolved in np" pn scattering, "as
theories and domains. Different attempts made by the
O

shown in Fig. l.1(c).


distinguished physicists from time to time in persuing

!.-----0::::::::- the goal of unification of forces are listed in Table 1.4.

~-H:- n
W

.
v, . l:,able 1.4

Some of the milestones towards the
uniffcation of forces
M

(a) Electromagnetic (b) Weaknuclear


Physidst( s) Achievement In
Year
unification
,,~ . ~--<:-} Issac Newton 1687 Unified celestial and terres-
,/

Tn
(c)
(\.
'Y
Strong-nuclear
trial domains by explaining
both types of phenomena by
applying the same laws 'of
motion and law of gravitation.
Fig. ,1.1 Origin of fundamental forces due to Hans Cristian 1820 Showed experimentally that
exchange of particles. Oersted electric' and magnetic
Michael 1830 phenomena are inseparable
4. Gravitational force is believed to be caused by the Farady aspects of a unified domain
exchange of yet to be detected particles called gravitons. called electromagnetism.
PHYSICALWORLD 1.11

Physicist(s) Achievement in extreme position. The energy changes from potential


Year
unification to kinetic as the bob moves from either extreme
James Clerk 1873 Unified electromagnetism and position to the mean position. At all points of its
Maxwell optics by discovering that light motion, total energy of the bob remains constant.
~ an ~lectromagn~tic wave. __
Albert 1905- Asserted the equivalence of
Einstein 1916 mass and energy and uni- fied •• •
the motions of space, time and .•.
\. When a body moves under a conservative force, its
gravitation in his theories of total mechanical energy is conserved. In the presence
~lativity-=- _ _ __ __ of a non-conservative forces such as friction or air
resistance, mechanical energy is not conserved. It

R
Sheldon 1979 Showed that the weak
Glashow, nuclear force and the changes into heat, sound, etc.
Abdus Salam, electromagnetic force are AI Mechanical energy is conserved whether acceleration
Steven different aspects of a unified is constant or variable.
\yeinberg _ electro-weak force
-- - AI Inspite of all kinds of violent phenomena occurring in
Carlo Rubia, 1984 Experimentally verified the the universe all the time, the total energy of the

SI
Simon theory of electro-weak universe remains constant. In fact, universe is an
Vander Meer force.
example of the most ideal isolated system possible .
.•.\. According to Albert Einstein, mass and energy are
1.19 CONSERVATION LAWS interconvertible. In 1905, he established the mass-
27. What are conserved quantities and conservation energy equivalence. The energy associated with mass
laws? mis given by
Conservation laws. In any physical process involv- E =mc2
ing the different forces, some physical quantities remain un- where c is the speed of light in vacuum.
.•.
\. As mass can be converted into energy, so in law of
IT
changed with time. Such quantities are called conserved
quantities. The laws which govern the conservation of these conservation of energy, we include mass also.
quan,tities are called conservation laws.
In classical physics, we usually deal with the 30. State the law'of conservation of linear momentum.
following four conservation laws: Give some examples in which this law is obeyed.
1. Law of conservation of energy. Law of conservation of linear momentum. This
law states if no external force acts on a system, then its linear
H
2. Law of conservation of linear momentum.
3. Law of conservation of angular momentum. momentum remains constant.
4. Law of conservation of charge. Examples: (i) A rifle gives backward kick on firing
a bullet. Before firing, both the bullet and the rifle are at
28. What is an isolated system ?
rest and initial momentum of the system is zero. As
Isolated system. Any system (assembly of particles or
O

soon as bullet is fired, it moves forward with a large


bodies)on which no externalforce acts is calledan isolatedsystem.
velocity. In order to conserve momentum, the rifle
1.20 LAW OF CONSERVATION OF ENERGY moves backward with such a velocity that the final
momentum of the system is zero.
29. State the law of conservation of energy. Give
some examples in which this law is obeyed. . (it) Suppose a radioactive nucleus, initially at rest,
M

decays spontaneously into fragments. To conserve


Law of conservation of energy. This law states that
energy can neither be created nor destroyed but it can be 'momentum, the heavier and lighter fragments will fly
changed from one form to another. Equivalently, we can say in opposite directions, with the former having a
that the total energy of an isolated system remains constant. proportionately smaller speed than the latter.
Examples : (i) When a body falls freely, under 1.21 LAW OF CONSERVATION OF
gravity, its potential energy gradually changes into
ANGULAR MOMENTUM
kinetic energy. But its total mechanical energy (kinetic
energy + potential energy) remains constant at any 31. State the law of conservation of angular momen-
point of its motion. tum. Give some examples in which this law is obeyed.
. (ii) During the oscillation of a simple pendulum, Law of conservation of angular momentum. A
the energy of the bob changes gradually from kinetic to body rotating about an axis has a rotational inertia,
potential as it moves from mean position to either called moment of inertia. Also, it is associated with a
1.12 PHysiCS-xr

momentum, called angular momentum. We shall prove


later on that
.•., In classical physics, we df'jl with the four conservation
Angular momentum (L) = Moment of inertia (I) laws of energy, momentum, angular momentum and
x angular speed (00) charge. But in nuclear and particle physics, we also
deal with the conservation of quantities like parity,
The law of conservation of angular momentum states baryon number, strangeness, hypercharge, etc.
that if no external torque acts on a system, then its angular J
..•.
, A conservation law is a hypothesis, based on observa-
momentum remains constant. tions and experiments. A conservation law cannot be
Examples: (I) While revolving in its elliptical orbit, proved. It can only be verified, or disproved, by
when a planet approaches the sun, its moment of experiments.

R
inertia about the sun decreases. To conserve the .•.
, Some conservation laws may hold for one funda-
angular momentum, its angular speed increases. mental force but not for the other. For example, parity
(il) In a Tornado as the air rushes towards the is conserved by the strong and electromagnetic forces
centre, its moment of inertia decreases. To conserve the but not by the weak force. Also, strangeness is con-
served by the strong force but not by the weak force.
angular momentum, the angular speed of the air

SI
increases. 1.23
RELATION BETWEEN CONSERVATION
1.22 LAW OF CONSERVATION OF CHARGE LAWS AND SYMMETRIES OF NATURE
32. State the law of conservation of charge. Give 33. How are the conservation laws related to the
symmetries of the nature ?
some examples in which this law is obeyed.
Conservation laws are closely related to the
Law of conservation of charge. This law states that
symmetries of the nature. The symmetries of space, time
the total charge of an isolated system remains constant. This
and other types of symmetries have played an important role
implies that the electric charges can neither be created
in.deoeloping the modern theories of fundamental forces.
IT
nor destroyed, only they can be transferred from one
(i) If we perform an experiment at a certain place
body to another.
today and repeat the same experiment after one year at
Examples: (i) When a glass rod is rubbed with, silk the same place, we obtain exactly the same results. This
cloth, both develop charges. It is observed that the symmetry of nature with respect to translation or
positive charge developed on the glass rod has the displacement of time is called homogeneity of time and it
same magnitude as the negative charge developed on leads to the law of conservation of energy.
H
silk cloth. So total charge after rubbing is zero as before (ii) Laws of nature take the same form everywhere
rubbing i.e., electric charge is conserved. in the universe i.e., there is no preferred location in the
d1i) Electric charge is conserved during the fission of universe. This symmetry of the laws of nature with
a 92 U nucleus by a neutron. respect to translation in space is called homogeneity of space
and gives rise to the law of conservation of linear momentum.
O

In + 235U ~ 14IBa + 97.Kr + 3In + Energy


o 92 56 36 - 0
(iii) Isotropy of space (i.e., there is no preferred
Total charge before fission (0 + 92) direction in space) gives rise to the law of conservation
= Total charge after fission (56 + 36 + 3' x 0) of angular momentum.
M

Very Short Answer Conceptual Problems

Problem 1. Why do we call physics an exact science? life science. So many authorities in the other fields of
[Himachal O6C,08] knowledge consider physics to be the basics of all sciences.
Problem 3. Why was science called natural philo-
Solution. It is because of high precision and accuracy sophy in earlier days ?
with which the physical quantities are measured in
Solution. This is because in earlier days, scientific
physics. knowledge was gained from the direct study of natural
Problem 2. Why is physics regarded as basic science? phenomena without any experimentation.
Problem 4. Should a scientific discovery which has
Solution. Physics has played a key role in the deve- nothing but dangerous consequences for mankind be
lopment of physical sciences, technology, medicine and made public?
PHYSICAL WORLD 1.13

Solution. Science is the search for eternal truth. Any Problem 9. How is science different from
discovery good or bad must be made public. A discovery technology ?
which appears dangerous today may prove useful to the Solution. Science is the study of natural laws while
mankind later on. However, a strong public opinion should technology is the practical application of these laws to the
be built up against the misuse of a dangerous discovery. daily life problems.
Problem 5. Is science not on speaking terms with Problem 10. The physicists think at a level far
huminities? Comment. higher than a normal individual. Is it true ?
Solution. No, science helps in the growth of humini- Solution. Yes, the physicists are always ahead of
ties by preserving old manuscripts and articles of historical their times. In order to predict events much in advance,

R
values, enriching music, etc. they have to think far higher than normal individual.
Problem 6. Is physics more of a philosophy or more Problem,l1. What is gravitational force?
of a mathematical science ? [Himachal 06J
Solution. Physics is more of a philosophy than a Solution. It is the force of mutual attraction
mathematical science. In fact, understanding and appre- between two bodies by virtue of their masses.

SI
ciation of physics without a philosophica! outlook is
Problem 12. What is electromagnetic force?
incomplete.
[Himachal 06J
Problem 7. Does imagination play any role in
physics? [Himachal 07CJ Solution. It is the force due to interaction between
two moving charges. It is caused by exchange of
Solution. Yes, imagination has played an important
photons (y) between two charged particles.
role in the development of physics. Huygen's principle,
Bohr's theory, Maxwell equations, Heisenberg's uncer- Problem 13. What is nuclear force? [Himachal06J
tainty principle, etc. were the imaginations of the scientists Solution. It is the strongest attractive interaction
IT
which successfully explained the various natural which binds together the protons and neutrons in a
phenomena. nucleus.
Problem 8. What are the similarities between Problem 14. What is weak nuclear force?
science and arts ?
Solution. It is the forcethat appears only between
Solution. Both science and arts are creative. Both
elementary particles involved in a nuclear process such
portray realm of experience.
as I3-decayof a nucleus.
H
Short Answer Conceptual ProbleIlls

Problem 1. A theory is not scientific if it is not and electromagnetic forces are sufficient to explain these
falsifiable. This means that ifa theory is so constructed macroscopic phenomena. On the other hand, quantum
O

that it can accommodate any prediction or observation, mechanics deals with microscopic phenomena at the
it is not science. Use this strong argument (and any minute scales of atoms, molecules and nuclei. Here the
strong and weak nuclear forces become dominant.
additional arguments you can think of) to challenge
Astrology's claim to being a science. Problem 3. Discuss the relation of physics to
chemistry. [Himachal 07J
Solution. Astrology predicts events on the basis of
M

positions of different planets at different times. It predicts Solution. Refer to the answer of Q. 14 on page 1.4.
the same fate for all children born at the same time which Problem 4. Discuss the relation of physics With
is not found to be true. An astrologer can make any technology. [Himachal 05, 07, 07CJ
prediction unchallenged. So astrology is not based on Solution. Refer to answer of Q. 16 on page 1.6.
scientific laws. It cannot be a science.
Problem 5. The fate of society is linked to the
Problem 2. Distinguish between classical physics developments in physics. Comment.
and quantum mechanics. [Himachal 06C, 07C, 08CJ
Solution. Classical physics mainly deals with Solution. Refer to the answer of Q. 15 on page 1.4.
macroscopic phenomena which may be at the laboratory,
Problem 6. What is the role of physics in your daily
terrestrial and astronomical scales. Here the particle size
> 10- 8 m and particle velocity« 108 m / s. Here we need life? ,'z [Himachal 03, 05, 08J

not consider strong or weak nuclear forces. Gravitational Solution. Refer to the answer of Q.16 on page 1.6.
1.14 PHYSICS-XI

guideline. toNCERT Exercises •


1.1. Some of the most profound statements on the nature of 1.5. No physicist has ever "seen" an atom. Yet, all physicists
science have come from Albert Einstein, one of the greatest believe in the existence of atoms. An intelligent but super-
scientists of all time. What do you think did Einstein mean stitious man advances this analogy to argue that 'ghosts' exist
when he said : "The most incomprehensible thing about the though no one has 'seen' one. How will you refute his argument?
world is that it is comprehensible" ?
Ans. Many phenomena which are based on the
Ans. The physical world is highly complex. It has vast
assump- tion of existence of atoms have been theoretically

R
orders of magnitude in respect of mass, length and time.
predicted and experimentally verified. These phenomena
Even then all the physical phenomena can be explained in
indirectly establish the existence of atoms. But no such
terms of few basic principles of physics. That is, it is possible
to comprehend nature in terms of few basic laws inspite of phenomenon has been observed which proves the
its vast complexities. This is what Einstein meant to say. existence of ghosts or which can be explained on the
assumption of ghosts.
1.2. Every great physical theory starts as a heresy and ends

SI
as a dogma." Give some examples from the history of science of 1.6. The shells of crabs found around a particular coastal
the validity of this incisive remark. location in Japan seem mostly to resemble the legendary face of a
Ans. Any opinion against the conventional belief is a Samurai. Given below are two explanations of this observed fact.
heresy, while dogma is an established belief. The Which of these strikes you as a scientific explanation ?
geocentric theory of Copernicus started as a heresy but it (a) A tragic sea accident several centuries ago drowned a
ended as dogma when his theory was explained by Tycho young Samurai. As a tribute to his bravery, nature
,Brahe and [ohnes Kepler later on. through its inscrutable ways immortalised his face by
1.3. "Politics is the art of the possible". Similarly; "Science imprinting it on the crab shells in that area.
is the art of the soluble". Explain this beautiful aphorism on the
IT
(b) After the sea tragedy, fishermen in that area, in a gesture
nature and practice of science.
of honour to their dead hero, letfree any crab shell caught
Ans. In politics, everything is possible. The politi- by them which accidentally had a shape resembling the
cians do not follow any principle, discipline or norm.
face of a Samurai. Consequently, the particular shape of
They tend to remain in power by any means, fair or foul.
the crab shell survived longer and therefore in course of
But a scientist observes a phenomenon carefully and
patiently, collects data, analyses' it and formulates time the shape was genetically propagated. This is an
example of evolution by artificial selection.
H
prindples, thus solving the mystery of nature.
:, 1.4. Though India has a large base in science and technology Ans. Explanation (b) is a scientific explanation to the
which is fast expanding, it is still a long way from realising its fact that the shells of crabs resemble the face of Samurai.
potential of becoming a world leader in science. Name some 1.7. The industrial revolution in England and Western
<important f4ctors which in your view have hindered the
Europe more than two centuries ago was triggered by some key
ad?ancemeizt of science in India.
O

scientific and technological advances. What were these advances?


Ans, There are J;llanyfactors which have hindered the
advancement of science in India. Some of these are: Ans. Some of the key advances responsible for
industrial revolutions were as follows:
, ':'. 1.,' There is lack of clear cut policies at the political
, .level. 'There is undue political interference and 1. Invention of steam engine by James Watt in 1769
bureaucratisation in' the management of science Ap made it possible to drive machines by using
M

and technology. steam power.


2. Due to excessive population, our country cannot 2. Invention of flying schuttle by John Key and of
afford to expend sufficient funds on science and power-loom by Cartright brought revolution in
technology , textile industry.
3. Talented scientists and technologists are not 3. Setting up of blast furnace helped to convert low
, " encouraged to persue research work in the home- grade iron into steel.
land. instead, they prefer to migrate to developed 4. Invention of safety lamp by Humphry Davy
countries where they get better salaries and helped work safely in mines.
research facilities.
1.8. It is often said that the world is witnessing now a
4. There is lack of coordination between researchers
and industraliasts. Industralists like to borrow second industrial revolution, which will transform the society
technology from ad~anced countries rather than to as radically as did the first. List 'so'me key contemporary areas of
use indigeneous technology. science and technology which are responsible for this revolution.
PHYSICAL WORLD 1.15

Ans. The key areas of science and technology which may (c) Prenatal sex determination.
radically transform the present society are as follows : , (d) Computers for increase in work efficiency,
1. Laser technology which is used for bloodless (e) Putting artificial satellites into orbits around the Earth ..
surgery and for effective automatic control of if> Development of nuclear weapons.
rockets and satellites. (g) Development of new and powerful techniques if
2. Fabrication of superconductors near the room chemical and biological warfare.
temperature will help to. transmit electric po.wer
(h) Purification of water for drinking.
without any wastage of energy. '
(i) Plastic surgery.
3. The extensive use of computers will increase work
(j) Cloning.

R
efficiency .
4. Biotechnology. Ans.
(a) Good, mass vaccination protects us from this
1.9. Write in about 1000 words afiction piece based on your
speculation on the science and technology of the twenty-second drea'ded disease.
century. (b) Good, it educates,' entertains and creates
awareness amongst the people.

SI
Ans. These days, we go.to.distant places by means of a
car or an aircraft fuelled by petrol. In the twenty-second ,(c) It cannot be clearly catego.rised as it may be
century, we may plan for a journey to.a distant star located misused by people, -t-,

hundreds of light years away from the earth by means of a (d) Good, computers work very fast and with great
spaceship, without taking care of any fuel needs. accuracy.
The spaceship is sent into.space by firing a rocket engine (e) Good, artificial satellites are useful for studying
from a launching pad. As it enters the region of magnetic universe, as communication means and in
field in space, it is propelled by electricity generated due forecasting weather.
to. electromagnetic induction. The current induced is fed if> Bad, nuclear weapo.ns are weapo.ns of mas's
destruction,
IT
to. the electric motors via superconducting wires. This
avoids wastage of electric energy in the form of heat. (g) Bad, these techniques are the weapo.ns of mass
Now suppo.se the spaceship enters a region in space destruction,
where the temperature is very high. The connecting wires (h) Good, otherwise the polluted water may cause
at once lose the superconducting properties. This may many diseases. '
cause a panic in the spaceship as no. po.wer is available. (i) Good, it helps to.remove deformations in the body,
Another spaceship containing both matter and antimatter (]) Good.
H
in separate chambers may come to. its rescue. Hence the
spaceship may continue its journey to.distant star without 1.12. India has had a long and unbroken tradition of great
bothering about any fuel crisis. scholarship - in mathematics, astronomy, linguistics,' logic and
ethics. Yet, in parallel with this, several superstitious and
-.1.10•. Attempt to Jc1ffiiUfate' your .'moral views on the
r
obscurantistic attitudes and practices flourished in our soc,iety,
·practic~ence .. Imagi~e, yourself stumbling upon a
and unfortunately continue even today - among many educated.
O

discouerv, which has' great academic interest but is certain to


people too. How will you use your knOwledge' of. science to
have nothing but dangerous consequences for the human develop str~tegies tocounter these attitudes? .'
society. How, if at all, will' you resolve your dilemma ? .
Ans, Educating the common man is the only way..to.
Ans. Science is the search for eternal truth. It is the get rid of superstitious and obscurantistic attitudes and
moral duty of a scientist to.expose the truth. If there is any practices flourished in our society. The phenomena which
danger to.the mankind from a discovery, he should try to. people attribute to. evil spirits and- gods should be
M

build a public opinion against the misuse of discovery. explained scientifically through mass media such as
, Moveover, he should develop .the means to. prevent its radio, television, newspapers,' cinema, etc. The school
misuse. It is equa:lly possible that this' discovery may students must be explained the day to. day phenomena
pro.ve to.be of immense 'importance to.the mankind later on, occurring around them in an effective scientific manner ..
1.11. Science, like any knowledge, can be put to good or bad use, 1.13. Though the law gives women equal status in' India;
depending on the user. Given below are some of the applications of many people hold unscientific views on a woman's innate
science. Formulate your views on whether the particular application nature, capacity and intelligence, and in pr.actice give- them a'
is good, bad or something that cannot be so clearly categorised : secondary status and role. Demolish this "view using scientific:
(a) Mass vaccination against small pox to curb and finally arguments, and by quoting examples of great women in sciimce
eradicate this disease from the population. (This has and other spheres; and persuade yourself and others that, given
already been successfully done in India). equal opportunity, women are on par with men.
(b) Television for eradication of illiteracy 'and for mass Ails. The development of human mind depends
communication of news and ideas. mainly on the nutrition content of prenatal and postnatal
1.16 PHYSICS-XI

diet and the environment around it. The gender factor has Bohr, Heisenberg, Chandrasekhar and Feynman. You are urged
no role in the development of human mind. When given to make special efforts to get access to the general books and
equal opportunities, mental development of females will writings by these and other great masters of physics. Their
be as rapid as that of males. Madam Marie Curie won the writings are truly inspiring.
Noble prize twice in science which is a rare gesture. Mrs. Ans. There is no doubt that the great laws of physics
Indira Gandhi and Mrs. Margret Thatcher have flourished are at once simple and beautiful. The Einstein's mass-
excellently well in politics. energy equivalence relation : E = mc2 has tremendous
1.14. "It is more important to have beauty in the equations impact on various physical phenomena and on human
of physics than to have them agree with experiments ". The great lives yet it is so simple. Bohr's quantum condition :
British physicist P. A. M. Dirac held this view. Criticize this L = nh / 21t and Planck's quantum condition : E = hv are

R
statement. Look out for some equations and results in this book quite simple, yet quite important laws of physics.
which strike you as beautiful. Heisenberg's uncertainty principle : ax· Sp ~ h / 21t is a
Ans. Dirac was his genious best when he pronounced simple law but changed the thinking of physicists
this statement. A physical equation must be simple and drastically. These are just few examples of the beauty of
hence beautiful. It will automatically agree with the physics laws which inspired the physicists and opened up
new vistas in physics.

SI
experimental r~sults. For example, consider Einstein
mass-energy relationship: E = mc2 1.16. Textbooks on science may give you a wrong
This simple equation not only governs energy impression that studying science is dry and all too serious and
generation in the sun and the other stars but it is also that scientists are absent-minded introverts who never laugh or
grin. This image of science and scientists is patently false.
mainly responsible for energy generation on the earth
Scientists, like any other group of humans, have their share of
from the processes of nuclear fission and nuclear fusion.
humorists, and many have led their lives with a great sense of
However, it is not always true. Although some equa- [un and adventure, even as they seriously pursued their
tions of Quantum Mechanics and Theory of Relativity are scientific work. Two great physicists of this genre are Gamow
highly cumbersome and difficult to understand, yet they and Feynman. You will enjoy reading their books.
IT
agree with the experiments. Ans. True, the scientists like any other group of
1.15. Though the statement quoted above may be disputed, humans have their share of humorists, and many have led
most physicists do have a feeling that the great laws of physics their lives with a great sense of fun and adventure inspite
are at once simple and beautiful. Some of the notable physicists, of pursuing their scientific work seriously. Two such great
besides Dirac, who have articulated this feeling, are: Einstein, physicists were Gamow and Feynman.
H
Text Based Exercises
1 Mark Each
O

1. What is science? 14. Some exciting and educative physical phenomena


2. What is the origin of the word science? and experience lead to the development of a
3. What is meant by the sanskrit word Shastra and theory. Give two such examples.
Arbic word Ilm ? 15. Name two Indian born physicists who have been
4. What is the basic aim of science? awarded Noble Price in physics. (Chandigarh02]
5. What is the difference between physical and 16. Name the scientist who replaced circular orbits by
M

biological sciences ? elliptical orbits in the heliocentric theory of the sun.


6. What is scientific attitude? 17. Name the scientist who discovered X -rays.
7. What is a theory ? 18. Who discovered electrons?
8. What is physics? 19. Name the scientist who first proposed the wave
9. What is the origin of the word physics? theory of light.
10. What are the two basic quests in physics? 20. Name the famous scientist who discovered the law
n. Name the domain of physics that deals with of gravitation.
phenomena intermediate between macroscopic. 21. Name the physicist who first proposed the nuclear
and microscopic domains. model of the atom.
12. Physics has a very limited scope and is only the 22. Who gave the quantum model of hydorgen atom?
pastime of a few blessed ones. Is it true? 23. Name the scientists responsible for the develop-
13. Stateone law that holds good in all natural processes. ment of Quantum Mechanics.
PHYSICAL WORLD 1.17

24. Name the physicist who first proposed the concept 48. Name the four fundamental forces in nature .
• of antiparticle. 49. Name the forces having the longest and shortest
25. Which physicist first confirmed the existence of ranges of operation. .
positrons experimentally? 50. Arrange the weak-nuclear force, electromagnetic
26. Name the physicist who first measured the charge force and gravitational force in the decreasing
on an electron experimentally. order of the strengths.
27. Name the physicist associated with wave- particle 51. Give the ratio of the strengths of the four funda-
duality. mental forces in nature.
28. Who first discovered neutrons? 52. What is the nature of the intermolecular Van der
29. Name the scientist who received Nobel Prize twice Walls' forces ?

R
in physics. 53. How many times is the strong nuclear forcestronger
30. Name the physicist who first gave the exchange than the electromagnetic force ?
theory of nuclear forces. 54. Is strong nuclear force a true fundamental force?
31. Name the scientist who was first awarded two 55. Which force mainly governs the structure of atoms
Nobel Prizes. and molecules?

SI
32. What were the important contributions of Madame 56. Which fundamental force governs the large scale
Marie Curie ? motion in the universe?
33. Who first discovered radioactivity ? 57. What is meant by charge independent character of
34. Who first gave theory of expanding universe ? strong nuclear force ?
35. Mention some important contributions of Albert 58. Which class of elementary particles experiences the
Einstein to physics. weak nuclear forceand not the strong nuclear force?
36. Name the Indian physicist, who was first awarded 59. Among which type of elementary particles does
the Nobel Prize. the electromagnetic force act ?
37. What was the impo~t discovery of C.V. Raman? 60. Name the class of elementary particldes on which
IT
38. What was the major contribution of Indian strong nuclear force acts.
physicist S.N. Bose? 61. Which particle initiates the electromagnetic force
39. With which field work was the famous Indian between two electrons? ,
physicist H.J. Bhabha associated? 62. What are the exchange particles for the operation of
40. Name the Indian physicist associated with the (I) strong nuclear force and (il) weak nuclearlorce?
triple helical structure of proteins. 63. How much is the range of gravitational force ?
41. Mention the major areas of modem scientificresearch What are the messenger particles for this force?
H
in chemical and biologicalsciencesthat are bound to 64. Name the physicists who first unified the electric
affectthe human society of the twenty-first century. and magnetic phenomena.
42. On which scientificprincipledoes an aeroplanework? 65. Name the unified domain of weak nuclear force
43. On which scientific principle calculators and and electromagnetic force.
computers are based ? 66. Name the physicists who first predicted the
O

44. Which technology of physics has triggered the existence of electro-weak force.
computer revolution in the last three decades of 67. What are the conserved quantities in nature? Give
the twentieth century ? two examples of such quantities.
45. What does the word LASER stand for? On what 68. State the law of conservation of momentum. From
basic principle does it work? which symmetry principle is this law obtained?
46. Name the phenomenon used in the production of 69. Which symmetry laws lead to (i) law of conser-
M

ultra high magnetic fields. vation of energy and (ii) law of conservation of
47. Name the philosopher who said about science "we angular momentum ?
know very little yet it is astonishing that we know 70. Some conservation laws are true for one funda-
so much and still astonishing that so little knowledge mental force but not for other. Give two such
(of science)can give us so much power". examples.

Answers

1. .Science is a systematised knowledge about the 2. The word science originated from the Latin verb
various natural phenomena gained by the man scientia which means to know.
through his careful experimentation, keen obser- 3. Bothof these words stand for organised knowledge.
vation and accurate reasoning.
1.18 PHYSICS-XI

4. The basic aim of science is to search for truth. It 33. A.H. Becquerrel in 1896.
tends to analyse the natural phenomena occurring 34. Edwin Hubble of U.S.A.
around us'. 35 .. (I) Mass-energy equivalence (il) Photoectric effect
5. Physical sciences deal with the properties and and (iii) Theory of relativity.
behaviour of non-living matter while biological 36. C.V. Raman.
sciences deal with living things. 37. Inelastic scattery of light by molecules.
6. Scientific attitude requires a flexible open-minded 38. Quantum statistics (Bose-Einstein statistics).
approach towards solving problems in which
39. Cascade process in cosmic radiation.
other important points of view are not neglected.
40. G.N. Ramchandran.
7. Theory is the name given to a minium number of
41. Genetic engineering, biotechnology, new chemical

R
laws in terms of which the behaviour of a physical
materials, etc.
system can be explained.
42. Bernoulli's principle in fluid dynamics.
8. Physics is the study of the basic laws of nature and
43. Digital logic of electronic circuits.
their manifestations in various natural phenomena.
44. Technology of silicon chip.
9. The word physics originates from a Greek word

SI
meaning nature. 45. The word LASERstands for light amplification by
stimulated emission of radiation. It works on the
10. (i) Unification and (ii) Reductionism.
principle of population inversion.
11. Meso-scopic physics which deals with a few tens
46. Superconductivity.
or hundreds of atoms.
47. Bertrand Russel.'
12. No, physics has a very wide scope. We find
application of physics in every walk of life. 48. (i) Gravitational force (il) Electromagnetic force

13. One such law is the Newton's law of gravitation. It (iil) Strong nuclear force and
states that every body in the universe attracts (iv) Weak nuclear fOIDi.
every other body with a force which is directly 49. Gravitational force has the longest range and
IT
proportional to the product of their masses and nuclear force has the shortest r~ge.
inversely proportional to the square of the distance 50. Electromagnetic force> Weak nuclear force>
between them. Gravitational force.
14. (i) The fall of an apple from a tree and motion of 51. Fe: Fw : FE: Fl'J = 1: 1<f5: 1036: 1038.
the moon led Newton to his famous law of 52. Intermolecular Vander WaIls' forces are electro-
gravitation. magnetic in nature.
H
(ii) The spherical shape of the liquid drops led to 53. 100 times.
the concept of surface tension.
54. No, strong nuclear force is now thought to·be a
15. (I) C.V.Raman & (il) Subramanium Chandrasekhar. force derived from quark-quark force.
16. Johann Kepler. 55. Electromagnetic force.
17. W.K. Roentgen. 56. Gravitational force.
O

18. J.J. Thomson. 57. Nuclear forces between a proton and proton, .a
19. Christiaan Huygens. neutron and a neutron, and a proton and a neutron"
20. Isaac Newton. are nearly equally strong. This indicates that the
21. Earnest Rutherford. strong nuclear force does not depend on the
22. Neils Bohr in 1913. charge of the nucleons.
M

23. Heisenberg and Schriilinger. 58. Leptons.


24. Paul Dirac in 1930. 59. Electromagnetic force acts on all electrically
25. Carl Anderson in 1932. charged particles.
26. R.A. Millikan. 60. Hadrons.
27. de Broglie. 61. Photon (y).
28. James Chadwick in 1932. 62. (I) Mesons
29. John Bardeen. (iI) Vector bosons.
30. Hideki Yukava of Japan. 63. The range of gravitational force is infinite.
31. Madame Marie Skladowak Curie, for physics in Gravitons are thought to be the exchange particles
1903 and for chemistry in 1911. for this force.
32. (i) Discoveries of radium and polonium. 64 .. Oersted and Faraday.
(ii) Studies on natural radioactivity. 65. Electro-weak force.
PHYSICAL WORLD 1.19

66. Sheldon Glashow, Abdus Salam, Steven Weinberg 69. (i) Homogeneity of time leads to the law of
in 1979. conservation of energy.
67. The physical quantities that remain unchanged in (ii) Isotropy of space leads to the law of conser-
a process are called conserved quantities. For example, vation of angular momentum.
energy, linear momentum, etc. 70. (i) Parity is conserved by the strong and electro-
6S. Law of conservation of linear momentum states magnetic forces but not by the weak force. .
that if no external force acts on a system then, its (ii) Strangeness is conserved by the strong force
total linear momentum remains constant. This law but not by the weak force.
follows from the homogeneity of space.

R
2 or 3 Marks Each

1. What is a scientific method? Mention the various 21. What is a weak nuclear force? Give its important
steps involved in a scientific method. properties. Give an example of a weak nuclear force.
2. Mathematics is an important tool in the develop- 22. Name the four fundamental forces in nature. Out

SI
ment of physics. Comment on this statement, of the four which one is (a) strongest, and (b) weak-
3. Mention four important discoveries of physics est ? [Delhi 03]
which have revolutionised modem chemistry. 23. Write the names of the four fundamental forces in
4. Name some advancements in physics which have nature. Give any two main characteristics of each
played an important role in the development of type. [Delhi 03C]
medical sciences. . 24. How do we classify elementary particles as baryons,
5. Name two advancements made in technology on mesons and leptons? Amongst which of these parti-
the basis of physics. [Himachal 06C] cles is the strong nuclear force dominant and
6. Theory and experiments go hand in hand in physics
IT
amongst which is the weak nuclear force dominant?
and help each other's progress. Give two suitable 25. State the law of conservation of energy. Give a
examples in support of this view.
suitable example for it.
7. The scope of physics is truly vast. Comment.
26. State the law of conservation of linear momentum.
S. Physics is a science of excitement. How? Give an example for it.
9. Give reasons for the tremendous progress in 27. State the law of conservation of angular momen-
physics in the last few centuries.
H
tum. Give an example for it.
10. Discuss the relation of physics with other sciences.
2S. Explain the law of conservation of charge with the
[Himachal 02, 041
help of a suitable example.
11. Discuss the relation of physics with mathematics
29. How are the conservation laws related with the ,
and chemistry. [Himachal 03]
symmetries of nature?
12. What is physics ? Discuss the relation of physics
O

30. Match the scientist's name against discovery :


with technology. [Himachal 03, 04, 05]
13. Discuss the scope and excitement of physics. A. Scientist B. Discovery
[Himachal 04] 1. Faraday 1. Law of gravitation
14. Explain the electromagnetic nature of the contact 2. Rutherford 2. Quantum model of
forces between two bodies. hydrogen atom
M

15. Briefly discuss the nature of the force of friction. 3. Chadwick 3. Unification of light and
electromagnetism
16. Explain how does the elastic force in a spring arise.
4. Bohr 4. Theory of Relativity
17. Briefly explain the origin of Van der Walls' force
between two neutral molecules. 5. Newton 5. Inelastic scattering of light
by molecules
IS. Taking a suitable example from daily life, show
6. Maxwell 6. Unification of weak and
that electromagnetic force is enormously stronger strong electromagnetic
than the gravitational force. interaction
19. Although gravitational force is the weakest of the 7. Salam 7. Law of electromagnetic
four fundamental forces, yet it governs the large induction
scale motion in the universe. Explain how. 8. Einstein 8. Expansion of the Universe
,20. What is a strong nuclear force? Give its important 9. Raman 9. Neutron
properties. Give an example of nuclear force. 10. Hubble 10. Nuclear model of the atom
1.20 PHYSICS-XI

31. Match the scientist in column A against the A. Technology B. Scientific


country of origin in column B : principlets)
3. Radio and T.V. 3. Superconductivity
A. Scientist B. Country of origin 4. Computers 4. Role of DNA in
1. Michelson 1. Denmark heredity
2. Newton 2. U.S.A. 5. Lasers 5. Thermodynamics
3. Landau 3. Italy 6. Production of 6. Faraday's law of
ultra high induction
4. Bhabha 4. France
magnetic fields
5. Bohr 5. India
7. Rocket propulsion 7. Conversion of
6. Archimedes 6. Germany gravitational
potential energy into

R
7. Galileo 7. U.S.S.R.
electrical energy
B. Curie B. Britain
B. Genetic B. Motion of charged
9. Heisenberg 9. Japan
engineering particles in
10. Yukawa 10. Greece electromagnetic fields
32. Match the technology in column A to its related 9. Electric generator 9. Fission of uranium

SI
scientific principle(s) in column B : by slow neutrons
10. Hydroelectric 10. . Amplification by
A. Technology B. Scientific power population inversion
principlets) II. Aeroplane 11. Digital logic of
1. Steam engine 1. Propagation of electronic circuits
electromagnetic 12. Particle 12. Bernoulli's principle
waves accelerators in fluid dynamics
2. Nuclear reactor ~. Newton's laws of
motion
IT
Answers
1. Refer answer to Q. 4 on page 1.1. 13. Refer answer to Q. 11 & Q. 12 on page 1.3.
2. Refer answer to Q. 14 on page 1.4. 14. Refer answer to Q. 20(/) on page 1.8.
15. Refer answer to Q. 20(ii) on page 1.8.
3. (I) X-ray diffraction techniques (ii) Study of radio-
16. Refer answer to Q. 20(iii)on page 1.8.
H
activity (iii)Discovery of isotopes by mass spectro-
graphs (iv) Nuclear magnetic resonance. 17. Refer answer to Q. 20(iv) on page 1.8.
4. (I) Electron microscope in studying structure of a 18. Refer answer to Q. 21 on page 1.8.
cell. 19. Refer answer to Q. 22 on page 1.8.
(ii) X-ray and neutron diffraction techniques in 20. Refer answer to Q. 23 on page 1.9.
O

studying the structure of nucleic acids. 21. Refer answer to Q. 24 on page 1.9.
(iii) Radioisotopes in radiation therapy for curing 22. Refer answer to Q. 17 on page 1.7.
skin cancer. 23. Refer answer to Q. 18 and Q. 19 on page 1~7and
(iv) Ultrasonics in diagonosing the human body. Q. 23 and Q. 24 on page 1.9.
5. Refer answer to Q. 16 on page 1.6. 24. Refer to 'For your knowledge' box on page 1.9.
6. (i) The alpha scattering experiments of Ruther- 25. Refer answer to Q. 29 on page 1.11.
M

ford gave the nuclear model of the atom. 26. Refer answer to Q. 30 on page 1.11.
(i/) Dirac's theoretical work introduced the concept 27. Refer answer to Q. 31 on page 1.12.
of antiparticle, which was later confirmed
28. Refer answer to Q. 32 on page 1.12.
experimentally by Anderson's discovery of
29. Refer answer to Q. 33 on page 1.12.
positron.
30. (1-7); (2 -10); (3 - 9); (4 - 2); (5 -1) ; (6 - 3);
7. Refer answer to Q. 11 on page 1.3.
(7 - 6); (8 - 4); (9 - 5); (10 - 8~
8. Refer answer to Q. 12 on page 1.3.
31. (1- 2) ; (2 - 8) ; (3 - 7) ; (4 - 5) ; (5 - 1) ; (6 - 10) ;
9. Refer answer to Q. 13 on page 1.4.
(7 - 3); (8 - 4); (9 - 6); (10 - 9~
10. Refer answer to Q. 14 on page 1.4.
32. (1-5); (2-9); (3-1); (4-11); (5-10); (6-3);
11. Refer answer to Q. 14 on page 1.4. (7 - 2);(8 - 4);(9 - 6);(10 -7);(11-12);(12 - 8).
12. Refer answer to Q. 16 on page 1.6.
Competition
Section

Physical World

R
GLIMPSES

SI
Science. Science is the systematised and organised 8. Physics is a basic science. Because of its important
knowledge about the various natural phenomena role played in the development of life science,
which is obtained by careful experimentation, medicine, technology and industry.. physics is
keen observation, and accurate reasoning. It has considered as the most basic of all sciences.
two main branches, (a) physical sciences, 9. Physics and mathematics. Mathematics has
(b) biological sciences. proved to be the most important tool in the
(I) Physical sciences. These sciences deal with the development of physics.
IT
properties and behaviour of non-living things. 10. Mechanics. It deals with the slow motion or
(I) Biological sciences. These sciences deal with equilibrium of material bodies.
the behaviour of living things .. U. Optics. It deals with the nature and propagation of
2. Physic&-,The word physics comes from a Greek light.
word which means nature. Physics is the study of 12. Thermodynamics. It deals with the change in
the natural laws and their manifestation in the internal energy, temperature, entropy etc., of the
natural phenomena. It deals with the concepts of
H
macroscopic systems through external work and
.. , space. time, moficn.rnatter, energy, radiation, etc. heat.
3. : Chemistry, Chemistry is the study of every 13. Electrodynamics. It .deals with electric and'
substance, its structure, its composition and magnetic phenomena associated with charged and
changes in which it takes part.
magnetic bodies.
4. Scie"tific attitu.de. Scientific attitude requires a
O

14. Quantum mechanics. It deals with the mechanical


...." flexlble, open-minded approach towards solving behaviour of atoms, molecules and nuclei.
, "problems in which other important points of view
15. Relativity. It deals with the particles, having
are not neglected.
speeds comparable to the speed of light. It is the
Scl~tific method. The step by' step approach
theory of invariance in nature.
used by a scientist in studying natural phenomena
16. Physics, technology and society, Sometimes,
M

. and e.s~ablis~ir,tg 4~s which govern these


phenomena,' is called Scientificmethod. It involves physics generates new technology. At other times
the steps like (I) experimentation and observation, technology gives rise to new physics. Both have a
(ilJ formation of hypothesis, (iii) verification of direct impact on society.
hypothesis and (iv) theoretical predictions. 17 •. Scope of physics. The scope of physics is truly
6:",,,~~fic, tl:teo~. 'Theory is the name given to a vast. It covers a wide range of magnitudes of
,:'t :::":: se't'of"a iimited number of laws in terms of which physical quantities such as mass, length, time,
, '. the beha~our' of a physical system can be energy, etc. .
explained. 18. Basic quests in physics. The two basic quests in
Physics is an exact science. It is because of high modern physics' are : (I) unification and
precision and accuracy obtainable In the measure- (ii) reductionism. These have resulted in great
ment of phy~cal quantities: !'" " . advances in physics.
",", I

1.21
COMPETITION

PHYSICS-XI

Fundamental forces in nature. There are four nuclear process such as the f3-decay of a nucleus.
fundamental forces which govern both macro-- It is caused by exchange of vector bosons.
scopic and microscopic phenomena. These are: (i) 24. Isolated system. Any system on \Yhichno external
gravitational force, (ii) electromagnetic force, (Hi) force acts is called isolated system. Universe is the
strong nuclear force and (iv) weak nuclear force. most ideal isolated system possible.
The relative strengths of these forces are 25. Law of conse~ation of energy. It states that the
Fc: Fw : FE: Es = 1: 1025 : 10 36: 1038. total energy of an isolated system remains constant.
It follows from the nature's symmetry called
20. Gravitational force. It is the force of mutual
homogeneity of time,
attraction between two bodies by virtue of their
masses. It obeys Newton's law of gravitation. It is 26. Law of conservation of linear momentum. It

R
caused by the exchange particles called gravitons. states that if no external force acts on a system,
21. Electromagnetic force. It is the force due to then its total linear momentum remains constant.
interaction between two moving charges. It is It follows from nature's symmetry called homo-
caused by exchange of photons (y) between two geneity of space.
charged particles. 27. Law of conservation of angular tnomentum.lt states

SI
22. Strong nuclear forfe. It is the stronge~t attractive that if no external torque acts on a system, then its
interaction which binds together the protons and angular momentum remains constant. It follows
neutrons in a nucleus. Electrons do not experience from nature's symmetry called isotropy of space.
this force.
28. Law of conservation of charge. It states that the
23. Weak nuclear force. It is the force that appea~ total charge of an isolated system remains constant.
only between elementary particles involved in a

Multiple Choice Questions


IT
1. A supposition that is put forward as a probable The scientist who made. this statement is
solution to a natural phenomenon is called a (a) Dirac {b) Maxwell
(a) hypothesis (b) theory (c) Faraday (d) Raman.
H
(c) law (d) relief. 6. "The most incomprehensible thing about the
2. A thought experiment in physics is one world is that it is comprehensible." This statement was
(a) which is theoretically possible but experi- made by the scientist
mentally not feasible. (a) Aristotle (b) Newton
(b) which is neither theoretically nor experi-
O

(c) Galileo (d) Einstein.


mentally feasible.
7. The country which awards the prestigeous Nobel
(c) which is performed by non-physicist. prize is
(d) none of the above. (a) U.S.A. (b) U.K.
3. "Nature is pleased with simplicity and effects not (c) Sweden (d) France.
M

the pump ofsuperfluous causes". It has been said by


8. An Indian scientist who won Nobel prize for
the great scientist
physics is
(a) Einstein (b) Dirac
(a) Sir J.e. Bose (b) H.J. Bhaba
(c) Russel (d) Newton.
(c) M.N. Saha (d) Sir c:v. Raman.
4. The book titled 'Principia' was written by [AFMC 01)
(a) Maxwell (b) Einstein 9. The man who has won Nobel Prize twice in
(c) Newton (d) Galileo.' [AFMC 01] physics is
5. "It is more important to have beauty in the (a) Einstein (b) Bardeen
equations of physics than to have them agree with (c) Heisenberg (d) Faraday.
experiments" . [AFMC 01]
SECTION

PHYSICAL WORLD 1.23

10. Madam Marie Curie won Nobel Prize twice (b) Unification of weak and electromagnetic forces
which were in the field of -, (c) Superconductivity
(a) Physics and chemistry (d) Laser technology.
(b) Chemistry only 20. Neutrons were discovered by
(c) Physics only (a) Fermi (b) Yukawa
(d) Biology only. (c) Dirac (d) Chadwick.
11. Prof. Albert Einstein got noble prize in physics 21. Louis de-Broglie is credited for his work on
for his work on (a) Theory of relativity

R
(a) special theory of relativity (b) Electromagnetic theory
(b) general theory of relativity (c) Matter waves
(c) photoelectric effect (d) Law of distribution of velocities.
(d) theory of specific heats. [CBSE 91] 22. The country to which de Broglie belongs is
12. The value of universal gravitational constant Ie'

SI
(a) Germany (b) England
was first experimentally determined by
(c) France (d) America.
(a) Newton (b) Galileo
23. Lightningwas discovered by [AFMC 96]
(c) Kelvin (d) Cavendish.
(a) Faraday (b) Franklin
13. Charge on an electron was first determined by :
(c) ,Ohm (d) Edison.
(a) Millikan (b) Bohr 24. The discoverer of loudness and intensity of
(c) Thomson (d) Rutherford. sound is
IT
14.J. D. Van der Walls discovered equation of gases (a) Newton . (b) Bell
and liquids. He was a (c) Leplace (d) Edison.
(a) English scientist (b) French scientist 25. The man who is known as the Father of
(c) Dutch scientist (d) German scientist. Experimental Physics is
15. X-rays were discovered by (a) Newton (b) Albert Einstein
.(a) Coolidge (b) Roentgen (c) Galileo . -(d) Rutherford .
H
(c) Maxwell (d) Fermi. 26. The total number of discoveries made by
16. The working of an aeroplane is based on Michael Faraday is
(a) Newton's third law of motion (a) 16 (b) 41
(b) Bernoulli's principle (c) 5 (d) 16401.
O

(c) Newton's law of gravitation ·27. The person who has been awarded the title of
(d) Law of conservation of momentum. the Father of Physics of 20th century is
(a) Madame Curie . (b) Sir C.V. Raman
17. Chandershekher, an America based Indian
scientist was awarded Nobel Prize in physics on the (c) Neils Bohar (d) Albert Einstein.
M

subject concerning 28. It has been postulated that there miiY be some
(a) Geophysics (b) Astronomy particles moving with speed greater than the speed of
(c) Superconductivity light. Such particles are known as
(d) Laser Technology. (a) Electrons (b) Neutrons
18. Radioactivity was discovered by (c) Nucleons (d) Tachyons. [AFMC 01]
(a) Chadwick (b) Rutherford 29. The book "Pisces of Physics" has been written
(c) Becquerel (d) Roentgen. by
19. Abdus Salam, an America based Pakistani (a) Einstein (b) Newton
physicist won Nobel Prize in the field of (c) Archimedes
(a) Inelastic scattering of light by molecules (d) Galileo. [AFMC 01]
COMPETITION

1.24 PHYSICS-XI

30. Which of the following is wrongly matched ? 32. The idea of calculus was given by
(a) Barometer-Pressure (a) Newton
(b) Lactometer-Milk (b) Einstein
(c) Coulomb's law-charges (c) Marconi
(d) Humidity-Calorimeter [AFMC 01) (d) Planck. [AFMC 03)
31. C.V. Raman got Nobel Prize for his experiment 33. Which of the following principles is being used
on in Sonar Technology?
(a) dispersion of light (a) Reflection of ultrasonic waves

R
(b) reflection of light (b) Newton's laws of motion
(c) deflection of light (c) Reflection of electromagnetic waves
(d) scattering of light. [AFMC01) (d) Laws of thermodynamics
[DPMr 2011]

SI
Answers
1. (a) 2. (b) 3. (d) 4. (c) 5. (a) 6. (d)
7. (c) 8. (d) 9. (b) 10. (a) 11. (c) 12. (d)
13. (a) 14. (c) 15. (b) 16. (b) 17. (b) 18. (c)
19. (b) 20. (d) 21. (c) 22. (c) 23. (b) 24. (b)
p. (d)
IT
25. (c) 26. (d) 28. (d) 29. (a) 30. (d)
31. (d) 32. (a) 33. (a)
H
O
M
R
SI
ITUNITS AND MEASUREMENTS

2.1 NEED FOR THE MEASUREMENT "When you can measure what you are speaking about
1. Explain the need for measurement in physics. and express it in numbers, you know something about
it, but when you cannot measure it and express it in
Need for the measurement. Physics is an exact numbers, your knowledge is of a meagre and unsatisfac-
H
science which gives an accurate knowledge about the tory kind, but you have scarcely, in your thoughts,
nature and the natural phenomena. It expresses advanced to the stage of science whatever the matter
various natural phenomena in terms of the relation- may be."
ships among the quantities involved. The exactness or
accuracy of these relationships depends upon the 2.2 PHYSICAL QUANTITIES :
O

measurements we make. The accuracy of the measure- FUNDAMENTAL AND DERIVED


ments, in turn, depends on the accuracy of the
2. What are physical quantities? Distinguish between
measuring instruments and techniques. The recent
fundamental and derived quantities.
progress in science and technology has been possible
only due to the development of high precision instru- Physical quantities. All those quantities which can be
measured directly or indirectly and in terms of which the
M

ments. One can measure mass as small as that of an


laws of physics can be expressed are calledphysical quantities.
electron (-10- 30 kg) and as large as that of the universe
For example, length, mass, temperature, speed, force,
(_1055 kg).
electric current, etc. The physical quantities are the
Prof. William S. Franklin emphasised the impor- building blocks of physics in terms of which the basic
tance of measurement in the following words: laws of physics can be expressed in mathematical
forms.
"The most important thing for a young man to acquire
from his first course in physics is an appreciation for Physical quantities are of two types - fundamental
precise details." and derived.
(i) Fundamental quantities. The physical quantities
Stressing upon the importance of measurement, which can be treated as independent of other physical
Lord Kelvin, a great English physicist of 19th century, quantities and are not usually defined in terms of other
once remarked that physical quantities are called fundamental quantities. To
1:!...a
2.2 PHYSICS-XI

give a consistent and unambiguous description to all Desirable characteristics of a physical unit :
physical quantities, we need a minimum of seven 1. It should be well-defined.
fundamental or base quantities. These are mass,
2. It should be of convenient size, i.e., neither too
length, time, electric current, temperature, luminous
small nor too large in comparison with the
intensity and amount of substance.
measurable physical quantity.
(ii) Derived quantities. The physical quantities whose 3. It should not change with time.
defining operations are based on other physical quantities are
4. It should be easily reproducible.
called derived quantities. All physical quantities other
than the seven base quantities are derived quantities. 5. It should be imperishable or indestructible.
For example, velocity, acceleration, force, momentum, 6. It should not be affected by the change in

R
etc. physical conditions such as pressure, tem-
perature, etc.
2.3 •. THE MEASURING PROCESS 7. It should be internationally acceptable. __
3. What is meant by the term measurement of a 8. It should be easily accessible.
physical quantity? How is the result of measurement of

SI
a physical quantity expressed ? 2.5 'If' FUNDAMENTAL AND DERIVED UNITS
Measurement. The measurement of a physical 5. Although the number of physical quantities which
quantity is the process of comparing this quantity with we measure is very large, yet we do not need a very large
a standard amount of the physical quantity of the same number of units for this measurement. Why ?
kind, called its unit. This is possible because the various physical
To express the measurement of a physical quantity, quantities are related to each other and so their units
we need to know two things : can be expressed in terms of just seven basic or
(i) The unit in which the quantity is measured. fundamental units.
IT
(ii) The numerical value or the magnitude of the 6. What are fundamental and derived units ? Give
quantity i.e., the number of times that unit is some examples.
contained in the given physical quantity. Fundamental units. The physical units which can
.. Measure of a physical quantity neither be derived from one another, nor they can befurther
=Numerical value of the physical quantity resolved into more simpler units are called fundamental
H
x size of the unit units. The units of fundamental quantities such as
mass, length, etc. are fundamental units.
or Q=nu
Let length of a room = 5 m = 500 em. Derived units. All the other physical units which can
be expressed in terms oj the fundamental units are called
Clearly, the smaller the size of the .uiit, the larger is
derived units. Let us consider the unit of speed.
O

the numerical value associated with the physical


quantity. Distance travelled
Spee d =
Time taken
Thus the numerical value (n) is inversely proportional to
the size (u) of the unit. Unit of distance
1 .. Unit of speed
n a: - or nu = constant. Unit of time
M

u
metre -1
If n1 and 11z are numerical values for a physical =---=ms
second .
quantity Q corresponding to the units u1 and "L., then
Thus the unit of speed (ms - 1) is a derived unit as it
Q= n1 u1 = n2 "L. ••• (2.1) has been expressed in the fundamental units of length
and time.
2.4 •. PHYSICAL UNIT AND ITS DESIRABLE
CHARACTERISTICS 2.6 •. SYSTEMS OF UNITS
4. What is a physical unit ? Write the essential 7. What is a system 0/ units ? Mention the various
requirements that a physical unit/standard must meet. types of systems of units.
Physical unit. The standard amount of a physical System of units. A complete set of units which is used
quantity chosen to measure the physical quantity of the same to measure all kinds offundamental and derived quantities is
kind is called a physical unit. called a system of units.
UNITS AND MEASUREMENTS 2.3

Some of the commonly used systems of units are as The seven basic SI units are defined as follows :
follows:
(i) Metre (m), It is the SI unit of length. One metre is
(i) cgs system. It was set up in France. It is based on defined as the length of the path travelled by light in
centimetre, gram and second as the fundamental units of vacuum during a time interval of 11299, 792, 458 of .
length, mass and time respectively. a second.
(ii) fps system. It is a British system based on foot, (ii) Kilogram (kg). It is the SI unit of mass. One
pound and second as the fundamental units of length, kilogram is the mass of prototype cylinder of
mass and time respectively. platinum-iridium alloy (whose height is equal to its
(iii) mks system. It is also a French system based on diameter) preserved at the International Bureau

R
metre, kilogram and second as the fundamental units of of Weights and Measures, at Sevres, near Paris.
length, mass and time respectively. (iii) Second (s). It is the SI unit of time. One second is
(iv) 51: The international system of units. SI is the the duration of 9, 192, 631, 770 periods of the
abbreviation for "Systeme Internationale d' Unites", radiation corresponding to the transition between
which is French equivalent for international system of two hyperfine levels of the ground state of the cesium

SI
units. It is a modernised and extended form of the -133 atom.
metric systems like cgs and mks systems. This system
(iv) Ampere (A). It is the SI unit of electric current.
was adopted by eleventh General Conference of
One ampere is that constant current which, if
Weights and Measures in 1960. It covers all branches of
science and technology. It is based on the following
maintained in two straight parallel conductors of
seven basic units and two supplementary units. infinite length, of negligible cross-section, and placed
1 metre apart in vacuum, would produce between
Lable 2.1 Basic 51 quantities and units these conductors a force equal to 2 x 10-7 newton
per metre of length.
IT
S. Basic physical Symbol (v) Kelvin (K). It is the SI unit of temperature. One
Basic unit
No. auantity kelvin is thefraction 11273.16 of the thermodynamic
l. Length metre m temperature of the triple point of water. The triple
2. Mass kilogram kg point of water is the temperature at which ice,
3. Time second s water and water vapour co-exist.
4. Temperature kelvin K (vi) Candela (cd). It is the SI unit of luminous
H
5. Electric current ampere A intensity. One candela is the luminous intensity, in
6. Luminous intensity candela cd a given direction, of a source that emits mono-
7. Quantity of matter mole mol chromatic radiation of frequency 540 x 10 12 hertz
- and that has a radiant intensity of 1/683 watt per
Lable 2.2 Supplementary 51 units steradian in that direction.
O

(vii) Mole (mol). One mole is that amount of a substance


S. Supplementary Symbol which contains as many elementary entities as there
Basic unit
No. auantity are atoms in 0.012 kg of carbon-12 isotope. The
l. Plane angle radian rad entities may be atoms, molecules, ions etc.
2. Solid angle steradian sr The two supplementary SI units are defined as
M

follows:
• • (a) Radian (rad), It is defined as the plane angle
.\. The cgs, mks and SI are metric or decimal systems of sub tended at the centre of a circle by an arc equal
units. This is because the multiples and sub-multiples in length to the radius of the circle.
of their basic units are related to the practical units by
powers of 10. S (.m ra dilans ) =---
Arc =-I
Radius r
.\. The fps system is not a metric system. This system is
not in much use these days. (b) Steradian (sr). It is defined as the solid angle
subtended at the centre of a sphere by a surface of
2.7 DEFINITIONS OF BASIC AND the sphere equal in area to that of a square,
SUPPLEMENTARY SI UNITS having each side equal to the radius of the sphere.
8. Name and define all the basic and supplementary . dian) Surface area
n (in stera an = 2
units of SI. Radius
2.4 PHYSICS-XI

2.8 COHERENT SYSTEM OF UNITS


A HISTORICAL VIEW OF BASIC SI UNITS 9. What is a coherent system of units ? Give examples.
Coherent system. It is a system of units based on a
metre (m)
certain set offundamental units from which all derived units .
•••••In 1791, the Paris Academy of Sciences defined metre
can be obtained by simple multiplication or division without
as one ten millionth part of the distance measured
introducing any numerical factor. For example, mks
along the meridian from the north pole to the equator.
system is a coherent system of units in mechanics. All
•••••In 1889, the General Conference of Weights and
derived units in mechanics such as those of area,
Measures defined metre as the distance between two
lines marked on a platinum-iridium rod preserved 1t a volume, density, acceleration, force, etc.,: can be

R
constant temperature of 273.16 Kat 1 bar pressure in obtained by the multiplication or division of the
the International Bureau of Weights and Measures at fundamental units of mass, length and time. SI is a
Sevres, near Paris. coherent system of units for all branches of physics.

2.9 ADVANTAGES OF SI

SI
10. State the advantages of SI over other systems of
units.
Advantages of 51 over other systems of units:
(i) SI is a coherent system of units. All derived
units can be obtained by simple multiplication
or division of fundamental units without
introducing any numerical factor.
Fig. 2.1 Prototype metre. (ii) SI is a rational system of units. It uses only one
IT
unit for a given physical quantity. For example,
•••••In 1960, the standard metre was defined in terms of all forms of energy are measured in joule. On
wavelength of light. One metre is defined as the distance the other hand, in mks system, the mechanical
which contains 1650, 763.73 wavelengths of certain energy is measured in joule, heat energy in
orange-redradiation(of wavelength6057.8021 N emitted calorie and electrical energy in watt hour.
by krypton-86 source kept at the temperature of triple
point of nitrogen. (iii) SI is a metric system. The multiples and sub-
H
multiples of SI units can be expressed as powers
kilogram (kg) of 10.
•••••In 1791 in France, originally kilogram was defined as
(iv) SI is an absolute system of units. It does not use
the mass of one cubic decimetre (or 1 litre) of water at
gravitational units. The use of' g' is not required.
4°C. Water has maximum density a .s temperature.
•••••In 1889, the General Conference of Weights and (v) SI is an internationally accepted system of units .
O

Measures defined kilogram as the mass of platinum-


2.10 GUIDELINES FOR WRITING SI UNITS
iridium cylinder preserved at Sevres.
IN SYMBOLS
second (5)
11. State the rules that are followed in writing SI
•••••The Paris Academy of Sciences defined second as the units in symbolic form.
time taken by a simple pendulum of one metre length
M

Rules for writing 51 units in symbolic form :


to swing from one extreme position to the other.
•••••A solar day is defined as the time interval between the (i) Small letters are used for symbols of units .
noons of two successive days. A mean solar day is the (ii) Symbols are not followed by a full stop.
average of the solar days in one complete year. The (iii) The initial letter of a symbol is capital only
1 or __ 1_ part of a mean solar day is when the unit is named after a scientist.
24 x 60 x 60 86,400
(iv) The full name of a unit always begins with a small
called mean solar second.
letter even if it has been named after a scientist.
•••••In 1956, the second was defined in terms of the tropical
(v) Symbols do not take plural form.
year 1900, the duration of which was taken as 365.242
mean solar days. So second was defined as the·
2.11 .• ABBREVIATIONS IN POWERS OF TEN
fracti
raction 1 or 1 0f
365.242 x 86,400 31,556,925.9747 12. Make a list of various prefixes used for powers of
the tropical year 1900 and was called ephimeris second. 10. Give some examples.
UNITS AND MEASUREMENTS 2.5

Abbreviations in powers of ten. When the B. Practical units used for measuring large
magnitudes of the physical quantities are very large or distances:
very small, it is convenient to express them in the (i) Light year. It is the distance travelled by light in
multiples or submultiples of the SI units. The various vacuum in one year.
prefixed used for powers of 10 are listed in table 2.3.
1 light year = Speed of light in vacuum x 1 year
l,able 2.3 Prefixes for powers of ten =3x 108 ms-1 x 365:25x 24x 60x 60 s
•. 1 light year = 11y = 9.467x 1015 m
Multi- Sub-
Prefix Symbol Prefix Symbol Light year is used in astronomy to measure
pie multiple
distances of nearby stars. Alpha centauri, the nearest
101 deca da 10 -1 deci d
star outside the solar system is 4.3 light years away

R
102 hecto h 10 - 2 centi c from the. earth.
103 kilo k 10 - 3 milli m (ii) Astronomical unit. It is defined as the mean
106 mega M 10 - 6 micro distance of the earthfrom the sun. It is used in astronomy
fl
to measure distances of planets.
10 9
giga G 10 - 9 nano n
1 astronomical unit = 1 AU = 1.496 x 1011 m

SI
1012 tera 'T 10 -12 pico P (iii) Parsec (parallactic second). It is the largest
1015 peta P 10 -15 femto f practical unit of distance used in astronomy. It is defined
1018 exa E 10 -18 atto a as the distance at which an arc of length 1 astronomical unit
subtends an angle of 1 second of arc.
Examples: I I
As 8=- r=-
1 mega ohm = 1 Mn = 106 n 1 milliampere or r 8
ImA=1O-3A 1 parsec = 1 AU = 1.496 x 1011 m =3.08 x 1016 m
IT
1 kilometre = 1 krn = 103 m 1 microvolt or I" 1 1t
--x -rad
IflV=1O-6V 3600 180
1 decagram = 1 da g = 10 g 1 nanosecond or 1 parsec = 3.08x 1016 m = 3.261y
1 ns = 10 - 9 S

1 centimetre = 1 ern = 10 - 2 m 1 picofarad or sun


1 pF = 10 - 12F.
H
1AU
1"
2.12 SOME COMMON PRACTICAL UNITS ~
1 parsec Earth
13. Make a list of some commonly used practical
units. How these units are related to SI units? Fig. 2.2 Parsec
A. Practical units for measuring small distances :
O

(i) Fermi. It is the small practical unit of distance Relations between astronomical unit, light year
used for measuring nuclear sizes. It is also called and parsec
femtometre. 1 AU = 1.5 x 1011 m
1 fermi = 1 fm = 10 -15m 1 Iy = 9.46x 1015m
The radius of a proton is 1.2 fermi.
M

1 parsec = 3.08x 10 16m


(ii) Angstrom. It is used to express wavelength of
light. 15
~ = 9.46x 10 = 6.3 x 104
1 angstrom = 1 A = 10-10 m = 10 -8 cm 1 AU 1.5x 1011

(iii) Nanometre. It is also used for expressing wave- 1ly = 6.3 x 104 AU
length of light.
16
1 parsec 3.08x 10 = 3.26
1 nanometre = 1 nm = 10-9 m Also
11y 9.46x 1015
(iv) Micron. It is the unit of distance defined as
micrometre. 1 parsec = 3.26 ly

1 micron = 1 um '= 10-6 m Clearly, 1 parsec> 11y> 1 AU


2.6 PHYSICS-XI

C. Practical units for measuring areas: G. Practical units used for measuring pressure:
(i) Barn. It is used for very small areas, such as 1 bar= 1 atmospheric pressure
nuclear cross-sections. = 105 Nm - 2 = 105 pascal (Pa)
1 barn = 10-28 m 2 1 millibar = 102 Pa
(ii) Acre. It is used measuring large areas. 1 torr = 1mm of Hg column
1acre = 4047 m 2 1 atmospheric pressure
(iil) Hectare. It is also used for measuring large areas. = 1 bar = 760 mm of Hg column
1 hectare = 104 m 2
= 760 torr.
D. Practical units used for measuring large masses:

R
1 tonne or 1 metric ton = 1000 kg
1 quintal = 100 kg
CONVERSIONS USED
1 slug = 14.57 kg
lkg m-3 = 10-3 g em-3
1 pound = 1 lb = 0.4536 kg
1g em - 3 = 103 kg m-3

SI
1 Chandra Shekher limit = 1 CSL = 1.4 times the'
1N= 105 dyn
mass of the sun.
lamu = 1.66 x 10- 27 kg
CSL is the largest practical unit of mass.
1 AU = 1.496 x 1011 m
-; E. Practical unit used for measuring very small 11y = 9.46 x 10 15 m
masses:
1parsec = 3.08 x 1016 m
Atomic mass unit. It is defined as ~th of the mass lA = 10-10 m= 0.1 nm
12
of one 1~ C atom.
How many light years are there in one metre?
IT
EXAlWPLEl.
1 atomic mass unit = 1 amu = 1 u = 1.66x 10- 27 kg
[Himachal 06C]
The mass of a proton or a neutron is of the order of
one amu. Solution. 11y = 9.46 x 10 15 m

F. Practical units used for measuring time: or 9.46x 10 15


m = 11y
(i) Solar day. It is the time taken by the earth to 1m= 1 15 ly = 1.057x 10 -16 ly.
complete one rotation about its own axis w.r.t. the 9.46x 10
H
sun. EXAMP1.E 2. What is the number of electrons that would
(ii) Sedrial day. It is the time taken by the earth to weigh 1 kg ? Mass of an electron is 9.11 x 10 - 31 kg.
complete one rotation about its own axis w.r.t. a Solution. Mass of each electron = 9.11 x 10 - 31 kg
distant star.
Total mass = 1 kg
O

(iii) Solar year. It is the time taken by the earth to


complete one revolution around the sun in its orbit. .. Number of electrons
Total mass 1kg
1 solar year = 365.25 average solar days
Mass of each electron 9.11 x 10 -31 kg
= 366.25 sed rial days
= 1.1x 1030•
(iv) Tropical year. The year in which there is total
M

solar eclipse is called tropical year. EXAMPLE .1. The radius of gold nucleus is 41.3 fermi.
Express its volume in ~.
(v) Leap year. The year which is divisible by 4 and
in which the month of February has 29 days is called a Solution. Here r = 413 fermi = 41.3x 10 - 15 m
leap year. Volume, V = i nr3 =ix 22 x (41.3x 10 -15 m/
(vi) Lunar month. It is the time taken by the moon 3 3 7 -
to complete one revolution around the earth in its = 2.95x 10- 40 m 3.
orbit. ExA,lfPLE4. Convert an accelerationof2 km h" 2 into cm s" 2.

1 lunar month = 27.3 days . km 2 x 10 em5


-2
Solution. a =2 -2 = 2 = 0.0154 em s .
(vii) Shake. It is the smallest practical unit of time.
1 shake = 10- 8S h (36~0 s)
UNITS AND MEASUREMENTS 2.7

EXAMPLES. The Young's modulus of steel is 1.9 x 10 11 Nm-2. 2. 1 parsec = 3.08 x 1016 m
Express it in dyn em" 2. 1
.. 1m = ~16parsec
Solution. Y = 1.9x 10 11 N2 3.08 x io-
m = 3.25 x 10 -17 parsec.
5
=1.9 x 10 11 x 10 dyn 3. As 1 parsec = 3.26 light year
(100 cm)2
.. 1 light year = _1_ parsec = 0.3067 parsec.
=1.9 x 1012 dyn em -2. 3.26
4. Density = 1.293 kg m - 3 = 1.293 x 10- 3 g ern - 3
EX.'l..UPLE6. The density of a material is 0.8 g em 3. Express
it in Sl units. = 0.001293 g cm - 3 •

R
Solution. As 1 g cm -3 = 103 kg m - 3 . m 10 x 10- 3 krn
5. Acceleration = 10 ~ = ( 1 h)2
•. O.8gcm -3 = 0.8 x 103 kg m" 3 = 800 kg m -3.
3600
EXA}~fPLE 7. How many amu would make up 1 kg ?
= (3600)2 x 10- 2 krn h- = 1.29 x10s km h-2•
2
Solution. As 1 amu = 1.66 x 10 - 27 kg

SI
Total mass
. 1 u 6. No. of protons = -------
.. 1 kg = 27 amu = 6.024 x 10 amu. Mass of each proton
1.66 x 10-
10- 3 kg = 5.99 x10 23
EXIBIPI..E 8. Express the average distance of the earth from 1.67 x 10- 27 kg
the sun in (i) light year and (ii) parsec.
Solution. A verage distance of the earth from the sun,
2.13 ORDER OF MAGNITUDE
r = 1 AU = 1.496 x 10 11 m
14. Define order of magnitude of a physical quantity.
(i) As 1 light year =9.46 x 10 15 m How is it determined? Illustrate it with same examples.
IT
r= 1.496 x 10 11 I =1.58xlO-51. . Order of magnitude. The order of magnitude of a
.. 9.46 x 10 15 Y Y physical quantity is that power of 10 which is ciosestto its
magnitude. It gives an idea about how big or how small
(ii) As 1 parsec =3.08 x 10 1~ m a given physical quantity is.
1.496 x 10 11 _ To determine the order of magnitude of a number
.. r= 16 parsec = 4.86 x 10 6 parsec.
3.08 x 10 N, we first express it as
H
N =nx lOX.
:-: PROBLEMS FOR PRACTICE If 0.5 < n :s; 5, then x will be the order of magnitude
1. Calculate the number of astronomical units in one of N.
metre. (Ans. 6.67 x 10 -12 AU) Few examples are illustrated in table 2.4.
O

2. How many parsec are there in one metre ?


(Ans. 3.25 x 10-17 parsec)
l:.,able 2.4 Order of magnitude

3. How many parsec are there in one light year? Measure Expressed in Order of
number N neartest power of 10 magnitude
(Ans. 0.3067 parsec)
8 0.8 x 101 1
4. The density of air is 1.293 kg m - 3. Express it in cgs
M

units. (Ans. 0.001293 g ern - 3) 49 4.9 x 101 1


5. Convert an acceleration of 10 ms - 2 in krn h - 2. 52 0.52 x 102 2
(Ans. 1.29 x 105 krnh - 2) 555 0.555 x 103 3
6. The mass of a proton is 1.67 x 10- 27kg. How many 999 0.999 x 103 3
protons would make 1 g ? (Ans. 5.99 x 1023) 1001 1001 x 103 3
753000 0.753 x 106 6
:-: HINTS
0.135 135 x 10-1 -1
1. As 1AU=1.5x1011 m
0.05 5 x 10- 2 -2
1 m= 1 11 AU = 6.67 x10 -12 AU. 0.99 0.99 x ~Oo 0
1.5 x 10
2.8 PHYSICS-XI

X HINTS

1. (i) 8=0.8x1d (ii) 49=4.9x1d


FORMULAE USED (iii) 52 = 0.52 x 102 (iv) 999 = 0.999 x 103
To determine the order of magnitude of a number (v) 1001 = 1.001 x 10 3 (vi) 753000 = 0.753 x 106
N, we express it as (vii) 0.05 = 5 x 10- 2 (viii) 0.99 = 0.99 x 100
N = n x io'. 2. Astronomical unit is the mean distance of the earth
If 0.5 < n ~ 5, then x will be the order of magnitude from the sun. 1 AU = 1.496 x 1011 m
ofN. :. Order of magnitude = 11.
CONVERSIONS USED 3. 1day = 24 x 60 x 60 = 86400s = 0.864 x 105 s
1 AU = 1.496 x 1011 m,l parsec = 3.08 x 1016 m

R
CONSTANT USED
Speed of light in vacuum, C= 3 x 108 ms - 1 2.14
--------
.. Order of magnitude = 5.

MEASUREMENT OF LENGTH BY
DIRECT METHODS
EXAMPLE 9. Write the order of magnitude of the following 15. Define Length. Name the device used for measuring

SI
measurements: directly the lengths (i) from 10- 3 m to 102 m (ii) to an
(i) 25,710,000 m (ii) 0.00000521 kg.
accuracy ofl0- 4 m and (ii) to an accuracy ofl0- 5 m.
Solution. We first express each number in terms of
nearest power of 10. Length. Length may be defined as the distance of
separation beiuieen two points in space.
(i) 25,710,000 m =2.571 x 107 m
(i) Metre scale.
As 0.5 < 2.571 < 5
(ii) Vernier callipers.
:. Order of magnitude = 7.
(iii) Screw gauge or spherometer.
IT
(ii) 0.00000521 kg = 0.521 x 10- 5 kg
As 0.5 < 0.521 < 5 2.15 INDIRECT METHODS FOR MEASURING
.. Order of magnitude = - 5. LARGE DISTANCES
EXA."'IPLE 10. Express 1light year in terms of metres. What 16. Describe an indirect method to measure the
is its order of magnitude ? height of an accessible tree or a tower.
Triangulation method for the height of an
H
Solution. One light year is the distance travelled by
light in one year. accessible object. Let AB = h be the height of the tree or
Speed of light, C =3 x 108 ms-1 the tower to be measured. Let C be the point of
t = 1 year = 365.25 days = 365.25 x 24 x 60 x 60 s observation at distance x from B. Place a sextant at C
and measure the angle of elevation,
:. 1 light year = ct .
O

A
= 3x 108 x 365.25x 24x 60x 60 m LAOB=9.
= 9.467x
Order of magnitude
1015 m = 0.9467 x
of light year
1016 m.
= 16.
From right MBC, we have

tan 9 = AB =~
CB x cL:J:
M

X PROBLEMS FOR PRACTICE x


or height, h =x tan 9
1. Write the order of magnitude of the following:
Knowing the distance x, the Fig. 2.3 Height of
(i) 8 (ii) 49 (iii) 52 (iv) 999 height h can be determined. a tree.
(v) 1001 (vi) 753000 0.05
(vii) (viii) 0.99.
17. Describe a method to measure the height of an
(Ans. 1, 1, 2, 3, 3, 6, - 2, 0) inaccessible object like a mountain.
2. What is one astronomical unit ? Express it in Triangulation method for the height of an
metres. Write its order of magnitude. inaccessible object. Let AB = h be the height of the
(Ans. Order of magnitude = 11) mountain to be measured. By using a sextant, we first
3. What is the order of magnitude of seconds in a measure the angle of elevation of its peak from a point
day? (Ans. 5) C on the ground. Let it be 91 .
UNITS AND MEASUREMENTS 2.9

Move the sextant to another position 0 such that CD = d.


Again measure the angle of elevation, L. ADB = 82 .

D B
I--- d --01+--- x ~

R
Fig. 2.4 Height of a mountain.
Earth
CB x
In rt. MBC, cot 81 = AB =h
Fig. 2.6 Distance of a planet.
DB d+ x
In rt. ~ABD, cot 82 = AB =-h- 8=~=k

SI
Now
Radius S
. d+x x d
cot 8 - cot 8 = -- - - =- s=k.
2 1 h h It
8
d
or It=----- where 8 = L. APB = <11 + ~, is the parallactic angle.
cot 82 - cot 81
19. Describe the parallax method for the determi-
Knowing d, the height h can be determined. nation of the distance of a nearby star from the earth.
18. What is meant by parallax and parallactic angle? Distance of a nearby star by parallax method.
How can we find the distance of the moon (or any
IT
As shown in Fig. 2.7, suppose N is the nearby star
planet) by parallax method ?
whose distance d from the earth is to be found. F is a far
Parallax. Parallax is the apparent shift in the position of off star whose direction and position is fixed for all
an object with respect to another when we shift our eye positions of the earth in its orbital motion. When the
sidewise. The closer object always appears to move in earth is at position A, the parallax angle between
the direction opposite to that of our eye. distant star F and nearby star N is determined. Let it be
To understand parallax, hold a pen 0 at distance 5 81' After six months, the earth is at diametrically
H
from the eyes. Look at the pen first by the left eye L opposite position B. The parallax angle, L. NBF = 82 is
(closing the right eye) and then by the right eye R measured. Fe eF
I I
(closing the left eye). The position of the pen appears to Total parallax angle I N I
I I
change with respect to the subtended by N on the
o I I

background. This is called I I


O

earth's orbital diameter

D
I I

parallax. The distance between ABis


I
I
I
I
the two points of observation I
I
I
I
is called basis. In this case, the 8 = 81 + 8 2
: 91 92 :
distance (LR = b) between the I I
L b R As 8=~ I I
two eyes is the basis. Radius I I

L. LOR = 8 is called paraI/ax A B


M

8= AB
angle or parallactic angle. Fig. 2.5 Parallax
d
Distance of the moon or any planet. To measure Fig. 2.7 Distance of a
d= AB.
the distance 5 of the moon or a far away planet P, we or 8 nearby star.
observe it simultaneously from two different positions
The parallax method is useful for measuring
(observatories) A and B on the earth, separated by a
distances of the stars which are less than 100 light years
large distance AB = b. We select a distant star S· whose
away from the' earth.
position and direction can be taken approximately
same from A and R 20. Briefly describe a method for the determination
Now L.PAS' =<11 and L.PBS' =~ are measured of distance of a far away star.
from the two observatories at the same time. As b« S, Intensity method. This is a spectroscopic method .
so we can take AB as an arc of length b. based on inverse square law of intensity. According
2.10 PHYSICS-XI

to this law, the intensity of illumination at a point is Then from the right angle triangle shown in the figure,
inversely proportional to the square of the distance from the we find that
source of light. Here we assume that the intrinsic R
brightness of all the stars is same. We compare the ~=SinE
Res
intensity II of the faint image of a far away star taken
on a photographic plate with the intensity 12 of the Hence the distance of the planet from the sun is
bright image of a nearby star. Let r1 and r2 be the Rps = sin E. Res = sin E • AU
respective distances of these two stars.
where Res is the average distance of the earth from the
From inverse square law of intensity, sun and is called astronomical unit (AU).
!J. = r2: or r
1 = r2 [12] 1/2
23. How can we determine the distance of a superior

R
planet from the earth ?
~ ~ ~
Distance of a superior planet. By knowing the
knowing the distance r2 of the nearby star, the distance
r1 of the far away star can be determined. This method distance of any planet from the sun, we can determine
is useful for measuring distances of stars which are the distance of any superior planet. For this purpose
more than 100 light years away from the earth. we use Kepler's third law of planetary motion. This

SI
law states the square of the period (T) of revolution of a
21. What are inferior and superior planets? Give the
planet round the sun is proportional to cube of the
names of these planets.
semi-major axia (a) of the orbit i.e.,
Inferior planets. The planets which are closer to the
sun than the earth are called inferior planets. Mercury and T2 a: a3
Venus are the inferior planets.
If Tl and T2 are periods of revolution of two planets,
Superior The planets which are farther from
planets. and a1 and a2 are their respective semi-major axes, then
the sun than the earth are called superior planets. Jupiter,
3 r2 (T. 2/3
IT
Saturn, Uranus, Neptune and Pluto are the superior
a23 = r22 or a2 = a1 r2
planets.
a1 1 J
1
22. Describe a method for determining the distance
Knowing the values of a1, Tl and T2, we can find out a2.
of an inferior planet.
24. Describe a method to measure the diameter of
Copernicus method. Copernicus assumed circular
the moon or any planet.
orbits for the planets. The angle formed at the earth
H
between the earth-planet direction and earth-sun Diameter of the moon.
direction is called the planet's elongation. As shown in Let AB = Dbe the diameter of
Fig. 2.8, let the moon (or planet) which is
Rps = distance of the planet from the sun to be measured by an
observer 0 on the earth. A
Rpe = distance of jhe planet from the earth
O

telescope, is focussed on the


Res = distance of the earth from the sun
moon and the angle AOB
E = planet's elongation subtended by it on the point
o of the earth is found.
Ase=~
M

Radius
• AB D
-=-
OB 5 Fig. 2.9 Diameter of
or D=S8 the moon.

Linear diameter = Distance x angular diameter


Earth Knowing 5 and 8, D can be determined.

Fig. 2.8 Copernicus method. 25. Describe a method to measure distance of a hill.
When the elongation' attains its maximum value Reflection or echo method. To find the distance of
and the planet appears farthest from the sun, the angle a hill, a gun is fired towards the hill. The time interval
subtended by the sun and the earth at the planet is 90°. between the instant of firing the gun and the instant of
UNITS AND MEASUREMENTS 2.11

hearing the echo of the gun shot is noted. During this


time interval, the sound first travels towards the hill
and then back from the hill to the place of firing. If v is
FORMULAE USED
the speed of sound, 5 is the distance of the hill and t is
the total time taken, then 1. Reflection or echo method
cxt vxt
25 =vx t or 5=vxt 5=-- or
2 2
2
2. Triangulation method
26. Describe a reflection method for measuring the (i) Height of an accessible object
distance of the moon.
h=xtan6
Laser method. The word laser stands for light where x is the distance of observation point

R
amplification by stimulated emission of radiation. Laser is a from the foot of the object.
source of very intense, highly monochromatic (of (ii) Height of an inaccessible object
one wavelength) and highly directional beam of d
h=-----
light. A laser beam is sent towards the moon and its cot 62 - cot 61
reflected pulse is received. If t is the time elapsed

SI
where d is the distance between the two
between the instants the laser beam is sent and
observation points.
received back, then the' distance of the moon from
3. Parallax method. The distance of an astronomical object
the earth is given by
5= Basis !!.
5 = cx t Parallactic angle 6
2
4. Size of an astronomical object,
-where c = 3 x 108 ms - 1, is the speed of light.
Linear diameter = Distance x angular diameter
21'. Briefly describe a reflection method for estimating or D= 5 x 6
the distance of a nearby planet from the earth.
IT
5. Copernicus method
Radar method. The word RADAR stands for radio
(i) The distance of a planet from the sun,
detection and ranging. A radar can be used to measure
rps = sin e. res = sin e. AU
accurately the distance of a nearby planet (such as
Venus). Here radiowaves are sent from a transmitter (ii) The distance of a planet from the earth,
which after reflection from the planet are detected by rpe = cas E. res = cas e. AU
the receiver. By measuring the time interval (t) 6. From Kepler's law of periods
H
between the instants the radiowaves are sent and 3
a2 122 [ 12] 2/3
received, the distance of the planet can be determined ~ 3 = 1i 2 or a2 = 1i .~
as
cx t UNITS USED
5=-
2 Distances 5, x, d, h, a, b and D are in metre, angle 6
O

where c is the speed of the radiowave. This method can in radian, velocities v and c in ms-1
also be used to determine the distance of an aeroplane. CONVERSIONS USED
28. How is a sonar used in finding the depth of the 1" = 4.85 x 10- 6 rad
sea-bed?
11. Calculate the angle of (a) 10 ( degree) (b) l'
M

Sonar method. The word SONAR stand" for sound EXA.MPI.E


navigation and ranging. On a sonar, ultrasonic waves (minute of arc or arcmin) and (c) 1" (second of arc or arc
(sound waves having frequencies greater than 20,000 Hz) second) in radians. Use 3600 = 2n rad, 10 = 60' and l' = 60".
are transmitted through the ocean. They are reflected [NCERT]
by the submerged rock or submarines and received by Solution. (a) As 360 0
= 2n rad
the receiver. By measuring the time delay t of the
:. 10
=~ rad = 1.745 x 10-2 rad.
receipt of the echo, the distance 5 of the submerged 360 .
rock or submarine can be determined as (b) 1 = 60' = 1.745 x 10- 2 rad
0

5= v x t I' = 2.908x 10- 4"" 2.91 x 10-4 rad


2 (c) l' = 60" = 2.908 x 10- 4 rad.
where v is the speed of the ultrasonic waves in water. :. 1" = 4.847x 10- 4 rad ""4.85x 10-6 rad.
2.12 PHYSICS-XI

EXrl1HPLB 12. The shadow of a tower standing on a level Solution. Here parallactic angle,
plane is found to be 50 m longer when sun's altitude is 30° 8 = 1°54' =114' =(144 x 60)"
than when it is 60°. Find the height of the tower.
= 114 x 60 x 4.85 x 10- 6 rad =3.32x 10- 2 rad
7
Basis, b = AB = 1.276 x 10 m
The distance of the moon from the earth,
7
" , S =!!.-= 1.276x 10 = 3.84x 108 m.
8 3.32x 10- 2

EXr1MPLB 15. The angular diameter of the sun is 1920". If


the distance of the sun from the earth is 1.5 x 1011 m, what

R
is the linear diameter of the sun ? . [NCERT]
Solution. Distance of the sun from the earth,
S= 1.5 x 10 11 m
Angular diameter of the sun,

-
••..•
- --- 50 m ----->101

SI
8 = 1920" = 1920 x 4.85 x 10- 6 rad
Fig. 2.10 Linear diameter of the sun,
D = S x 8 = l.5x 1011 x 1920x 4.85x 10- 6
9
= 1.4x 10 m.
d 50
h=----- EXAMPl1,; 16. Assuming that the orbit of the planet
cot 82 - cot 81 cot 30° - cot 60° Mercury around the sun to be a circle, Copernicus deter-
h= 50 = 50.J3 = 25.J3 mined the orbital radius to be 0.38 AU. From this, determine
or
.J3-1/.J3 3-1 the angle of the maximum elongation for Mercury and its
IT
distance from the earth where the elongation is maximum.
= 25x 1.732 = 43.3 m.
Solution. Here rps = 0.38 AU,
EXflMPLE 13. A man wishes to estimate the distance of a
nearby towerfrom him. He stands at a point A in front of the ~s = 1 AU [By Definition]
tower C and spots a very distant f Distant The angle of maximum elongation E for a planet is
object 0 in line with AC. He then : object a given by
H
walks perpendicular to AC upto B,
a distance of100 m, and looks at 0 sin E = rps = 0.38 = 0.38
c
and C again. Since 0 is very ~s 1
distant, the direction BO is E = sin - 1 (0.38) = 22.3°
practically the same as AO ; but Distance of mercury from the earth is
O

he finds the line of sight of C


shifted from the original line of rpe = cos E. AU = cos (22.3°) x 1.496x 1011 m
sight by an angle 8 = 40° (8 is A 1+--100 m ----+I B = 0.9252x 1.496 x 1011 m
known as 'parallax'). Estimate the = 1.384 x 1011 m = 1.384x 108 km.
distance of the tower C from his Fig. 11
EXA1l11'LB17' In the case of Venus, the angle of maximum
original position A. [NCERT]
M

elongation is found to be approximately 47°. Determine the


Solution. Here parallax angle, 8 = 40° distance between Venus and the Sun rvs and the distance
AB between Venus and the Earth.
Now tan8=-
AC Solution. Here angle of maximum elongation, E =47°
AC = AB = 100 m = 100 m = 119 m. Distance between Venus and the sun is
tan 8 tan 40° 0.8391
rvs= sin E. AU = sin 47° x 1 AU
EXAiUPLE The moon is observedfrom two diametrically
14. = 0.73x 1.496 x 10 11 m = 1.09 x 10 11m.
opposite points A and B on the earth. The angle 8 subtended
Distance between Venus and the earth is
at the moon by the two directions of observation is 1°54'.
Given the diameter of the earth to be 1.276 x 107 m, compute rve = cos E • AU = cos 47° x 1 AU
the distance of the moon from the earth. [NCERT] = 0.68x 1.496x 1011 m = 1.02 x 1011 m.
UNITS AND MEASUREMENTS 2.13

EXA.lI-fPLE 18. Suppose there existed a planet that went 8. Find the period of revolution of planet mars about
around the sun twice as fast as the earth. What would be its the sun compared with that of the earth about it.
orbital size as compared to that of the earth ? The mean distance of the Mars from the Sun is
Solution. Let the period of revolution of the earth =~ 1.52 AU. (Ans. 1.87 year)

As the planet goes round the sun twice as fast as the X HINTS
earth, so its period of revolution is
1 _ V x t _ 1.450 x 200 _ km
1.S-- - -145.
Tp = 2" Te 2 2
Orbital size of the earth, ae = 1 AU 2S 2 x 1595
3. t = - = = 2.2 s.
v 1450

R
Orbital size of the planet, ap =?
From Kepler's law of periods, ~ h= d 1km 13
=.~ km .
cot 82 - cot ~ cot 30° - cot 45°
T2 a3
--.L =L 5. Here parallactic angle,
T 2 a3
e e
8 = 57 ' = 57° = 57 x ~ rad

SI
T ]2/3 [ 2 ] 1/3 60 60 180
"» =[~ . x ae = ~{ x 1 AU Basis, b = Radius of earth = 6.4 x 106 m
Distance of the moon from the earth,
= (0.5) 2/3 AU = 0.63 AU.
b 6.4 x 106 x 60 x 180 8
S= - = = 3.86 x 10 m.
X PROBLEMS FOR PRACTICE 8 57 x IT
1. In a submarine fitted with a SONAR, the time 1° 1 IT
6. 8 = 1.0 min =- = - x- rad
interval between the generation of an ultrasonic 60 60 180
wave and the receipt of its echo is 200 s. What is the S= ~ = 2 x 6400 x 180 x 60
IT
distance of the enemy submarine ? The speed of 8 IT
sound in water is 1.450km s- 1. (Ans. 145 km) = 4.4 x 107 km = 4.4 x 1010m
2. A radar signal is beamed towards a planet and its
= 4.4 x UyOAU = 0.293 AU.
echo is received 7 minutes later. If the distance 1.5x Uy1
between the planet and the earth is 6.3 x UyOm,
calculate the speed of the signal. (Ans. 3 x 108 ms-1) 7. Here rps = 0.5 AU, res= 1AU
H
3. A rock under water is 1595m deep. Find the time in sin e = Tps= 0.5 = sin 30°, :. s = 300.
which an ultrasonic signal returns after reflection res 1
from the rock. Given speed of ultrasonic waves in Also, rpe = cos e. AU = cos 30° AU = 0.866 AU.
water = 1450ms-1. (Ans.2.2s)
8. Fo~the mars: am = 1.52 AU

fJ'':~~[~~l=':\:=1.87
4. The angle of elevation of the top of a hill is 30°from
O

a point on the ground. On walking 1 km towards


the hill, angle is found to be 45°. Calculate the
height of the hill. (Ans. 1.366 km)
FO:~: you.

5. The moon subtends an angle of 57 minutes at the


base-line equal to the radius of the earth. What is
the distance of the moon from the earth ? Radius of 2.16 INDIRECT METHODS FOR MEASURING
M

the earth = 6.4 x 106 m. (Ans. 3.86x 108 m) SMALL DISTANCES


6. The parallax of a heavenly body measured from 29. Explain how will you determine the radius of an
two points diametrically opposite on equator of earth atom by Avogadro's hypothesis.
is 1.0 minute. If the radius of the earth is 6400 km, Atomic radius by Avogadro's hypothesis. Atoms
find the distance of the heavenly body from the are spherical in shape. So when a large number of
centre of the earth in AU. atoms are packed together, some empty spaces are left
Given 1 AU"'; 1.5 x Uy1 m. (Ans. 0.293 AU) between them. According to Avogadro's hypothesis, the
7. Assuming that a planet goes round the sun in a actual volume occupied by the atoms in one gram of a substance
circular orbit of radius 0.5 AU, determine the angle is two-third of the volume of one gram of the substance.
of maximum elongation for the planet and its dis- Let M be the molecular mass of a substance. Then
tance from the earth when elongation is maximum. M grams of the substance will contain N (Avogadro
(Ans. 30°, 0.866 AU) number) of atoms.
2.14 PHYSICS-XI

Number of atoms in 1 gram = N S. Length


Size of object or distance (ni)
M No.
If r be the radius of each atom, then volume of 5. Wavelength of light 10-7
. N 4 3
6. Size of red blood corpuscle 10-5
atoms ill one gram = - . - 1tr .
M 3 10-4
7. Thickness of a paper
Let V be the actual volume occupied by molecules
8. Height of a person 10°
in 1 gram of the substance. Then by
. N 4 3 2
9. Height of Mount Everest 104
Avogadro's hypothesis, -. - 1tr = - V 10. Radius of the earth 107
M 3 3
II. Distance of the moon from the earth 108

R
If p is the density of the substance, then
12. Radius of the sun 109
p= Mass = ! or V = .! 13. Distance of the sun from the earth io "
Volume V p 1013
14. Distance of Pluto from the earth
3
t t 3
15. Size of Milkway 1021

SI
~. ~ 1tr = ~. or r=[21t:p 16. Distance to Andromeda galaxy 1022
17. Distance to the boundary of observable 1026
Thus, the radius of an atom of the substance can be universe
determined.
30. Describe a method for measuring the molecular Exam les based on
size of oleic acid.
Size of molecule of oleic acid. Oleic acid is a soapy FORMULAE USED
liquid with large molecular size. We dissolve 1 cm ' of 1. Molar volume = Volume of 1 mole of a gas at S.T.P.
IT
oleic acid in 20 em 3 of alcohol and then redissolve 1 cm3 = 22.4 L
of this solution in 20 em 3 of alcohol. Then the 4
concentration of oleic acid is 1/400 cm3 in 1 cm ' of 2. Volume of a sphere = - rrr3
3
alcohol. We then determine the approximate volume
3. Thiickn ess 0 f an '1 fil Volume of oil drop
m = ------..:..
of each drop(V em3). Now pour ndrops of the solution 01
Area of the film
on the surface of water taken in a broad vessel. We
stretch the film carefully. As the alcohol evaporates, a
H
UNITS USED
very thin film of oleic acid is left on water surface. We Volume V is in m 3 or litre, radius r in metre.
measure the area A of the film using a graph paper.
CONSTANT USED
Volume of n drops of the solution = nV em3
Avogadro's number,
Amount · aciid iill
0 f 0IeIC tthiIS so I'ution = -nV cm 3 N = 6.023 x 1023
O

400
Thickness of the oil film, CONVERSIONS USED
1A=1O-10m,lL=1O-3m3
Volume ofthe film nV
t= =--em
Area of the film 400 A
EXAMPLE 19. The radius of a muonic hydrogen atom is
Assuming that the film has one molecular thickness, 2.5 x 10-13 m. What is the total atomic volume in m3 of a
M

then t will be approximately the size or diameter of a mole of such hydrogen atoms ?
molecule of oleic acid. The value of t is found to be of Solution. Radius, r = 2.5 x 10-13 m
the order of 10- 9m.
Volume of one atom = i 1tr3
l:.,able 2.5 Orders 01 Magnitude 01 length 3
Number of atoms in 1 mole = 6.023 x 1023
S. Length Volume of 1 mole of H-atoms
Size of object or distance
No. (m)
I. Radius of a proton 10 -15 = Nx i1tr3
3
2. Size of atomic nucleus 10 -14
3. Size of hydrogen atom 10 -10
= 6.023 x 1023 x i x 3.14 x (2.5 x 10-13)3
3
4. Size of typical virus 10-8 =3.94x 10-14 m3.
UNITS AND MEASUREMENTS 2.15

EXAMPLE 20. A drop of olive oil of radius 0.25 mm spreads Solution. Areal magnification
into a circular film of radius 10 em on the water surface. 1.0 m x 1.5 m
Estimate the molecular size of olive oil. 24 x 10- 3 m x 36 x 10- 3 m
Solution. Thickness of oil film
'" 1736
_ Volume of oil drop
Linear magnification = .J1736 = 41.67.
Area of the film
4
- 1t x (0.025)3 cm ' 4 :-: PROBLEMS FOR PRACTICE
= 3 =-x (25)3 x 10-11 em 1. If the size of an atom (=- 1A) were enlarged to the tip
1t x (10)2 Cllf2 3

R
of a sharp pin (=- 10- 5 m), how large would the
= 2.08 x 10- 7 em height of Mount Everest be ? (Ans. 109 m)
2. If the size of an atom (=-lA) were enlarged to the
Assuming that the film has one molecular thick-
ness, then molecular size of olive oil size of the earth (=- 107 m), how large would its
nucleus be ? Take size of nucleus = 10- 15 m ,
= 2.08x 10-7 em.

SI
(Ans. 100 m)
3. If the universe were shrunk to the size of the earth,
:-: PROBLEM FOR PRACTICE
how large would the earth be on this scale ?
1. A drop of olive oil of radius 0.30 mm spreads into a
(Ans. 10-11 m i.e. size of the atom)
rectangular film of 30 cm x 15 cm on the water
surface. Estimate the molecutar size of olive oil. :-: HINTS

(Ans. 7.5 x 10- 7 cm) if 10- 5 m 5


1· Magnl ication = 10- 10 m = 10
IT
Apparent height of Mount Everest
= Actual height x magnification
FORMULAE' USED
· if Final size Size of image = 104 m x 105 = 109 m.
1. Lmear magm ication = ----
Initial size Size of object 7
2 · M agnlifitea ti on = Size of earth = 10 m
= 1017
2. Linear magnification = ~ Areal magnification Size of atom 10- 10 m
H
UNITS USED Apparent size of nucleus
Magnification has no units, = Actual size x magnification
= 10-15 x 1017 = 100 m ,
7
3 · M agmifica ti on = Size of earth 10 m
BXI1MPLE 21.If the size of a nucleus ('" 1O-15m) is scaled = --- = 10-18
Size of universe 1025 m
upto the tip of a sharp-pin ( '" 10- 5 m), what roughly is the
O

size of the atom ? [NCERT) Apparent size of earth = 107 m x 10-18 = 10-11 m ,

Solution. Magnification in size


Size of the tip of sharp pin = 10- 5 = 1010 2.17 MASS AND WEIGHT
Size of nucleus 10- 15 31. Define the terms mass and weight. Give their SI units.
M

Actual size of an atom = 1 A = 10- 10 m, which is Mass. The mass of a body is the quantity of matter
scaled up by a factor 1010. contained in it, It is a basic property of matter, It does
not depend on the temperature, pressure or location of
.'. Apparent size of atom = 10-10 x 1010 = 1m. the body in space, The SI unit of mass is kilogram (kg).
Thus a nucleus in an atom is as small in size as the tip Weight. The weight of a body is the force with which a
of a sharp pin placed at the centre of a sphere of radius body is pulled towards the centre of the earth, It is equal to
about one metre, the product of the mass (m) of the body and the
acceleration due to gravity (g) of the earth on body.
Example 22. A 35 mm wide slide with a 24 mm x 36 mm
picture is projected on a screen placed 12 cm from the slide, Thus W=mg
The image of the slide picture on the screen measures As the value of 'g' changes from place to place, so
1.0 m x 1.5 m: What is the linear magnification of the the weight of a body is different at different places.
projector-screen arrangement? The SI unit of weight is newton (N).
2.16 PHYSICS-XI

32. Distinguish between mass and weight.


Mass Weight
l. Mass is the measure of Weight is the measure
inertia. of gravity
2. It is a scalar quantity. It is a vector quantity.
3. It is a constant quantity.It varies from place to
place.
4. It cannot be zero for a Weight of a body is zero
body. at the centre of the earth.
5. It is an essential pro- It is not an essential

R
perty of material bodies. property.
6. It is not affected by the It is affected by the pre-
presence of other bodies. sence of other bodies. Fig. 2.12 Inertial balance.
7. Its units are gram, Its units are dyne, Thus T ex .Jm or T 2 ex m
kilogram, etc. newton, etc.

SI
Let "1 and 111z be the inertial masses of two objects
2.18 INERTIAL AND GRAVITATIONALMASSES and Tl and T2 be their corresponding periods of
vibration, then
33. Distinguish between inertial mass and gravitational
mass. or
Inertial mass. The mass of a body which determines its
inertia in translatory motion is called its inertial mass. It is If "1 is a standard mass, then unknown mass 111z can
defined by Newton's second law of motion and is be determined.
equal to the ratio of the external force applied on the
35. How is gravitational mass of a body measured by
IT
body to the acceleration produced in it. By ewton's
second law, a common (or physical) balance ?
F Gravitational mass by a physical balance. A
or m,=-
I a physical balance works on the principle of moments i.e.,
when an object is balanced under the action of several
Here mj is the inertial mass of the body which can
forces acting in the same plane, the sum of the
be measured by using an inertial balance.
clockwise moments is equal to the sum of anticlock-
H
Gravitational mass. The mass of a body which deter- wise moments. The essential parts of a physical
mines the gravitational pull acting upon it due to the earth is balance are shown in Fig. 2.13. .
called its gravitational mass. It is defined by Newton's law
of gravitation. According to this law, the force of gravi- Stirrup Beam Balancing
tation of the earth on ~ body of mass mg is given by screw
O

GMmg FR2
F=--- or m =--
R2 g GM
Here mg is the gravitational mass of the body which
can be measured by using a physical balance.
M

34. Briefly explain how can we measure the inertial


mass of a body.
Measurement of inertial mass. The inertial mass of
a body is measured by using a device called inertial
balance. As shown in Fig. 2.12, it consists of a long strip
of metal.
One end of the strip is clamped to a table. The other
end of the strip carries a pan in which body whose
inertial mass is to be measured is kept. When the strip <,
vibrates horizontally, its 'inertia comes into play and
Levelling
not the force of gravity of the earth. It is found that screw
period of vibration T is directly proportional to the square
root of the inertial mass m of the body. Fig.2.13 Physicalbalance.
UNITS AND MEASUREMENTS 2.17

The object to be weighed is placed in the left pan


and the standard weights are placed in the right pan.
The weights are adjusted till the beam becomes
FORMULAE USED
horizontal. In this condition, the gravitational force on
. Mass M
the object is equal to the gravitational force on the 1; Density = or p=-
standard weights. Hence the gravitational mass of the Volume V
4 3
object is equal to that of the standard weights 2. Volume of a sphere, V = - 11:r
(cancelling the effect of g). 3
UNITS USED
36. How can a spring balance be used to measure the
gravitational mass of a body? - Radius r is in metre, volume V in m 3 and

R
density p in kg m - 3.
Gravitational mass by a spring balance. In a
spring balance, the gravitational force on a body
stretches the spring and we measure the elongation of EXAMPLE 23. Consider a white dwarf and a neutron star
the spring. The elongation depends on gravitational each of one solar mass. The radius of the white dwarf is same
force which, in turn, is proportional to the gravitational as that of the earth C"- 6400 km) and the radius of the

SI
mass of the body. Thus the neutron star is 10 km. Determine the densities of the two
elongation of the spring gives types of the stars. Take mass of the sun =2.0 x 1030 kg.
a measure of the gravi- Solution. For white dwarf : M = 2 x 1030 kg,
tational mass. For this we r = 6400km =64x 105 m.
first calibrate the spring
Density of white dwarf
balance by using standard
masses and measuring the M 2 x 1030
elongations. The calibration =-4-= 4 =1.822 x 109 kg m-
3

- 1t? - 1t x (64 x 105)3


IT
of spring balance and the 3 3
mass measurement should be
done at same place because For neutron star: M =2 x 1030 kg, r = 10 km =104 m
the value of g varies from FiS 2.14 Spring Density of neutron star
place to place. balance. 30
2 x 10 = 4.77 x 1017 kgm -3.

l,able 2.6 Orders of magnitude of mass ! 1t x (104)3


H
3
S.No. Object Mass (kg) EXAMPLE 24. Assume that the mass of a nucleus is given by
l. 10 - 30 M = A mp' where A is the mass number and radius of a
Electron
nucleus r = ro A 1/3, where ro = 12 f. Estimate the density of
2. Proton 10 - 27 nuclear matter in kg m- 3. Given mp = 1.67 x 10- 27 kg.
O

Uranium atom 10 - 25
3.
-
Solution. Here mp = 1.67 x 10:- 27 kg,
4. Cell 10 -10
ro =1.2f= 1.2 x 10-15 m
5. Dus~ particl~ 10-9
-- -- Amp Amp 3mp
6. Rain drop 10-6 Nuclear density =-- - =-
! 1tr3 - ! 1t (r,
M

A1/3)3 41t r~
7. Mosquito 10-5 3 3 0
-~- -----
8. Grape 10-3 3x1.67xl0-27 k -3
---- 4x3.l4x(12xl0-15)3 gm
9. Man 102
-- -
10. Automobile 103 = 2.3 x 1017 kg m-3.
-----
11. Ship 105
:-: PROBLEM FOR PRACTICE
12. Moon 1023
----..=- -- 1. A neutron star has a density equal to that of nuclear
13. Earth 1025 matter (= 2.8 x 1017 kg m - 3). Assume the star to be
- --- -
14. Milkyway 1041
spherical, find the radius of the neutron star whose
mass is 4.0 x l(jO kg (twice the mass of the sun).
15. Observable universe 10 55
(Ans. 15 km)
2.18 PHYSICS-XI

2.19 '" MEASUREMENT OF TIME 6. Radioactive dating. This technique is used to


measure long time intervals by finding the ratio of the
3 7. What type of phenomena can be used as a time
number of radioactive atoms that have undergone decay
standard? Give some examples. Which one do you select
to the number of atoms left undecayed. Carbon dating
as the most suitable standard for measuring time?
is used ro estimate the age of fossils, whereas uranium.
Measurement of time. According to Einstein,
dating is used to estimate the age of the rocks.
"Time is simply what a clock reads. Any'phenomenon that
repeats itself after equal intervals of time can be used as a
time standard". Examples of such a phenomenon are
l:.,able 2.7 Orders 01magnitude 01 time intervals
1. Beating of human heart. s. Event
Time

R
2. Oscillations of a pendulum. No. (5)
1. Life of most unstable particle 10 -24
3. Rotation of the earth about its own-axis.
2. Time taken by light to cross a nuclear 10 -22
4. Revolution of the earth around the sun.
distance
5. Vibrations of a quartz crystal in quartz wristwatch.
3. Time period of electron in hydrogen. 10 -15 .
6. Period of vibration of cesium -133 atom.

SI
atom
Of the above examples, the period of vibration of 10 -15
4. Period of visible light waves
cesium - 133 atom serves as the most accurate standard r:
5. Life timerof an excited state of atom ... 10- 8
of time.
6. Period of radio wave 10- 6
38. What are the advantages of defining second in
terms of period of radiation from cesium - 133 atom ? 7. Period of sound wave 10-3
Advantages of defining second in terms of period 8. Wink of eye 10-1
of radiation from cesium - 133 atom: 9.
•Time between two successive heart beats 10°
IT
1. Cesium atomic clock has high accuracy of 1 part 10. Travel time for light from moon to earth 10°
in 1013. 11. Travel time for light from sun to earth 102
2. It can be easily reproduced in any good laboratory. 12. Time period of a satellite 104
3. It is not affected by physical conditions of
13. Period of rotation of the earth (one day) 105
pressure, temperature, etc.
14. Period of revolution of the earth (1 year) 107
4. It is imperishable.
H
15. Travel time for light from nearest star 108
39. Give a brief account of the various techniques
used for measuring small and large time intervals. 16. Average human life-span 109
Some techniques used for measuring time 17. Age of the universe 1017
intervals over different ranges:
O

1. Electrical oscillators. In India, a.c. is supplied at


a frequency of 50 Hz. A motor running synchronously ••.
\. In India, the National Physical Laboratory (New Delhi)
with this a.c. can be used to provide a time scale. has the responsibility of maintenance and improve-
2. Electronic oscillators. A junction transistor can ment of physical standards of length, mass, time, etc.
be used to produce oscillations of very high frequency ••.
\. The iodine stabilized helium neon laser has been used
which can be used to measure small time intervals. to realize the latest definition of standard metre.
M

3. Quartz clocks. A quartz crystal shows piezo- ...\. Italian physicist Galileo was the first to set up
electric effect. If fluctuating pressure is applied across a pendulum clocks.
pair of its parallel faces, an oscillatory emf is developed ••.
\. The cesium atomic clocks have an high accuracy of 1
across another pair of perpendicular faces and vice , part in 10 13. They lose OI:...gain not more than 3 J.1s in
versa. The oscillations produced can be used to- one year.
measure time intervals. ...\. Table 2.5 shows that the ratio of the largest and shortest
4. Atomic clocks. These clocks are based on . lengths of objects in the universe is about 10 41.
periodic vibrations taking place within the atoms. The ••.
\. Table 2.7 shows that the ratio of the largest and the
first cesium atomic clock was set up in 1964. shortest time intervals associated with the events in
5. Decay of elementary particles. The life of many the universe is also about 10 41.
elementary particles varies from 10 - 16 to 10 - 24 s. By ••.
\. Table 2.6 shows that the ratio of the largest and the
making use of their decay times, very small time smallest masses of the objects in the universe is about
intervals can be measured. (1041)2.
UNITS AND MEASUREMENTS 2.19

4. Age of the universe is about 1010 years whereas the


mankind has existed for 106 years. For how many
seconds would the man have existed if age' of
FORMULAE USED universe were 1 day? (Ans. 8.64 s)
. ;. Difference in time M
Fractional error m time =. .- = -
Time internal t X HINTS

UNITS USED 3. Here t = 70 years = 70 x 365 + 17


Times t and ilt are in second. [17 days added due to 17 leap years in 70 years]
or t = 25567 days = 25567 x 86400 s
EXAMPLE 25. The average life of an Indian is 56 years. Find

R
il t = 0.2s
the number of times the human heart beats in the life of an
Indian, if the heart beats once in 0.8 s. Fractional error in time
Solution. Average life of an Indian = 56 years ~ 0.2 = 0.904 x 10-10 =: 10-10.
t 25567 x 86400
= 56 x 365.25 x 24 x 60 x 60 s
So the accuracy shown by atomic clock is 10- 10 part

SI
Period of heart beat = 0.8 s
in 1 s or 1 s in 1010 S.
Total number of heart beats in 56 years
4. Magnification in time
= 56x 365.25x 24x 60x 60 =2.2x 109 times. 6
0.8 Age of mankind = 10 = 10- 4
Age of universe 1010
EXAMPLE 26. The mean life of an elementary particle pion
is 2 x 10 7 nanosecond. The ageof the universe is about 4 x 109 Apparent age of mankind
years. Identify a physically meaning time interval that is approxi- = 10- 4 x 1day = 10- 4 x 86400 s = 8.64 s.
mately half way between these two on a logarithmic scale.
IT
Solution. Mean life of pion, 2.20 DIMENSIONS OF A PHYSICAL QUANTITY
tl = 2 x 10- 7 ns -' 2 x 10- 7 x 10- 9 s 40. What do you mean by seven dimensions of the
= 2 x 10- 16 s "'-10- 16 S world?
Mean life of universe, Seven dimensions of the world. All the derived
9 9 7 physical quantities can be expressed in terms of some
t2 = 4 x 10 years = 4 x 10 x 3 x 10 S
c.ombination of the seven fundamental or base
H
= 1.2 x 1017 s "'-1017 s
quantities. We call these fundamental quantities as the
Let t be the time interval that is half way between tl
seven dimensions of the world, which are denoted with
and t2 on the logarithmic scale. Then
square brackets [ ].
1
log t = 2 [log tl + log t2]
Dimension of length = [L]
O

= !:.[log 10-16 + log 1017] Dimension of mass = [M]


2 Dimension of time = [T]
1
= - [-16 + 17] =0.5 "'-I Dimension of electric current = [A]
2
Dimension of thermodynamic temperature = [K]
t = 10 s
Dimension of luminous intensity = [cd]
M

which is the time taken by an athlete to run a 100 m track.


Dimension of amount of substance = [mol]
X PROBLEMS FOR PRACTICE
41. What do you mean by dimensions of a physical
1. Find the number of seconds in 1 year. Express them quantity? Explain with the help of an example.
in order of magnitude. (Ans. 3.158 x 107 s, 107)
Dimensions of a physical quantity. The dimensions
2. Find the number of times the human heart beats in of a physical quantity are the powers (or exponents) to which
the life of 60 years of a man, assuming that the heart
the fundamental quantities must be raised to represent that
beats once in 0.8,s. (Ans. 2.3668 x 109 s)
quantity completely,
3. Two atomic clocks.allowed to run for average life of
For example,
an Indian (say, 70 years) differ by 0.2 s only.
Calculate the accuracy of standard atomic clock in . Mass Mass
Density = ---
measuring a time interval of Is. (Ans. 1 sin HYo s) Volume Lengthx breadth x height
2.20 ?HYSICS-XI

Dimensions of density Examples: The dimensional formula of the volume


[M] = [ML-3]= [M1L-3To] is [MOL3p]and that of momentum is [MLr 1].
[L] [L] [L] Dimensional The equation obtained by
equation.
Hence the dimensions of density are T in mass, ,- 3' equating a physical quantity with its dimensional formula is
in length and '0' in time. called the di tensional equation of the given physical quantity.
2.21 DIMENSIONAL FORMULAE AND Examples: The dimensional equation of force is
DIMENSIONAL EQUATIONS [Force]= [MLr2]
42.c:What is meant by dimensional formula and The dimensional equation for pressure is
dimensional equation ? Give examples.
[Pressure] = [ML-1r2]

R
Dimensional formula. The expression which shows
how and which of the fundamental quantities represent the Dimensional formuale and 51 units of some
dimensions of a physical quantity is called the dimensional physical quantities are given in table 2.8.
formula of the given physical quantity.

SI
l,able 2.8 Dimension~1 Formulae and SI units oJ some physical quantities

"S. Relation with other


Physica~ Quantity Dimf:isional formula SI unit
No. quantities
.~ .......
. ~:. . . ,l'
£ _ •. -'"'.r.-" ,
C;
1. L2 = [MOL2TO] m2
Area Length x breadth Lx L =
.
2. Volume Length x breadth x height Lx L xL = lMOL3TO] m3
Mass
IT
3. Density --- ~ = [ML-3To] kgm-3
Volume L

4. Speed or velocity Distance .!:: = [MOLT-I] ms-I


Time T
Change in velocity ms-2
5. Acceleration Lrl = LT-2 = [MoLr2]
Time T
H
6. Momentum Mass x velocity M xLT-I = [MLrl] kg ms-1

7. Force' Mass x acceleration M xLr2 = [MLr2] N

8. Work Force x distance MLr2 xL = [ML2r2] J


9. Energy Amount of work [ML2r2] J
O

"
Work
-- ML2r2
___ = [ML2r3]
10. Power Time W
T

11. Pressure Force


-- ML1r2
Area
--2 - = [ML-Ir2] Pa or Nm-2
L
MLr2
M

12. Moment of force or torque Force x .L, distance xL = [ML2r2] Nm


Force x (distancej? MLr2L2
13. Gravitational constant 'G' = [M-1L3r2] Nm2kg-2
Mass x mass MxM
~ -
14. Impulse of a force Force » time MLr2 xT = [MLrl] ~ Ns
Force 2
15. Stress -- MLr = [ML-Ir2] Nm-2
Area L2
.-
16. Strain Change in dimension [MOLOTO](dimensionless) -
Original dimension
Stress ML-1r2
17. Coefficient of elasticity --
Strain
= [ML-1r2] Nm-2
1
UNITS AND MEASUREMENTS 2.21

S. Relation with other


Physical Quantity Dimensional formula SI unit
No. quantities
18. Surface tension Force 2 Nm-1
--- ML r = MT-2 = [MLOr2]
Length L
19. Surface energy Work ML2r2 Jm-2
-- --2- = MT-2 = [MLOr2]
Area L --

20. Coefficient of viscosity Force x distance MLT-2 xL Nm-2 s


= [ML-1r1]
Ar~ x velocity L2 xLT-1 or Pa s
-
21. Angle Arc

R
--- ~ = 1= [MOLoTo] (dimensionless) rad
Radius L.
-

22. Angular velocity Angle ..!. = rl = [MoLorl] rad s-l


Time T
Angular acceleration Angular velocity rad s" 2
23. rl = r2 = [MoLor2]
Time

SI
T
24. Moment of inertia . - Mass x (distancer' ML2 = [ML2TO] kgm2

25. Radius of gyration Distance L = [MoLTo] m '{

26. Angular momentum Mass x velocity x radius M x LT -I x L = [ML2T -1] kg m2s-I

27. T-ratios Length


~ = 1= [MOLOTO](dimensionless) -
(sin e, cos e, tan 8) Length L

28. Time period Time T=[MOLOT] s


IT
29. Frequency 1 ..!. = rl = [MoLorl) s-l or Hz
Time period T

30. Planck's constant 'h' E


-
Energy ML2r2
___ = [ML2T-1) Js
=
v Frequency T-I

31. Relative density Density of substance


ML -: = 1= [MoLoTo] -
Density of water at 4°C ML-
H
(dimensionless)

32. Velocity gradient Velocity s-l


Lrl = rl = [MOLOT-1)
Distance L

33. Pressure gradient Pressure ML-1T-2 Pam-1


= [ML-2T-2]
Distance
O

L
Force constant Force 2 Nm-1
34. MLr =MT-2 = [MLor2]
Displacement L
no..
.
... . 'i '; .

35. Heat or enthalpy Energy [ML2r2] J


M

36. Specific heat Heat ML2 T-2 J kg-1 K-1


='[MOL2T -2K-I]
Mass x Temperature M·K

37. Latent heat Heat ML2T-2 J kg-1


-- = [MOL2T-2]
Mass M

38. Thermal conductivity Heat x distance ML2 T-2. L Js-1m-1K-1


= [MLT-3 K-1]
Area x temp. x time L2·K·T

39. Entropy Heat ML2r2 = [ML2T-2K-1]


___ JK-1
Temperature K

40. Universal gas constant PV ML-1 T-2L3 J mol-1 K-I


-
nT mol-K
= [ML2T-2K-1 mol-I]
2.22 PHYSICS-XI

S. Relation with other


Physical Quantity Dimensional formula SI unit
No. quantities
41. Boltzmann's constant Energy ML2r2 . JK-1 ,
= [ML2T -2 K-1]
Temperature K

42. Stefan's constant Energy ML2 T-2 Js-1 m" 2 K-4


= [MLOT-3 K-4]
Area x time x (temp.j'' e·T·K4
--

43. Solar constant Energy ML2 T-2 Js-1m-2


= [MLo r3]
Area x time L2·T
- - -

R
~T-2
44. Mechanical equivalent J= W = [MoLoTo] -
of heat H ML2 T-2
(dimensionless)
. :;
. ( -: :- "
.8 -: • .••,.:> ,'d}: .•..- .,.';.:,.!"i':··f;;!~C~,f<.,
...''1\~:,_.

SI
45. Electric charge Time ;Current T, A = [MOLOTA] C (coulomb)

46. Electrical potential Work ML2 T-2 V (volt)


--- = [ML2T -3A -I]
Charge TA
-
47. Resistance Potential difference ML2T-3A -I n(ohm)
= [ML2T -3A -2]
Current A
48. Capacitance Charge TA = [M-1L-2T4A2] F (farad)
Potential difference ML2 T-3i\-1

49. Inductance EMF ML2T-3A -1 H (henry)


= [ML2T -2A -2]
IT
Current / time AT 1

50. Permittivity of free space I: = ql q2 AT·AT A 2c2 N-1m- 2


° Fr
2 MLT 2e
= [M-1C3T4A2]
-
51. Relative permittivity or e, or 1C = 1:0 a pure ratio = [MoLoTo] -
dielectric constant I: (dimensionless) .'
-
H
52. Intensity of electric field E= £. = Force
MLr
2
= [MLT-3A-1] NC10r
q Charge AT Vm-I

53. Conductance C=..! 1 = [M-1L-2T3A2] n-I


R ML2 T 3A 2 or mho
RA ML2 T -3A -2 . L2
O

54. Specific resistance or p=- [ML 3T -3 A-2] nm


resistivity 1 L

55. Specific conductance or 1 [~IL-3T3 A2] n-Im-I


0'=-
conductivity p

56. Electric dipole moment - q x 21 AT.L=[MoLTA] Cm


M

m. . . • ,
','
Magnetic field F 2
57. B=-'-- ML r = [MLoT -2A-I] T (tesla)
qv sin e I
AT·Lr ·1 :-

58. Magnetic flux $=BA MT-2A-1. L2 = [ML2 r2A-I] Wb(weber)


Permeability of free space 4rrr.F 2
59. L·MLr ~ [MLT-2A -2]
1-10 =ITl
A2·L
1 2 _.
60. Magnetic moment Current x area A, L2 = [MoeTo A] Am2
Magnetic moment
'-
61. Pole strength AL2 = [MoLToA] Am
Magnetic length L
UNITS AND MEASUREMENTS 2.23

Exam les based on (iv) Coefficient of viscosity,


Force Force Distance
'11= =--x---
CONCEPT USED . Areax velocity gradient Area Velocity
The dimensional formula of a physical quantity 2
[ ]= MLr ,L = (ML-tT-t],
can be obtained by defining its relation WIth other '11 L2, LT-1
quantities, whose dimensions in M, L and rare
known, (v) Surface tension,
Force
cr=
ExA.,VIPLE 27. Name the physical quantities whose dimensional Length
formulae are as follows : MLT-2

R
[cr]= L' [Mr2).
(i) Me
T-2 (ii) Me T-3 (iii) MT - 2

(iv) ML 1 T -1 (v) ML IT - 2, (vi) Planck's constant,


Solution. (i) ML2 T -2 = ML T -2 ,L h= Energy
Frequency
= Forcex distance = Work.

SI
2 2 2
(")
II
ML2T-3 = ML r ~ =--=
Work P ower. [h] = ML T- = [ML2rl].
T Time - r1
MLT-2 Force 29. Deduce the dimensional formulae of the
Mr 2 = --
EX4MPLE
(iii) =---
L Length following physical quantities:
~
(i) Heat (ii) Specific heat (iii) Latent heat
= Surface tension or force constant.
2 (iv) Gas constant
(iv) ML-1r1 = MLr = Force
(v) Boltzmann's constant
L2 T-1 Area x velocity gradient
IT
(vi) Coefficient of thermal conductivity
= Coefficient of viscosity, (vii) Mechanical equivalent of heat.
(v) ML-1r2 = MLT- 2
Solution. (i) Heat = Energy'
L2
Force .. [Heat] = [ML2r2],
= -- = Pressure or stress,
Area
H
") Spec ifi c h eat = ------,-. Heat
(II ----
EX.4MPLE 28. Deduce the dimensional formulae for the Massx temperature
following physical quantities:
:. [Specific heat] =
ML2r-2 = [L2r2K-1].
(i) Gravitational constant (ii) Power
M,K
(iii) Young's modul~~ (iv) Coefficient of viscosity
O

(v) Surface tension (vi) Planck's constant. ("')


III L atent h eat = --
Heat
Mass
Solution. (I) According to Newton's law of gravitation,
ML2r2
F=G~~ " [Latent heat] = [L2T-2] ,
? M
(iv) PV = nRT
.. [G] = [F][?]
M

R = PV = Force x volume
[~][~] or
nT
molx area x temp.
Work Forcex Distance
(ii) P ower=-- 2 3
Time Time [R] = MLT- , L
mol. L2 ,K
[P ower ] = ML r:"·, L = [ML2 T-3] ,
T = [ML2T-2K-1mol-1]"
(iii) Young's modulus, (v) Boltzmann's constant, _
Y = Longitudmal stress = F / A =.£ , J..- k = Heat
Longitudinal strain Lll/ 1 A Lll Temperature
2
[Y] = MLr , L = [ML-IT-2], ML2T-2
.. L2. L [k] = = [ML2r 2K-1].
K
2.24 PHYSICS-XI

(vi) Coefficient of thermal


conductivity, EXA1"lPI.E 33. If the velocity of light c, acceleration due to
Heat x distance gravity g and atmospheric pressure p are the fundamental
K=---------------------------- quantities, find the dimensions of length.
Areax timex temperature difference
Solution. We have,
[K] = ~2r2. L =[MLT-3K-1]
L . T.K c=LT-l, g=LT-2, p=ML-l T-2
(vii) Joule's mechanical
J= Work
equivalent of heat,
.
2 2
L r
[Length] = L= --2-=
LT"
(Lrl)2
L'I"
[?]
2 = -
g
.
Heat
22 f:X.4lIfPLE 34. The number of particles crossing a unit area
[ J] = ML r = 1 = [MoLoTo].

R
ML T-2
2 .
perpendicular to X-axis in unit time is given by
n= _Dnz-~
ExAMPLE 30. Find the dimensional formulae of (i) charge
X2 -Xl
(iiJ potential (iii) resistance (iv) capacitance.
Solution. (i) Charge, q = Current x time where nl and nz are number of particles per unit uolume.for
the values of X meant to be Xl and x2. Find the dimensions of

SI
[q] = [AT]. the diffusion constant D.
(ii) Potential, n -n
Solution. A? n = - D _2__ 1
V = Work X2 -Xl
Charge
D = n (X2 - Xl)
(numerically)
ML2T-2 (nz-nl)
[V] = = [ML~A-lr3].
AT
Now n = number of particles per unit area per second,
(iii) Resistance,
In] = L-2rl
R = Potential difference
IT
nz - nl = number of particles per unit volume
Current
ML 2A-lr3 [nz _~]=L-3
____ = [ML2A -2T -3].
A x2 - Xl = position
(iv) Capacitance, .. [x2 - xd = L
C = Charge 2 l
Hence [D]= L- T- . L = [L2rl).
H
Potential L-3
[C] = AT = [M-1L-2A2T4]
ML2A-1r3 :-: PROBLEMS FOR PRACTICE

EX.4lI1PLE 31. Taking velocity,


time and force as the 1. Deduce dimensional formulae for (I) angle (il) angular
fundamental quantities; find the dimensions of mass.
O

velocity (iii) angular acceleration (iv) torque (v)


Solution. Force = Mass x Acceleration angular momentum and (vi) moment of inertia.
Velocity [Ans. (i) Dimensionless (ii) T -1 (iii) T - 2
= M ass x -----!-
Time (iv) ML2 T - 2 (v) ML2 T -I (vi) ML2]
2. Obtain dimensions of (i) impulse (ii) power
Force x Time
or Mass=----- (iii) surface energy (iv) coefficient of viscosity (v)
M

Velocity bulk modulus (vi) force constant.


[Mass] = [FTV-1]. [Ans. (i) MLT -1 (ii) ML2T - 3 (iii) MLOT- 2
(iv) ML-I T -I] (v) ML-I T - 2 (vi) MLo T - 2]
EY.4lIfPLE 32. If density p, acceleration due to gravity g and
3. By the use of dimension." show that energy per unit
frequency v are the basicquantities,find the dimensions offorce.
volume is equal to the pressure.
Solution. Wehavep=ML-3, g=Lr2, v=rl 4. Show that angular momentum has the same physical
Solving for M, L and T in terms of p, g and v, we get units as the Planck's constant h which is given by
M = ply - 6~ L = gv - 2, T = v-I the relation E = hv.
5. If force (F), length (L) and time (T) are chosen as the
[Force] = MLT - 2 = pg3v-6 . gv- 2. v2 fundamental quantities, then what would be the
= [pg 4 v -6] . dimensional formula for density? (Ans. FL- 4 rl)
UNITS AND MEASUREMENTS 2.25

6. Calculate the dimensions of force and impulse 2.22 DIFFERENT TYPES OF VARIABLES
taking velocity, density and frequencr as basic AND CONSTANTS
quantities. (Ans. pv4 »: ,p v4 y- 3)
43. How can we classify variables and constants on
7. Find the dimensions of linear momentum and the basis of dimensions ? Give examples of each type.
surface tension in terms of velocity v, density p and Different types of variables and constants. On the
frequency v as fundamental quantities. basis of dimensions, we can classify quantities into four
(Ans. pv4 »: 3, P v3 y-I) categories :
8. In the expression P = EI2 m-5 C-2 ; E, m, I and C 1. Dimensional variables. The physical quantities .
denote energy, mass angular momentum and which possess dimensions and have variable values are called
gravitational constant, respectively. Show that P is dimensional variables.

R
a dimensionless quantity. [Exemplar Problem) Examples. Area, volume, velocity, force, etc.
X HINTS 2. Dimensionless variables. The physical quantities
3. Energy per unit volume which have no dimensions but have variable values are called
Energy _ ML2 . T-2 2 dimensionless variables.
MLr
=----r:

SI
Volume - L3 Examples. Angle, specific gravity, strain, etc.
Force . 3. Dimensional constants. The physical quantities
= -- = Pressure.
Area which possess dimensions and have constant values are
4. Angular momentum, called dimensional constants.
L= mvr Examples. Gravitational constant, Planck's constant, etc.

[L] = MLrl.L = ML2rI 4. Dimensionless constants. The constant quantities


having no dimensions are called dimensionless constants.
As E= hv
Examples. 1t, e, etc.
IT
h=~
Y
2.23 APPLICATIONS OF
ML2 r2 DIMENSIONAL ANALYSIS
[h] = = ML2 rl =[L]
rl
44. Mention some applications of dimensional analysis.
6. v = LT - I, P = ML- 3, y = T - I
Applications of dimensional analysis. The metJiod
Solving for M, L and T in terms of v, p and u, we get of studying a physical phenomenon on the basis of
H
T=y-l, L=Vy-l, M=pV3y-3 dimensions is called dimensional analysis. Following are
[Force] = MLT - 2 =pv3 y-3 . Vy-I . y2 =pv4 y- 2 the three main uses of dimensional analysis:

[Impulse] = Force x time = pV4y-2.y-1 =pV4y-3 1. To convert a physical quantity from one system
of units to another.
7. Using the dimensions of M, Land T as obtained in
O

Problem 6, we get 2. To check the correctness ot'" d given physical


relation.
[ p] [mv] = ML T - I
=
3. To derive a relationship between different
=pV3y-3.Vy-l.y=pV4y-3
physical quantities.
. Force
Surface tension = ---
Length 2.24 CONVERSION OF ONE SYSTEM OF
M

UNITS TO ANOTHER
ML T - 2 = MT- 2 = PV3y- 3y2 = PV3y- 1.
L 45. How can a physical quantity be converted from
one system of units to another? Explain it with the help
8. [E]=ML2T-2, [M]=M,[I]=ML2T-I, of a suitable example.
[C] = Nr1L3T-2 To convert a physical quantity from one system of
units to another. It is based on the fact that the
~] = ML2T-2[ML2T-1]2
magnitude of a physical quantity remains the same,
[M C5 2 M5[Nr1L3T-2 f whatever may be the system of units. If u1 and Uz are the
3T6T- 4 units of measurement of a physical quantity Q and ~
=M L = [MOL°rD] and nz are the corresponding numerical values, then
M3L6T-4
Q=~~ =nzu2
2.26 PHYSICS-XI

Let M i- L1 and T1be the sizes of fundamental units of EXAMPLE 35' The value G in CG5 system is 6.67 x 10- 8
mass, length and time in one system; and M2, L2, T2be dyne ~ g- 2. Calculate the value in 51 units.
corresponding units in another system. If the dimen-
sional formula of quantity Q be Ma l!' r-. then Solution. As F = G ~ "2
~ =Mt L~T/
.. G=--
Fr 2
a Lb 1:2c
and ~ = Mi 2 ~"2
[aM2 L 2b 1:c] 2 2
.. n1 [Ml a Llb T1c] = ~ 2 . [G]= MLr . L = 111L3T-2
MM
or .. a = - I, b = 3, c = - 2

R
CGS units SI units
This equation can be used to find the numerical
11 = 6.67 x 10- 8 ~ =?
value in the second or new system of units.
M1=lg M2 = 1 kg = 1000 g
Example. Let us convert one joule .into erg.
L] = 1cm L2 = 1m = 100 em

SI
Joule is 51 unit of energy and erg is the CG5 unit of
energy. Dimensional formula of energy is ML2T - 2. TI = Is T2 = Is
.. a=l, b=Z, c=-2.
SI CGS
M1 = 1 kg = 1000 g ~=lg
LI = 1 m = 100 em L2 =1 em
TI = Is T2 = 1 s
~ = 1 (joule) ~ = ?(erg) = 6.67 x 10- 11
IT
Hence in SI units,

~ =~ [~:J [~:J a b [~:] c


G= 6.67x 10-11Nm2kg-2•

= 1 [10100 r r [ir
[1~0 2
ExAMPLE

.
36. Find the value of 60 J per min on a system
that has 100 g, 100 cm and 1 min as the base units.
joule 60 joule
H
3 4 7
Solution. P = 60 ~- = = 1 watt
= 1 x 10 x 10 = 10 min 60s
1 joule = 107 erg. which is the SI unit of power.

Now = ML2T-3
[Power]
• • .'. a = I, b = 2 , c = - 3
•., The above conversion technique is applicable to only
O

absolute systems of units. The gravitational or other


practical units must be first converted into absolute SI New System
units before using the above technique. 11 = 1 ~ =?
Exam M, = lkg = 1000g M2 = 100g

= 1m = 100 em
M

LI L2 = 100 em
TI = Is T2 = 1 min = 60 s
FORMULAE USED

1. 11u1 = ~ ~
Tn [~:r[~:n
~:r
~:J rI~:
"' = "t

r
2. 11[M~ L~ =~ [M~ L~ T~]

3. ~ = 11[ a [ ~: = 1 [1000] 1 [100] 2 [~] - 3


100 100 60
UNITS USED
= 2.16 x 106
The SI units of mass, length and time are kg, m and
s and the corresponding CGS units are ~ em and s. .. 60 J min-1 ':"2.16x 106 new units of power.
UNITS AND MEASUREMENTS 2.27

EXi1MPLE 37. In CGS system, the value of Stefan's constant :-: HINTS
is c = 5.67 x 10- 5 erg s- 1 em' 2 K- 4. Find its value in SI
3. [Density] = M1L-3
units. Given 1 J = 107 erg.
Solution. In CGS system, c = 5.67 x 10- 5 erg
~ = ~ [~:J [~:r3

r
s-lcm- 2K- 4. The SI unit of work is joule. We have,
1 erg = 10- 7 J and 1 em = 10- 2 m 3
.,.--" -
:. The value of Stefan's constant in SI units is = 13.6[ 11;gJ [~:
c = 5.67 x 10- 5[10-7 Jl s-l [10- 2mr 2 :rc- 4
= 5,67 x 10- 5 x 10- 7 x 104 Js-1m -2 :rc-4 = 13.6[~] [ 1em ]-3= 13.6 x 103
1000g 100em

R
= 5.67x 10-8 Js-1m -2K-4•
.. 13.6g em - 3 = 13.6 x 103 kgm - 3 •
EXAll-fPLE 38. If the unit offorce is 1 kN, unit of length 1 km
and the unit of time is 100 s, what will be the unit of mass? 4. [Surface Tension] = M1T-2
2 2 2
ti M MLT- . T FT
So Iu Ion. = -- _ 72[~]1 [~]-2
L L

SI
~ - lkg Is
1000 N>.<104 s2 = 104 Ns2
1000m
-2 2
m = 72 [~]1
1000g
[~]-2
Is
= 0.072
= 104 kg m s . s = 104 kg.
m .. 72dyne em-I = 0.072 Nm-1•
6. [Pressure] = MIL-lr2
:-: PROBLEMS F'OR PRACTICE

1. Convert one dyne into newton. [Himachal 091 ~=106[/;gJ[~:rl[~:r2 =105


IT
(Ans. 10-5 newton)
.. 106 dyne em - = 105 Nm -2.
2
2. If the value of universal gravitational constant in SI
is 6.6 x 10-11Nm2kg-2, then find its value in CGS 7. o= 5.67x10-8Js-1m-2K-4
system. [Himachal 091
= 5.67 x 10- 8 x 107 erg s-1 (100 emf 2 K- 4
(Ans. 6.6 x 10-8 dyne em 2g-2)
= 5.67 x 10-5 erg s-l em" 2 K" 4.
3. The density of mercury is 13.6 g em - 3 in CGS 8. [Work] = ML2 T -2
H
system. Find its value in SI units.
(Ans. 13.6x 103 kg m - 3) = 100[~]1 [~]2[_lS ]-2
~ 250 g 20 em 0.5min
4. The surface tension of water is 72 dyne em - 1.
Express it in SI units. (Ans. 0.072 Nm -1)
5. An electric bulb-has a power of 500 W. EXfress it in = 100[ 12~00:J [~O~:
] 1
2 [3 0SS]-2
O

CGS units. (Ans. 5 x 10 erg s-1)


= 100 x 4 x 25 x 30 x 30 = 9 x 106 new units.
6. If the value of atmospheric pressure is 106 dyne
em - 2, find its value in SI units. (Ans. 105 Nm - 2) 9. (i) MLT - 2 = 20 N (ii) ML2 T -2 = 200 J
(iii) LT - 1 = 5 ms - 1
7. In SI units, the value of Stefan's constant is
cr=5.67x10-8 Js-1 m-2 K-4. Find its value in Dividing (ii) by (i), L = 200 = 10 m
M

CGS system. (Ans. 5.67 x 10- 5 erg s-1 em - 2 K- 4) 20


8. Find the value of 100J on a system which has 20 em,
Putting the value of L in (iii),
250 g and half minute as fundamental units of 10 T -1 = 5 or T = 2 s
length, mass and time. (Ans. 9 x 106 new units) From (i),
9. If the units of force, energy and velocity are 20 N, M x 10 x (Z)" 2 = 20 or M = 8 kg.

\:g J [:: J[
200 J and 5 ms - 1,find the units of length, mass and
time. (Ans. 10 m, 8 kg, 2 s) 10. ~ = 36 [ 1 ~sm] -2
10. When 1 m, 1 kg and 1 min are taken as the funda-
mental units, the magnitude of the force is 36 units.
= 36[1000g] [100em]1 [60S] -2 =103 dyne.
What will be the value of this force in CGS system? 19 1cm Is
(Ans. 103 dyne)
2.28 PHYSICS-XI

2.25 CHECKING THE DIMENSIONAL Solution. The dimensions of the various terms are
CONSISTENCY OF EQUATIONS [x] = [L]
46. State the principle of homogeneity of dimensions. [xo] = [L]
What is its basis? [vot] = [Lr1] [T]~ [L]
Principle of homogeneity of dimensions. According [~at2] = [Lr2] [T 2]= [L]
to this principle, a physical equation will be dimensionally
Since the dimensions of all the terms are same,
correct if the dimensions of all the terms occurring on both
hence the given equation is dimensionally correct.
sides of the equation are the same. This principle is based
on the fact that only the physical quantities of the same EXAMPLE 40. Check whether thefollowing equation is dimen-

kind can be added, subtracted or compared. Thus, sionally correct.

R
velocity can be added to velocity but not to force. ~ mv2 = mgh. [NCERT, Himachal 06C]
47. How can we check the dimensional correctness of
a physical equation? Explain it with a suitable example.
Solution. [~ mv2] = [M] [LT" 1 = [ML2r 2] f
To check the dimensional correctness of a physical [mgh] = [M][Lr2][L] = [ML2 r2]

SI
equation. For this purpose we make use of the principle Dimensions of LHS = Dimensions of RHS
of homogeneity of dimensions. If the dimensions of all Hence the given equation is dimensionally correct.
the terms on the two sides of the equation are same,
then the equation is dimensionally correct. RHMPU' 4'. Check the correctness of the equation,
F5 = 1 mv2 _1 mu2
Example. Let us check the dimensional accuracy of 2 2
the equation of motion, where F is the force acting on a body of mass m and 5 is the
distance moved by the body when its velocity changes from u
s = ut + ~ at2
2 to v. [Delhi 08]

Dimensions of different terms are Solution.


IT
[s] = [L] L= ML2r2
[F5] = MLr2.
[ut] = [Lr 1] [T] = [L 1 [~mv2] = M[Lr1]2 = ML2r2
[~mu2] = M[Lr1f = ML2r2
[~at2 ] = [Lr2] [T2]= [L]
Since dimensions of all the terms in the given equation
are same, hence the given equation is dimensionally
H
As all the terms on both sides of the equations have
the same dimensions, so the given equation is dimen- correct.
sionally correct. EXA.Mf>U·; 42. Check the correctness of the relation r = la,
where 't is the torque acting on a body, 1 is the moment of
inertia and a is angular acceleration. [Punjab 1990]
A' A dimensionally correct equation need not be actually
O

Solution. Given t = la
a correct equation, but a dimensionally inconsistent
equation must be wrong. The equation of motion : As torque, t = Force x distance
s = ut + at2 is dimensionally correct but numerically it ['t] = MLr2. L= Me r2
is wrong.
Moment of inertia,
ExamR./es based on
M

1 = Mass x distance/
2
[l] = ML
Angular acceleration,
CONCEPT USED Angle
a = ---"---;:-
By the principle of homogeneity of dimensions, a (Timel
physicalrelation will be dimensionally correct if the
dimensionsof allthe terms in the equationare the same. [a]=~=T-2
T2
EXA.Mf>l.E 39. The distance x travelled by a body in time t
[Ia] = ML2r2
which starts from the position Xo with initial velocity Voand
has uniform acceleration a, is given by x = Xo + vot + ~ at2 . .'. Dimensions of LHS = Dimensions of RHS
Check the dimensionally consistency of this eouation. Hence the given equation is dimensionally correct.
UNITS AND MEASUREMENTS 2.29

ExAMPLE 43. Check the dimensional consistency of the


following equations : ._ (iv) RHS =..!. rITI =! [ ML r
2
] 1/2,
"-" h
[I] V(r;;] L ML-1
(i) de-Broglie wavelength, A = -
mv =!,LT-1=r1
(/'1') Escape ve IOCI'ty ,v = lR' GM
[Himachal 05] [LHS] = [v]=
L
r'.
Solution. (i) Given A"; ~ Hence the relation is correct.
mv EX.4.MPLE 45. By the method of dimensions, test the accuracy
As wavelength is a distance, mgl3
of the equation: 0= 3
=L

R
.. [A] 4bd Y
h ] Planck's constant where 0 is the depression produced in the middle of a bar of
A Iso [ - = -------
mv Mass x velocity length I, breadth b and depth d, when it is loaded in the
middle with mass m. Y is the Young's modulus of the
.: Dimensions of LHS = Dimensions of RHS
material of the bar.
Hence the given equation is dimensionally consistent.

SI
(")
II G'iven v = l GM
-R-
Solution. [LHS] = [0] = depression = L
3
[RHS] = [ mg 1 ]
4bd3 Y

r
[v]=LT-1
1 1 Mass x acceleration x length '

[2~MJ2 = [M-
1L3
Z- 2
,M = [L2 T- 2]~ = LT-1 4 x breadth x depth ' x Young's modulus
3
M. Lr2. L =L
': Dimensions of LHS = Dimensions of RHS L. L3 . ML-1 r2
IT
Hence the equation is dimensionally correct. .. [LHS] = [RHS]. Hence the relation is correct.
EXA.1UPLE 44. Check by the method of dimensions whether 46. Find the dimensions of a/b in the equation:
EX"lL'lrIPU,
the following equations are correct:
(i) E = m? (ii) T = 21t H [Delhi 98]
F = a../X + bt2, where F isforce, x is distance and t is time.
Solution. [a../X] = [F]
2
f-!, l£l
H
.. [a] = = MLr =ML!/2 r2
(iii) v = where v = velocity of sound, [../X] L1/2

[bt2]=[F]
P = pressure and p = density of medium, 2

2\ l,
[b]=l£l=MLT- =MLT-4
(iv) v = where _v = frequency 'of vibration, [t2] T2
O

I= length of the string, T = tension in the string and ML1/2 r2


[a/b]= =L-1/2T2•
m = mass per unit length, MLr4

Solution. EXA1~1PLE 47. Find the dimensions of a x b in the relation:


(i) [L.HS] = [E] = Energy = ML2r2 P'IS power,
P = --b - x2 h
; were x IS, diisiance an d t . time.
IS
,
M

[R.HS] = [m?J = M[Lr1]2 = ML2r2 at


L2
Hence the relation is correct. Solution. ,[b]=[x2]=L2 :. [P]=-
[at]
(ii) [RHS] = V f~~
illI = [L 1-"
r' 1/2
~ =T
or [a]=_L_=
2
[P] [t] ML2r3.
L2
T
=~lT2

[LHS] = [T] = T .
Hence [a x b] = M-1 L2 T 2.
Hence the relation is correct.
1 EXAMPLE 48. The Vander Wall's equation for a gas is
(iii) [RHS] = lID
=[ ML- !-2 ]1/2 = Lr1
(p :2)
VfPi
ML 3 . + (V - b) = RT

[LHS] = [v] = Lr1. Determine the dimensions of a and o. Hence write the SI
Hence the relation is correct. units of a and b. [Himachal 06C ; Delhi 95]
2.30 PHYSICS-XI

Solution. Since dimensionally similar quantities


can be added or subtracted, therefore, X PROBLEMS FOR PRACTICE

1. Test the dimensional consistency of the following


[P] = [: 2] equations: (i) v = u + at (ii) s = ut + !at2
(iii) v2 - if = 2as ._ [Himachal 07C]:
or [a] = [PV 2] = [ML-1 T -2][L3]2 = MLST-2
(Ans. All relations are dimensionally correct)
Also [b] = [V] = L3. 2. The viscous force 'F' acting on a small sphere of
The SI unit of a is kg mSs-2 and that of b is m'. radius 'r' moving with velocity v through a liquid is
EXAJVIPLE 49. When white light travels through glass, the given by F = 61tT]rv. Calculate the dimensions of ",
(Ans. 1'1--1 T -1)

R
the coefficient of viscosity.
refractive index of glass (Il = velocity of light in air/velocity
of light in glass) is found to vary with wavelength as 3. The distance covered by a particle in time t is given
Il = A + ~. Using the principle of homogeneity of dimensions, by x = a + bt + ct 2 + dt 3 ; find the dimensions of
A: a, b, cand d. (Ans. L, Lr1, Lr2, Lr3)
find the 51 units in which the constants A and B must be
4. The critical velocity of the flow of a liquid through a

SI
expressed.
pipe of radius r is given by Vc = K" where p is the
· H Velocity of light in air rp
So Iution. ere Il = ---..!..---'=-----
_Velocity of light in glass density and " is the coefficient of viscosity of the
= a dimensionless number liquid. Check if this relation is dimensionally
correct. (Ans. Correct)
[A] = [Il] = a dimensionless number 5. The rate of flow (V) of a liquid flowing through a
pipe of radius r and a pressure gradient (P/l) is
As [ ~ ] = [Il] 1t P r_4
given by Poiseuille's equation: V =
IT
= 1 . L2 = L2
[ B] = [Il] [')...2] 8 ,,1
Check the dimensional consistency of this equation.
Hence A, being dimensionless, has no units and SI
(Ans. Correct)
unit of B is m2.
6. Testif the followingequation is dimensionallycorrect:
EXA1VIPLE 50. In the equation: y == a sin (rot - kx), t and x h=2Scos8
stand for time and distance respectively. Obtain the rpg
H
dimensional formula for co and k.
where h = height, 5 = surface tension, p = density,
Solution. An angle is a dimensionless quantity,
r = radius, and g = acceleration due to gravity.
.. [rot] = 1 or [ro] = ~ =.!. = T-1• (Ans. Correct)
[t] -T
7. Find the dimensions of the quantity vin the equation,
1tp(a2 _x2)
O

[kx] =1 or [k] = ~ -=.!. = L-1•


[x] L v = , where a is the radius and I is the
2,,1
EXAMPLE 51. Rule out or accept the following formulae for length of the tube in which the fluid of coefficient of
1cineticenergy on the basis of dimensional arguments: viscosity" is flowing, x is the distance from the axis
of the tube and p is the pressure difference.
(i) ~mv2 (ii) .!.mv2 + ma [Central Schools 08]
M

(Ans. LT -1)
16 2
1 2 8. Find the dimensions of the quantity q from the
Solution. K.E. =-mv i
2 expression. T = 21t ~,- where T is the time
Dimensions of K.E. = [ML2T-2] ~
3Yq .
period of a bar of length I, mass and Young's
W Dimensions of :6 mv 2
= [ML2r2] modulus Y.
m
(Ans. L4)
9. An artificial satellite of mass m is revolving in 'a
This formula is dimensionally acceptable for K.E. circular orbit around a planet of M and radius R. If
the radius of the orbit of the satellite be r.
(ii) [~mv2] +[ma] = [ML2r2] + [MLr2]
Justify by the method of dimensions that the time
This formula cannot be accepted for K.E. as it is period- of the satellite is given by: T = 21t
.~~.
dimensionally inhomogeneous. R g
UNITS AND MEASUREMENTS 2.31

10. Find the dimensions of (a x b) in the equation :


b _x2
E = -- ; where E is energy, x is distance and t is
at
time. (Ans. M-1 1) e 9. [1'] = T
11. Find the dimensions of (a/b) in the equation:
a - t2
21t
[ R
P"]
fg
= 1. [~]1/2
L LT-2
= T.
P=--
bx
where P is pressure, x is distance and t is time. Hence the relation is correct.
T2
(Ans. MT-2) 11. [a]=[t2]=T2 :. [P]=-

R
[bx]
12. Time period of an oscillating drop 'of radius r,

density p and surface tension S is : t = K ~ or [b]=~= T2 =M-1T4


[P][x] ML-1r2. L

Check the correctness of the relation. [Himachal 04] [a/b]=~12 4 = Mr2.


M-T

SI
(Ans. correct)
13. Out of formulae (i) y'= a sin 21tt / T and (ii) y = asin vt 3
for the displacement y of particle undergoing a
12. [RHS] = [K fPr3]
Vs
= [[ML- ] ~L3]]l/2 = [1'] = [LHS]
[MT-]
certain periodic motion, rule out the wrong formula
on" dimensional grounds. (where a = maximum 13. Refer to the answer of NCERT Exercise 2.14.
displacement of the particle, v = speed of the
particle, T = time period of motion). [Delhj 09]
2.26 DEDUCING RELATION AMONG THE
X HINTS PHYSICAL QUANTITIES
IT
1. (i) [v] = L T - I, [u] = L T - I, 48. How can the method of dimensions be used to
[at]=LT-2.T=LT-1 deduce a relation among the physical quantities? Explain
it with the help of a suitable example.
(ii) [s]=L, [ut]=Lr1.T=L,
To derive the relationshiji among physical
2 2 quantities. By making use of the homogeneity of
[~at2] = Lr .T =L
dimensions, we can derive an expression for a physical
H
(iii) [v2]=[LT-1 f =L2 T-2, quantity if we know the various factors on which it
[if] = [Lr1f = L2 r2 depends.
[2 aSl = Lr2. L= L2 r2 Example. Let us derive an expression for the
centripetal force F acting on a particle ot mass m
4. [vcl = critical velocity = LT-1
moving with velocity v in a circle of radius r.
O

1 1
[K'rpIl] = 1. ML- r = Lr1 Let F ex nf vb rC or F =K nf vb rC ••• (1)
L. ML-3 where K is a dimensionless constant. Writing the dimen-
Hence the relation is correct. sions of various quantities in equation (1); we get
S. [V] = Rate of flow of liquid [MLr2l = 1[Mt[LT-1t[Lt
= Volume / Time = L3r1
M

or M1L1T-2= MaLb+cT-b.
2: Pr4 ] = ML-1 r2 . L4 = L3r1 Comparing the dimensions of similar quantities on
[ 8 'Ill ML-1 r1 . L both sides, we get

Hence the relation is correct. a=l


6. [h] = L b+c=l or c=1-b=1-2 =-1·
2 -2 =- b or b=2
2 Scos e]= MT- . 1 =L
[ rpg L . ML 3 . LT 2 From equation (I), we get

Hence the relation is correct. 2 -1 mv2


F= Kmv r =K--
r
This is the required expression for the centripetal
force.
2.32 PHYSICS-XI

Exam Equating the powers of M, L and T on both sides,


b=O, a-3b+c=1, -2c=-1

CONCEPT USED
On solving, a=-
1
2'
b= ° r
c =-
2
1

1 1
By using the principle of homogeneity of dimen- .,.. v=KA2pOg2=KJfi.
sions, the form of expression for a given physical
quantity can be obtained if we know the factors EXA.'HPLE 54. Assuming that the mass M of the largest
upon which that physical quantity depends. stone that can be moved by a flowing river depends upon 'u'
the velocity, 'p' the density of water and on 'g', the accele-

R
EXA.lUPLE {;2. Consider a simple pendulum, having a bob ration due to gravity. Show that M varies with the sixth
attached to a string, that oscillates under the action of the power of the velocity of flow.
force of gravity. Suppose that the period of oscillation of the Solution. Let M = Kva pb gC ... (i)
simple pendulum depends on (i) mass m of the bob (ii) length
where K = a dimensionless constant.
I of the pendulum and (iii) acceleration due to gravity g at
Dimensions of the various quantities are:

SI
the place. Derive the expression for its time period using
method of dimensions. [NCERTI [M]=M, [v]=LT-1, [p]=ML-3, [g]=LT-2
Solution. Let us assume that T ex: rrf lb gC Substituting these dimensions in equation (i), we get
or T = Krrf lb gC ... (i) [M] = [Lrl]a [ML-3]b[Lr2 r
where K is a dimensionless constant. M1LoTo = Mb La -3b+c T"" -2c

The dimensions of various quantities are Equating the powers of M, L and T, we get
[T]=T, [m]=M, [1]=L, [g]=LT-2 b=l, a-3b+c=0, -a-2c=0
IT
Substituting these dimensions in eqn. (I), we get On solving, a =6, b = 1, c =-3
T=[Mt [Lt [LT-2 r M = Kv6 pI g-3

or MOLoTI= M" Lb+ -r - 2c Hence M ex: v6.

Equating the exponents of M, L and T on both EXAMPLE 55. The velocity of sound waves 'u' through a
sides, we get 1 medium may be assumed to depend on :
H
a=O, b+c=O, -2c=1 (i) the density of the medium 'd' and
1 (ii) the modulus of elasticity 'E'.
On solving, a =0, b =~, c=--
2 Deduce by the method of dimensions the formula for the
velocity of sound. Take dimensional constant K = l.
T = Kmo/l/2g-1/2 =K IT Solution. Let the velocity of sound waves be given by
-. Vg
O

From experiments,K =21t.


v = Kd a Eb ... (i)

Therefore T = 21t if . where K = a dimensionless


Dimensions of the various quantities are
constant.

[v] = LT - 1, [d] = ML- 3 ,-


EXA1UPl£ 53. The velocity 'o' of water waves depends on
M

2
the wavelength 'A', density of water 'p' and the acceleration [E]= stre~s = force. MLr =ML-1r2
due to gravity 's: Deduce by the method of dimensions the stram area x stram L2 • 1
relationship between these quantities. [Central Schools 081 Substituting these dimensions in equation (I), we get
Solution. Let v = K J!pbl ... (i) [Lrl] = [ML-3]a [ML-1r2t
where K = a dimensionless constant. or ~Llrl = Ma +b L-3a -b T-2b

Dimensions of the various quantities are Equating the dimensions of M, L and T, we get
[v]=LT-1, [A]=L, [p]=ML-3, [g]=LT-2 a+b=O, -3a-b=1, -2b=-1
Substituting these dimensions in equation (i), we get 1 1
On solving, a = - - b =-
[Lrl] = [L]a [ML-3]b.[Lr2 r 2' 2

MOL1T-1 = MbLa - 3b + C T- 2c v = Kd" 1/2El/2 or v=~. [.: K=l]


UNITS AND MEASUREMENTS 2.33

EXAMPLE 56. Thefrequency 'v' of vibration of a stretched T = Ky3/2~1/2G-l/2


string depends upon :
K2 r3
(I) its length I, or T2= --
MG
(ii) its mass per unit length 'm' and
T2 oc~.
(iii) the tension T in the string.
Obtain dimensionally an expression for frequency v. EXAiVIPLB 58. Reynold number NR (a dimensionless
[Delhi 2002] quantity) determines the condition of laminar flow of a viscous
88tutioft.. Let the frequency of vibration of the
liquid through a pipe. NR is a function of the density of the
liquid 'p', its average speed 'v' and coefficient of viscosity '1'(
string be given by
Given that NR is also directly proportional to '0' (the

R
v = KI a nf T C ••• (i)
diameter of the pipe), show from dimensional considerations
where K = a dimensionless constant. that N oc·p v 0 .
Dimensions of the various quantities are : R "
[v] = T -1, [I] = L, [T] = Force = MLr2 The unit of" in 51 system is kg m' 1 S-l.

v""c

SI
Solution. Let NR = K pa 0 ...(i)
[m] = mass = M = ML 1 ,
length L . where K = a dimensionless constant.
Substituting these dimensions in equation (I), we get Dimensions of various quantities are
[r1] = [L]a [ML-1]" [MLr2]C [NR] = 1= ~ LOTO
01' ~LOrl :::M"+c La -"+C T -2c [p] = ML- 3, [v] = LT - 1,
Equating the dimensions of M, L and T, we get [,,]=Mr;l r= '. [O]=L
b+c=O, a-b+c=O, -2c=-1 Substituting these dimensions in equation (i), we get
IT
1 1
On solving, a = -1, b = - -, c = -. [~L0r>] = [ML-3]a [Lr1]"[ML-1r1]C[L]
2 2
v = Kl-1m-1/2T1/2
Equating the powers of M, L and T, we get
or v=~ ~.
a+c=O, -3a+b-c+l=O, -b-c=O
EXAMPLE 57' A planet moves around the sun in nearly On solving, a = 1, b = 1, c = -1.
H
circular orbit. Its period of revolution 'T ' depends upon : PvO
.. NR--K pv"
11 -10_K---
(I) radiue 'r' of orbit (ii) mass 'M ' of the sun and
(iil) the gravitational constant G.
or N oc
PvO "
Show dimensionally that T2 oc ~. R "
O

Solution. Let T = K,a M" GC ••• (i)


EXAMPLE 59. Deriue by the method of dimensions, an
where K = a dimensionless constant. expression for the volume of a liquid flowing out per second
through a narrow pipe. Assume that the rate offlow of liquid
Dimensions of the various quantities are :
depends on
[T] = T,
[r] = L, [M]= M
(I) the coefficient of viscosity ',,' of the liquid
2 2
M

[G]=~ MLr .L =~lL3r2. (ii) the radius 'r' of the pipe and
~~ MM
Substituting these dimensions in equation (I), we get (iii)· the pressure gradient ( f) along the pipe.
[T] = [L]a [M]"[~ 1L3r21'
Take K =~.
~LoT1 = M"-c I! +3c r2c 8
Equating tl\e dimensions of M, L and T, we get Solution. Let volume flowing out per second
through the pipe be given by
b-c=O, a+3c=O, -2c=1
On solving,
1. 1
.V = K,,: r" (fJ ...(1)
3
a=-, b=--, c=--
2 . 2 2 • where K = a dimensionless constant.
2.34 PHYSICS-XI

Dimensions of the various quantities are Dimensions of various quantities are


3 [v] = T - \ [r] = L, [p] = ML- 3, [S] = MT - 2
[V] = volume = L = L3r1
time T Substituting these dimensions in equation (i), we get
[11]=ML-1 T-1, [r]=L T-1 =[Lt [ML-3]b [MT-2 r
p] pressure force or M LDr-1 = Mb+c L" -3b r2c
O

[I = length = area x length Equating the powers of M, L and T on both sides,


2 b+c=O, a-3b=O, -2c=-1
MLr =ML-2r2
L2. L On solving,
3 1 1

R
Substituting these dimensions in equation (I), we get a=-2' b=-2' c=2
[L3r1] = [ML-1r1t [L]b [ML-2r2 r
MOL3r1 = Ma +c L-a +b-2c T"" -2c
v = Kr" 3/2 p- 1/2 S1/2 =K fI.
or ~pr3
Equating the powers of M, L and T, we get EXAMPLE 62. The escape velocity v of a body depends upon

SI
a+c=O, -a+b-2c=3, -a"":2c=-1 (i) the acceleration due to gravity of the planet and (ii) the
On solving, radius of the planet R. Establish dimensionally the relation-
a = - I, b = 4, c = 1 ship between v, g and R. [Himachal 06 i Delhi 05)
Solution. Let v = Kga Rb
V = K 11- 1 4
r ( fY where K = a dimensionless constant.
4 Putting the dimensions,
1t r p
or V = -- [Poiseuille's equation]
8111 Lr1 = [Lr2t[Lt = La+b r2a
IT
EXAMPLE 60. The period of vibration of a tuning fork Equating the powers of Land T,
depends on the length 1 of its prong, density d and Young's a + Ii = I, - 2 a = - 1
modulus Y of its material. Deduce an expression for the
period of vibration on the basis of dimensions. a=.!. b=.!.
2' 2
Solution. Let T = Kl" db Y c ... (i)
Hence v = K gl/2 R1/2 = K.fiR.
H
where K = a dimensionless constant.
Dimensions of various quantities are EXAMPLE 6.1. A large fluid star oscillates in shape under
the influence of its own gravitational field. Using
[T]=T, [1]=L, [d]=ML-3, r"j=ML-1r2
dimensional analysis, find the expression for period of
Substituting these dimensions in equation (i), we get oscillation (T) in terms of radius of star (R), mean density of
O

. T=[Lt [ML-3]b [ML-1 T-2]C fluid (p) and universal gravitational constant G.
[Chandigarh 04)
or ~LoT1 = Mb+c La-3b-c r2c
Solution. Let T = KRapbGc
Equating the powers of M, L and T, we get
b+c=O, a-3b-c=O, -2c=1
where l< is a dimensionless constant.
Putting the dimensions,
M

On solving,
a=l, b=.!. c=-.!. T1 = [Lt [ML-3]b[M-1L3r2 r
2' 2 or
T = K 11d1/2y-1/2 = Kl ~. Equating the powers of M, L and T, we get

EXAMPLE 61. The frequency v of an oscillating drop may b - c = 0, - 3b + 3c + a = 0, - 2 c = 1


depend upon radius r of the drop, density p of the liquid and On solving,
surface tension S of the liquid. Establish an expression for v 1 1
dimensionally. a = 0, b = - -, c=- -
2 2
Solution. Let v = K,a pb SC •.• (i)
T = KRop-1/2G- 1/2 =K _1_ .
where K = a dimensionless constant. .JPG
UNITS AND MEASUREMENTS 2.35

11. A U-tube of uniform cross-section contains mercury


X PROBLEMS FOR PRACTICE upto a height h in either limb. The mercury in one
1. The wavelength A.associated with a moving electron limb is depressed a little and then released. Obtain
depends on its mass m, its velocity v and Planck's an expression for the time period of oscillation
assuming that T depends on h, p and g.
constant h. Prove dimensionally that A. ex: ~.
mv
[Himachal 04 ; Chandigarh 08] (Ans. T= Kj¥)
2. Obtain an expression for the centripetal force F
12. The critical angular velocity we of a cylinder inside
acting on a particle of mass m moving with velocity
another cylinder containing a liquid at which its
vin a circle of radius r. Take dimensionless constant
K=1 [Himachal 2000] turbulence occurs depends on viscosity TJ,density p

R
and the distance d between the walls of the
( Ans. F = m;2) cylinder. Find an expression for we .

3. The orbital velocity v of a satellite may depend on (Ans. We= pKd iJ


its mass m, the distance r from the centre of the
13. A body of mass m is moving in a circle of radius r

SI
earth and acceleration due to gravity g. Obtain an
expression for its orbital velocity. (Ans. v = KJTi) with angular velocity co. Find expression for
centripetal force acting on it by the method of
4. A small spherical ball of radius r falls with velocity v
dimensions. [Himachal 03, 09C] (Ans. F = Kmr(2)
through a liquid having coefficient of viscosity 1'].
Find the viscous drag F on the ball assuming it 14. Consider a simple pendulum. The period of
depends on Tl,r and v. Take K = 61t. oscillation of the simple pendulum depends on its
(Ans. F = 61tTlrv) length'/' and acceleration due to gravity 'g'. Derive
the expression for its period of oscillation by the
5. The velocity of a freely falling body is a function of
method of dimensions. [Himachal 06, 06C, 07]
IT
the distance fallen through (h) and acceleration due
to gravity g. Show by the method of dimensions X HINTS
that v = K..[iii.
1. Let
6. Using the method of dimensions, derive an expression
for the energy of a body executing SHM ; assuming MOLlTO = [Mt [Lrlt[ML2rl]'
this energy depends upon its mass m, frequency y = Ma+eLb+2cT-b-e
and amplitude of vibration r. [Himachal 06]
a + c = 0, b + 2c = 1, -b - c = 0
H
..
(Ans. E= Kmy2,1)
On solving, a = -1, b = -1, c = 1
7. A body of mass m hung at one end of the spring executes
SHM. Prove that the relation T = 21tm / k is incorrect, .. A. = k ~ or A. ex: ~ •
mv mv
where k is the force constant of the spring. Also b
4. Let F = K TJar v', then
derive the correct relation. (Ans. T = K .Jm/ k)
O

MI LI T-2 = [ML-I T-It [L]b [LT-I],


8. Assuming that the critical velocity ve of a viscous
= MaL-a +b+e T-a-e
liquid flowing through a capillary tube depends
only upon the radius r of the tube, density p and the .. a=l, -a+b+c=l, -a-c=-2
coefficient of viscosity Tl of the liquid, find the On solving, a = b = c = 1
expression for critical velocity. (Ans. v = KTl)
rp Hence F = KTJrv= 61tTlrv. (Stoke's law)
M

e
6. Let E = Kma yb r, then
9. By the method of dimensions, obtain an expression
.for the surface tension 5 of a liquid rising in a MIL2T - 2 = [M]" [T -It [L]e = M" Le T- b
capillary tube. Assume that the surface tension .. a = 1, b = 2, c = 2 Hence E = Kmy2 ,1.
depends upon (i) mass m of the liquid (ill pressure p
of the liquid and (iii) radius r of the capillary tube. 7. [LHS) = [T) = T
Take K= 1/2 (Ans. 5= pr/2)
10. The depth x to which a bullet penetrates a human
[RHS) = [ 2:m]
= ;:_ 2 = T 2
body depends upon (i) coefficient of elasticity TJand 2
.: [k)= Force =MLr =MT-2]
(ii) kinetic energy ~.·By the method of dimensions, [
show that: x ex: [ ~J/3. . [LHS]
Length

*- [RHS].
L

Hence the relation is incorrect.


2.36 PHYSICS-XI

To find the correct relation, let T = K mQ v, then 4. It fails to derive relationships which involve
T1 = [Mt[MT - 2]b = M" +b T- 2b
trigonometric, logarithmic or exponential functions.
5. Sometimes, it is difficult to identify the factors
.. a + b = 0, - 2b = 1 on which the physical quantity depends. The
On solving, a = 1 b = _1 method becomes more complicated when dimen-
2' 2

Hence T = K,J/2 k-1/2 = K Iff. sional constants like G, h, etc. are involved.

2.28 .,. SIGNIFICANT FIGURES


8. Let Vc = Kr" pb 11c , then 50. What is meant by significant figures in a
LT -1 = [L]a [ML- 3t [ML-1 T -1]' measured quantity ?

R
= Mb+cLa -3b-cT-c
Significant figures. The significant figures are
normally those digits in a measured quantity which are
.. b + c = 0, a - 3b - c = 1, - c =-1 known reliably or about which we have confidence in our
On solving, measurement plus one additional digit that is uncertain.
a=-l, b=-l, c=l The larger the number of significant figures in a

SI
measurement, the higher is the accuracy of the
v = Kr-1 p-1111 = K11 measurement. Suppose the time period of a simple
c rp
pendulum is 1.62s. This digits 1 and 6 are reliable and
10. Let x = K11Q [E,l, then certain, while the digit 2 is uncertain. So the time
period has three significant figures. Again, suppose
L1 = [ML-1 T- 2]Q [ML2 T - 2t
the length of an object is measured as 273. 6 em. It has
= Ma +b L-a +2b T-2a -2b four significant figures. The digits 2, 7 and 3 are
.. a + b = 0, - a + 2b = 1, - 2a - 2b = a reliable while the digit 6 is uncertain,

f' f 51. State the rules for counting the number of


IT
On solving, a =- b=

Hence x = Kl1-1/3 Fi/3 = K[ ~J'3. significant figures in a measured quantity.


Rules for determining
figures:
the number of significant

12. Let Olc = K11a pb a ", then (/) All non-zero digits are significant. So 13.75 has
four significant figures.
T-1 = [ML-1 T-1 t [ML-3]b [L]'
H
(ii) All zeros between two non-zero digits are signi-
= Ma + b L- a - 3b + c T- a
ficant. Thus 100.05 km has five significant figures.
a + b = 0, - a - 3b + c = 0, - a = - 1 (iit) All zeros to the right of a non-zero digit but to the left
On solving, of an understood decimal point are not significant.
For example, 86400 has three significant figures.
a=l, b=.-l, c=-2
But such zeros are significant if they come from a
O

Hence Olc = K 111 P- 1 d" 2 = K~ . measurement. For example, 86400 s has five
pd significant figures,
(iv) All zeros to the right of a non-zero digit but to the left
of a decimal point are significant. For example,
2.27 .,. LIMITATIONS OF DIMENSIONAL
648700. has six significant figure.
M

ANALYSIS
(v) All zeros to the right of a decimal point are signi-
49. Mention the limitations of the method of dimen- ficant. So 161 em, 161.0 em and 161.00 cm have
sional analysis. three, four and five significant figures respectively.
Limitations of the method of dimensions: (vi) All zeros to the right of a decimal point but to the left
1. The method does not give any information of a non-zero digit are not significant. So 0.161 em
about the dimensionless constant K. and 0.0161 em, both have three significant
2. It fails when a physical quantity depends on figures. Moreover, zero conventionally placed
more than three physical quantities. to the left of the decimal" point is not significant.
3. It fails when a physical quantity (e.g., (vii) The number of significant figures does not
t
s = ut + at2) is the ~um or difference of two or depend on the system' of units. So 16.4 em, 0.164 m
and 0.000164 km, all have three significant figures.
more quantities.
UNITS AND MEASUREMENTS 2.37

• les based on
•\. In scientific notation, a number is expressed in the
power of 10 as a x io", where a is the number FORMULAE USED
between 1 and 10, and b is any positive or negative
exponent of 10. The decimal point is written after the i. Rules for rounding off ~ measurement, Refer-to
Q. ,52. " q ,ii "" ''. ""';;::;
first digit. Suppose the length of a rod is reported as
3.500 m. In scientific notation, it can be expressed in 2. RUles' for counting significant figures.' Refer to
different units as . Q.Sl.
3500 m = 3.500x 102 cm= 3.500 xl03 mm 3. Significant figures in the sum or difference of
= 3.500 x l O" 3 km. two numbers. The final result should be reported

R
.\. We retain only those zeros in the base number which to the same number of decimal places as that of the
are the result of a measurement. Now the power of 10 number with minimum number of decimal places.
is not relevant to the determination of significant 4. Significant figures in the product or quotient of two
figures. Each of the above numbers has four numbers. The final result should be reported to the
significant figures. same number of significant figures as that of t;p.e

SI
'humber with minimum numj:)erof sigriliicantfi~.,
52. State the rulesfor rounding off a measurement.
Rules for rounding off a measurement: EXAil'IPLE 64. State the number of significant figures in the
(i) If the digit to be dropped is smaller than 5, then
following: (i) 2.000 m (ii) 5100 kg (iii) 0.050 em.

the preceding digit is left unchanged. Solution. (i) Four: 2, 0, 0, 0 (iz) Four: 5, 1,0,0
(ii) If the digit to be dropped is greater than 5, then (iii) Two: 5, O.
the preceding digit is increased by 1.
EXAM.PLE 65. Round off thefollowing numbers as indicated:
(iiz) If the digit to be dropped is 5 followed by
(i) 18.35 upto 3 digits (ii) 143.45 upto 4 digits
IT
non-zero digits, then the preceding digit is
increased by 1. (iii) 18967 upto 3 digits (iv) 12.653 upto 3 digits
(iv) If the digit to be' dropped is 5, then the (v) 248337 upto 3 digits (vi) 321.135 upto 5 digits
preceding digit is left unchanged if it is even. (vii) 101.55 x 106 upto 4 digits
(v) If the digit to be dropped is 5, then the (viii) 31.325 x 10- 5 upto 4 digits.
preceding digit is increased by 1 if it is odd. 'Solution. (i) 18.4 (ii) 143.4 (iii) 19000
H
53. State the rules for determining the significant (iv) 12.7 (v) 248000 (vi) 321.14
figures in the sum, difference, product and quotient of (viz) 1016 x 106 (viiz) 31.32 x 10- 5 .
two numbers. EXAJUPLE 66. Add 7.21, 12.141 and 0.0028, and express
Arithmetic operations with significant figures. the result to an appropriate number of significant figures.
O

The result of an arithmetic operation involving Solution. 7.21


measured values of quantities cannot be more accurate
+ 12.141
than the measured values themselves. So certain rules
have to be followed while doing arithmetic operations
+ 0.0028
with significant figures so as to ensure that the Sum =193538
M

precision of the final result is consistent with the Corrected sum = 1935 [Rounded off upto
precision of the original measured values. 2nd decimal place]
1. Significant figures in the sum or difference of two Here 7.21 has minimum number of decimal places
numbers. In addition or subtraction, thefinal result should (two), so result is rounded off upto second place of
be reported to the same number of decimal places as that of decimal point.
the original number with minimum number of decimal EXAMPLE 67. Subtract 4.27153 from 6.807 and express the
places. result to an appropriate number of significant figures.
2. Significant figures in the product or quotient of Solution. 6.807
two numbers. In multiplication or division, thefinal result -4.27153
should be reported to the same number of significant figures Difference = 2.53547
as that of the original number with minimum number of Corrected difference = 2.535 [Rounded off upto
significant figures. 3rd decimal place]
2.38 PHYSICS-XI

Here 6.807 has the lesser number of decimal places EXAMPLE 71.. The radius of a sphere is 1.41 em. Express its
(three), so difference is rounded off to 3rd place of volume to an appropriate number of significant figures.
decimal point. Solution. Radius of the sphere, r = 1.41 ern
EXAMPLE 68. Subtract 2.5 x 10- 6 from 4.0 x 10- 4 with (3 significant figures)
due regard to significant figures.
Volume of the sphere = i rrr3
Solution. Le x = 2.5 x 10- 6 = 0.0000025 3
(2 significant figures) = ix 3.14x (1.41)3 ern3 = 11.736 ern3
3
y = 4.0 x 10- 4 = 0.00040
(2 significant figures) = 11.7 cm3.

R
[Rounded off upto 3 significant figures]
y - x = 0.00040 - 0.0000025 = 0.0003975
= 3.975 x 10- 4 = 4.0 x 10-4 EXAMPLE 72. The length and the radius of a cylinder
[Rounded off upto 2 significant figures] measured with slide callipers are found to be 4.54 em and
1.75 em respectively. Calculate the volume of the cylinder.
EXAMPLE 69. Solve the following and express the result to

SI
Solution. Length of cylinder, h = 4.54 em
an appropriate number of significant figures:
(i) Add 6.2 g, 4.33 g and 17.456 g. (3 significant figures)
(iz) Subtract 63.54 kg from 187.2 kg Radius of cylinder, r = 1.75 ern
(iii) 75.5x 125.2x 0.51. (3 significant figures)
Volume of cylinder
(iv) 2.13x 24.78 2.51 x 10- 4 x 1.81 x 107
(v) 0.4463 = rr? h =3.14x (1.75)2 x 4.54 ern3
458.2
= 43.657775 ern3 = 43.7 em 3.
IT
Solution.
[Rounded off upto 3 significant figures]
(i) 6.2 g + 4.33 g + 17.456 g =27.986 = 28.0 g.
[Rounded off to first decimal place] EXAMPLE ?3. The mass and radius of the earth are
(ii) 187.2 kg- 63.54 kg= 123.66 kg=123.7 kg. 5.975x 102 kg and6.37x 106 m respectively. Calculate the
average earth's density to correct significant figures. Take
[Rounded off to first decimal place]
rr =3.142.
(iil) 75.5 x 125.2 x 0.51 = 4820.826 = 4800.
H
Solution. Radius of earth, R =6.37x 106 m
[Rounded off upto two significant figures]
(3 significant figures)
(iv) 2.13x 24.78 0.115193 = 0.115.
458.2 Mass of earth, M = 5.975 x 1024 kg
[Ro~ded off to 3 significant figures] (4 significant figures)
O

7 Average density
(v) 2.51 x 10- 4 x 1.81 x 10 = 10.1795 x 103
0.4463 M M 5.975x 1024
= 10.2 x 103. V = i rrR3 i x 3.142x (6.37x 106 l
[Rounded off to 3 significant figures] 3 3
= 0.005517 x 106 kgrn-3
M

~UPLE 7Q. Each side of a cube is measured to be 7.203 m.


= 5.52 x 103 kg m -3.
What are the total surface area and the volume of the cube to
appropriate significant figures ? [NCERT] [Rounded off upto 3 significant figures]
Solution. Side of the cube = 7.203 m EXAMPLE 74. 5.74 g of a substance occupies 1.2 rnf.
Total surface area Express its density keeping significant figures in view.
[NCERT]
= 6 x side2 = 6 x (7.203)2 m
= 311.299254 m2 =3113 m2 Solution. Density
[Rounded off to 4 significant figures] Mass = 5.74 g = 4.783 gem" 3
Volume = side3 =(7.203)3 = 373.714754 m3 Volume 1.2 ern3
=373.7 m3. = 4.8 g cm-3•
[Rounded off to 4 significant figures] [Rounded of upto 2 significant figures]
UNITS AND MEASUREMENTS 2.39

2. Precision. It refers to the resolution or the limit to


:w: PROBLEMS FOR PRACTICE
which the quantity is measured. Precision is determined
1. State the number of significant figures in the following by the least count of the measuring instrument. The
measurements: smaller the least count, greater is the precision. If we repeat
(i) 0.009 m2 (ii) 5.049 Nm - 2 (iii) 0.1890 gem - 3 a particular measurement of a quantity a number of
(iv) 1.90 x 1011kg (v) 0.020800 m (vi) 5.308 J. times, then the precision refers to the closeness of the .
set of values so obtained.
[Ans. (i) 1 (ii) 4 (iii) 4 (iv) 3 (v) 5 (vi) 4]
We can illustrate the difference between accuracy
2. Subtract 2.5 x 104 from 3.9 x 105 with due regard to
and precision with the help of an example. Suppose
significant figures. (Ans. 3.7 x 105)
three students are asked to find the mass of a piece of
3. Round off the following numbers as indicated: metal whose mass is known to be 0.520 g. They obtain

R
(i) 15.654 upto 3 digits (ii) 15.75 upto 3 digits the data given in Table 2.9.
(iii) 15.654 upto 4 digits (iv) 15.65 upto 3 digits
(v) 142667upt05digits (vl)5.996x1OSupto3digits. l,able 2.9 Data to illustrate accuracy and precision
(vii) 0.7995 upto 1 digit Measure- Measure- Measure- Average
Student

SI
(viii) 2.5946 x 10- 4 upto 2 digits. ment 1 ment 2 ment 3 mass
[Ans. (i) 15.7 (ii) 15.8 (iii) 15.65 (iv) 15.6 0.50 g 0.51 g
(v) 142670 (vi) 6.00 x 105 (vii) 0.8 (viii) 2.6 x 10- 4] - A-- ~52_g_ ~lg I-

B _0.516 g 0.515 g__ 0.514 g 0.515 g


4. A jeweller puts a diamond in a box weighing 1.2 kg. ~ --"'-
Find the total weight of the box and diamond with C 0.521 g 0.520 g 0.520 g 0.520 g
due regard to significant figures, if the weight of
diamond is 5.42 g. (Ans. 1.2 kg) The data obtained by the student A are neither very
precise nor accurate, the individual values differ
5. The diameter of a circle is 1.06 m. Calculate the area
to an appropriate number of significant figures. widely and also the average value is not accurate. The
IT
Take 1t = 3.14. (Ans. 0.882 m 2) data for student B are more precise, as they vary
slightly from one another but the average mass is not
6. The radius of a solid sphere is measured as 11.24 em. accurate. The data for student C are both precise and
What is the surface area of the sphere to appro- accurate. The resolution for A is 0.01 g and that for B or
priate significant figures? [Delhi 021 Cis 0.001 g.
(Ans. 1588 ern 2)
2.30 ERRORS IN A MEASUREMENT
H
7. The mass of a body is 275.32 g and its volume is
36.41 em 3. Express its density upto appropriate 55. What do you mean by error in a measurement?
significant figures. (Ans. 7.562gem - 3) Briefly explain the different types of errors and their
8. 9.74 g of a substance occupies 1.2 cm '. Express its causes. How can these errors be minimised ?
density by keeping the significant figures in view. Error in a measurement, Every measurement is
O

[Central Schools 091 done with the help of some instrument. While making the
(Ans. 8.1 gcm-3) measurement, some uncertainty gets introduced in the
measurement. As a result, the measured value is
always somewhat different from the actual or true
2.29 ACCURACY AND PRECISION
value. The error in a measurement is equal to the difference
54. Distinguish between the terms precision and between the true value and the measured value of the
M

accuracy of a measurement. quantity.


Every measurement is limited by the reliability of the
Error = True value - Measured value
measuring instrument and skill of the person making the
measurement. If we repeat a particular measurement, An error gives an indication of the limits within
we usually do not get precisely the same result as each which the true value may lie. Every measurement has
result is subject to some experimental error. This imper- an error. Every calculated value which is based on
fection in measurement can be described in two ways: measured values has an error.

1. Accuracy. It refersto the closeness of a measurement


Different types of errors:
to the true value of the physical quantity. It indicates the 1. Constant errors. The errors which affect each
relative freedom from errors. As we reduce the errors, observation by the same amount are called constant errors.
the measurement becomes more accurate. Such errors are due to the faulty calibration of the scale
2.40 PHYSICS-XI

of the measuring instrument. Such errors can be elimi- 4. Least count error. This error is due to the limitation
nated by measuring the same physical quantity by a imposed by the least count of the measuring instrument. It is
number of different methods, apparatus or technique. an uncertainty associated with the resolution of the
2. Systematic errors. The errors which tend to occur in measuring instrument. The smallest division on the scale
one direction, either positive or negative, are called of the measuring instrument is called its least count. For
systematic errors. We can eliminate such errors once we example, a metre scale has a least count of 1mm, its
know the rule which governs them. These errors may readings are good only upto this value. The error in its
be of the following types : reading will be half of this value i.e., ± 0.5 mm or
±ODS em.
(I) Instrumental errors. These errors occur due to the
inbuilt defect of the measuring instrument. For example, S. Gross errors or mistakes. These errors are due to
either carelessness of the person or due to improper

R
wearing off the metre scale at one end, zero error in a
vernier callipers (zero of the vernier scale may not adjustment of the apparatus. No corrections can be
coincide with the zero of main scale), etc. This error can applied for gross errors.
be detected by measuring a physical quantity with two
different instruments of the same type or by measuring
the same physical quantity by two different methods. •.\. Least count errors are random errors but within a

SI
limited size ; they occur with both random and
(ii) Imperfections in' experimental technique. These
systematic errors.
errors are due to the limitations of the experimental
•.\. The accuracy of measurement is related to the
arrangement. For example, error due to radiation loss in
systematic errors but its precision is related to the
calorimetric experiments, error due to buoyancy of air
random errors, which include least count error also.
when we weigh a body in air. Such errors cannot be
eliminated altogether but necessary corrections can be 2.31 ABSOLUTE ERROR, RELATIVE ERROR
applied for them. AND PERCENTAGE ERROR
(iil) Personal errors. These errors arise due to
IT
56. How is random error eliminated? What do you
individual's bias, lack of proper setting of apparatus or
mean by (i) absolute error (ii) mean absolute error
individual's carelessness in taking observations without
(iii) relative error and (iv) percentage error?
observing proper precautions, etc. For example, when an
observer (by habit) holds his head towards right, while Elimination of error. Normal or Gaussian law of
reading a scale, he introduces some error due to random errors shows that the probability of occurrence
parallax. Such errors can be minimised if measurements of positive and negative errors is same, so random error
H
are repeated by different persons or removing the can be minimised by repeating measurements a large
personal bias as far as possible. number of times. Then the arithmetic mean of all measure-
(iv) Errors due to external causes. These errors arise ments can be taken as the true value of the measured quantity.
due to the change in external conditions like pressure, If al'~' a3, ... an be the n measured values of a
temperature, wind, etc. For example, the expansion of physical quantity, then its true value, a is given by the
O

a scale due to the increase in temperature. Such errors arithmetic mean,


can be easily detected and necessary corrections may al+a +a + ... +a...!!.n = -1 n a..
be made accordingly. These errors can also be
a or amenn 2 3
-!.._""£"-_>L- __ L
n n i=l I

minimised by controlling the external conditions


(i) Absolute error. The magnitude of the difference
during the experimentation.
between the true value of the quantity measured and the
3. Random' errors. The errors which occur irregularly
M

individual measured value is called absolute error.


and at random, in magnitude and direction, are called If we take arithmetic mean a as the true value, then
random errors. Such errors occur by chance and arise the absolute errors in the individual measured values
due to slight variation in the attentiveness of the will be
observer while taking the readings or because of slight
ll~=a-al lla2=a-a2
variations in the experimental conditions. For example,
if a person repeats the observation a number a times, M3 = a - a3 llan = a - an
he may get different readings every time. Random . (il) Mean or final absolute error. The arithmetic mean
errors have almost equal chances for both positive and of the positive magnitudes of all the absolute errors is called
negative errors. Hence the arithmetic mean of a large mean absolute error. It is given by
number of observations can be taken as the true value A - IIIal I+ IIIa2 I+ ... + IIIan I 1 n
of the measured quantity.
aa = =- L 11la.1
n n i= 1 I
UNITS AND MEASUREMENTS 2.41

Thus the final result of the measure of a physical The absolute errors in the different measurements
quantity can be expressed as a = a ± !J.a. are
Clearly, any measured value of a will be such that
!J.~ = 2.48 - 2.48 = 0.00 m
a-!J.a~a~a+!J.ti
~ = 2.48 - 2.46 = 0.02 m
(ii/) Relative error. The ratio of the mean absolute error !J.4 = 2.48 - 2.49 = -0.01 m
to the true value oj the measured quantity is calledrelative error.
!J.ti !J.~ = 2.48 - 2.50 = - 0.02 m
Relative error, oa = -=-
a Ms = 2.48 - 2.48 = 0.00 m
(iv) Percentage error. The relative error expressed in • :E I !J.LI
The absolute error =--

R
percent is called percentage error. . 5
!J.ti
Percentage error = -=- x 100% = O.~ + 0.02 + 0.01 + 0.02 + 0.00 = 0.05 = 0.01 m
a 5 5
:. Correct length = 2.48 ± 0.01 m
001 .

SI
• The unit of absolute error is same as that of the
Percentage error = -'-- x 100 = 0.40%.
quantity being measured. 2.48
• It is the relative error or the percentage error and not
EXAMPLE 76. In successive measurements, the readings of
the absolute error which truly indicates the accuracy
of a measurement.
the period of oscillation of a simple pendulum were found to
be 2.63 s, 2.56 s, 2.42 s, 2.71 sand 2.80 s in an experiment .
. Calculate (I) mean value of the period of oscillation
(ii) absolute error in each measurement (iti) mean absolute
error (iv) relative error (v) percentage error and (vi) express
the result in proper form. [NCERT]
IT
True value. If~, tlz" a3,. •.•. an are the-readings of an
Solution. (i) Mean period of oscillation,
experiment, then true value of the quantity is
given by the arithmetic mean, T = 2.63 + 2.56 + 2.42 + 2.71 + 2.80
_ ~ + a2 + a3 + ..... + an 1 n 5
a= = -n i=1.:E a..
n
I ..
I
= 13.12 ,,;2.624 s
2. Absolute error = True value.-' Measured value 5
H
or Mi = a -Qi = 2.62 s [Rounded off to 2nd decimal place]
3. Final absolute error (ii) Absolute errors in different measurements are
= Arithmetic mean of absolute errors !J.T1 = 2.62- 2.63 = - 0.01 s
- I ~ I + I MZ I +
!J.a=~~~--~~~~----~~
I~ I + .... + I !J.anI !J.T2 = 2.62 - 2.56 = 0.06 s
n !J.T3 = 2.62 - 2.42 = 0.20 s
O

!J.T4 = 2.62;- 2.71 = -0.09 s


!J.Ts= 2.62- 2.80 = -0.18 s
4. Relative error or fractional error ... EI!J.T.I
Final absolute error or' 6a = t.jf (III) Mean absolute error = __ ,-
n
True value Ii
M

!J.T= 0.01+ 0.06+ 0.20+ 0.09+ 0.18


5. Percentage error «
Sa X 100%
-=- 5
a
•. 0.54 0
=-=0.18s=0.l1s.
EXA..WPLE 75. The length of a rod as measured in an 5
experiment was found to be 1.48 m, 2.46 m, 2.49 m, 2.50 m, [Rounded off to 2nd decimal place]
and 2.48 m. Find the average length, the absolute error in (iv) Relative error,
ea~ observation and the percentage error.
Solution. Average length sr = ~T = 0.11 = 0.04198= 0.04.
T 2.62
2.48 + 2.46 + 2.49 +.2.50 + 2.48 12.41
- --m (v) Percentage error in T =0.04x 100 = 4%.
5 5
=2.482 m =2.48 m (vi) In terms of absolute error, T = (2.62 ± 0.11) s.
[Rounded off to 2 places of decimal point] In terms of percentage error, T = (2.62 ± 4%) s.
2.42 PHYSICS-XI

EXAMPLE 72. In an experiment, refractive index of glass 3. In an experiment to measure focal length of a
was observed to be 1.45, 1.56, 1.54, 1.44, 1.54 and 1.53. concave mirror, the value of focal length in
Calculate (i) Mean value of refraciiue index ; (ii) Mean successive observations turns out to be 17.3 em,
absolute error; (iii) Fractional error; (iv) Percentage error. 17.8 em, 18.3 em, 18.2 em, 17.9 em and 18.0 em.
Express the result in terms of absolute error and percentage Calculate the mean absolute error and percentage
error. [Chandigarh 03] error. Express the result in a proper way.
Solution. (/) Mean value of refractive index, [Ans. 0.25 em, 1.4%, (17.9 ± 0.25) em]

~ = 1.45+ 1.56+ 1.54+ 1.44+ 1.54+ 1.53 = 1.51.


2.32 COMBINATION OF ERRORS
6
. 57. How can we estimate the error in the (i) sum

R
(ii) Absolute errors in different measurements are
(ii) difference (iii) product (iv) quotient(v) power of
~f.ll = 1.51-1.45= 0.06
different measured quantities. Deduce the general rule
~f.l2 = 1.51-1.56=- 0.05 for evaluating the error in a combined calculation.
~f.l3 = 1.51 -1.54 = - 0.03 Propagation or combination of errors. An experiment
involves several measurements and the final result is

SI
~f.l4 = 1.51- 1.44 = 0.07
~f.l5 = 1.51- 1.54 = - 0.03 arrived at using different physical relations. The error
in the final result depends on the individual measure-
~f.l6 = 1.51-1.53=- 0.02 ments as well as the mathematical operations done to
Mean absolute error, get the final result. Following rules are used to evaluate
~~= II~f.lil maximum permissible error in a measurement.
n (i) Error in the sum of two quantities. Let ~ A and
0.06 + 0.05 + 0.03 + 0.07 + 0.03 + 0.02 ~Bbe the absolute errors in the two quantities A and B
6 respectively. Then
IT
= 0.26 /6= 0.0433 ""0.04. Measured value of A= A±~A
(iii) Fractional error, Measured value of B= B± ~ B
~~ 0.04 Consider the sum, Z =A + B
Of.l =- =- = 0.02649 = 0.03.
~ 1.51 The error ~ Z in Z is then given by
~~ Z ± ~Z = (A ± M) + (B ± ~ B)
H
(iv) Percentage error = -=- x 100 = 3%
f.l = (A + B) ± (M + ~ B)

In terms of absolute error, f.l = 1.51 ± 0.04 =Z ±(~A+ ~ B)


or ~Z=~A+ ~B
In terms of percentage error, f.l = 1.51 ± 3%.
Hence the rule. The maximum possible error in the
O

X PROBLEMS F"OR PRACTICE sum of two quantities is equal to the sum of the
absolute errors in the individual quantities.
1. The diameter of a wire as measured by a screw
gauge was found to be 0.026 em, 0.028 em, 0.029 em, (ii) Error in the difference of two quantities.
0.027 em, 0.024 em and 0.027 em. Calculate (i) mean Consider the difference,
M

value of the diameter (ii) mean absolute error (iii) Z=A-B


relative error (iv) percentage error. Also express the
The error ~ Z in Z is given by
result in terms of absolute error and percentage.
error. [Ans. (i) 0.027 em (ii) 0.001 em (iii) 0.037 Z ± ~Z =(A± ~A)-(B± ~ B)
(iv) 3.7%; (0.027 ± 0.001) em, (0.027 ± 3.7%) em] =(A-B)±~A+~B
2. The refractive index of water as measured by the =Z±~A+~B
Real depth For error ~ Z to be maximum, ~ A and ~ B must
relation f.l = was found to have the
Apparent depth have the same sign, therefore
values 1.29, 1.33, 1.34, 1.35, 1.32, 1.36, 1.30, 1.33. ~Z=~A+ ~B
Calculate (I) mean value of f.l(il) mean value of absolute
Hence the rule. The maximum error in the
error (iii) relative error (iv) percentage error. difference of two quantities is equal to the sum of the
[Ans. (i) 1.33 (ii) 0.02 (iii) 0.015 (iv) 1.5%] absolute errors in the individual quantities.
UNITS AND MEASUREMENTS 2.43

(iii) Errorin the product of two quantities. Consider (v) Error in the power of a quantity. Consider the
the product, nth power of A,
Z=AB Z= An
The error ~ Z in Z is given by

~r
The error ~ Z in Z is given by
Z±~Z=(A±~A)(B±~B)
= AB±A~ B± B~A± ~A. ~ B Z± ~Z=(A±M)" = An(l±
Dividing L.H.S. by Z and RH.s. by AB,we get
~Z ~B ~A ~A ~B
1±-=1±-±-±-.-

R
Z B A A B
[.: (1 + x)" z.1 + nx, when x «1]
As ~ A and ~ B are small quantities, their product
A B Dividing both sides by Z, we get
term can be neglected. The maximum fractional error
in Z is ~Z M
1±-=I±n-
Z A

SI
~Z ~A ~B
-=-+-
Z A B ~Z M
or -=n.-
Hence the rule. The maximum fractional error in
Z A
the product of two quantities is equal to the sum of the Hence the rule. The fractional error in the nthpower
fractional errors in the individual quantities. of a quantity is n times the fractional error in that
(iv) Error in the division or quotient. Consider the quantity.
quotient, AP [3'1
Z=A General rule. If Z = --, then maximum
cr
IT
B
fractional error in Z is given by
The error ~ Z in Z is given by
sc
A±M
A(I± M)
A
~Z
-=p-+q-+r-
Z
M
A
~B
B e
Z±~Z= =----- The percentage error in Z is given by
B± ~B ( AB)
B 1±~
H
~Z M ~B sc 100
-x 100
=p-x 100
+q-x 100+
r-x
Z A B e
= ; ( 1 ±. ~AA) ( 1 ± ~BBr 1
Derivation of general rule using differentiation.
We have
or Z ± ~ Z = Z (1 ± ~) ( 1 + ~:)
O

AP [3'1
Z=--
[.: (l+x)n z.1+nx,whenx«I] e'
Dividing both sides by Z, we get Taking logarithms, we get

1±~ZZ=(1±~)(1+ ~:) log Z = P log A + q log B - rlog e


M

On differentiating both sides, we get


M-~B M ~B
=1±-+-±-.-
A B A B dZ dA dB de
-=p-+q--r-
Z A B e
As the terms M and ~B are small, their product
A B Writing the above equation in terms of fractional
term can be neglected. The maximum fractional error errors,
in Z is given by
~Z M ~B sc
~Z M ~B ±-=±p-±q-±-
-=-+- Z A B e
Z A .B
The maximum permissible error in Z is given by
Hence the rule. The maximum fractional error in
the quotient of two quantities is equal to the sum of ~Z M
-=p-+q-+r-.
~B ~e
their individual fractional errors. Z A B e
2.44 PHYSICS-XI

EXAMPLE 80. The initial and final temperatures of a water


bath are (18 ± O.5)OCand (40± 0.3)°C. What is the rise in
." In all mathematical operations, the errors are of
temperature of the bath ?
additive nature.
.\. When a quantity appears with a power n greater than Solution. Rise in temperature
one in an expression, its error contribution to the final = Final temperature':' Initial temperature
result increases n times. So quantities with higher = (40 ± 0.3) - (18 ± 0.5) = (40 - 18) ± (0.3 + 0.5)
power in the expression should be measured with
maximum accuracy. = (22 ± 0.8)° C.
.\. If a quantity appears with a power less than one in an . V
ExAMPLE 81. The resistance R = - , where V = 100 ± 5 V
. expression, then its error contribution in the final I

R
result is reduced. and I = 10 ± 0.2 A. Find the percentage error in R. [NCERT]

/es based on Solution. The percentage error in V is 5% and in I it


is 2%.

FORMULAE USED The total percentage error in R is given by

SI
~R ~V M
1. If Z= A 'l- 13, then the maximum possible error in 2, . - x 100 = - x 100 + - x 100
R V I
/),2= M'l-'M
=5%+2%=7%.
2. If 2 = A - B,then the maximum possible error in 2,
~2= M+t!.B EXAMPLE 82. If the errors involved in the measurements of
3. If 2 = AB, then the maximum fractional error in Z
a side and mass of a cube are 3% and 4% respectively, what
~2 M ~B is the maximum permissible error in the density of the
-=-+- material?
2 A B
4. If 2 = A / B,then the maximum fractional error in Z Soluti
o ution. D'ensity, p = -M = 3
M
IT
~2 M M
V t
-=-+-. The percentage error in density is given by
2 A B ,.
5. If 2 :;:An,' then the maximum fractional error in 2,
~p AM AL
- x 100 = - x 100 + 3 x - x 100
A2 M P M L
-=n.-
Z A = 4% +3 x 3% =13%.
AP J3'I
-cr' then the maximum fractionalerror in 2,
H
6. If Z = ExAMPLE 83. The error in the measurement of radius of a
sphere is 2%. What would be the error in the volume of the
~Z M M ~C sphere?
-=p-+q-+r-
2 ABC
The percentage error in 2
Solution. Given: M x 100 = 2%
r
~2 M M
rex
~C
O

Z x 100 = P x 100 + q Ii x 100 + 100 Volume of sphere, V = 11tr3


A
Percent error in volume,
UNITS USED ~V M
-x 100 =3 -x 100 =3 x 2 =6%.
The maximum possible error has the same units as V r
the quantity itself but fractional error has no units.
ExA.UPLE 8+ The percentage errors in the measurement of
M

EXAUPLE 78. Two resistances Rl = 100 ± 3 nand mass apd speed are 2% and 3% respectively. How much will
~ = 200 ± 4 n are connected in series. What is their be the maximum error in the estimate of kinetic energy
equivalent resistance ? [NCERT] obtained by measuring mass and speed ? [NCERT]

SOLUTION. Equivalent resistance, Solution. Given:


R = Rl + ~ =(100 ± 3) + (200 ± 4) Am x 100 =2% and ~v x 100 =3%
m v
= (100 + 200) ± (3 + 4) = (300 ± 7)n.
Kinetic energy, K = -t mv2
Ex..4111PLE 79. Two different masses are determined as
(237 ± 0.5) g and(17.6± 0.3) g. What is thesum of theirmasses? Percent error in K.E.,
Solution. Sum of the masses t:.K Sm t:.v
- x 100 = - x 100 + 2 - x 100
= (23.7± 0.5)+ (17.6± 0.3) K m v
= (23.7 + 17.6):t (0.5 + 0.3) = (41.3 ± 0.8) g. =2%+2 x3%=8%.
UNITS AND MEASUREMENTS 2.45

EXAMPLE 85. The length, breadth! and height of a The percentage error in X is given by
rectangular block of wood were measured to be : llX lla llb Sc 1M .
100 x- =2-xlOO+ 3-xlOO+-xlOO+ --xlOO
1=12.13±0.02em; b=8.16±0.01em; X a b c 2d
h = 3.46 ± 0.01 em =2 x 4%+3x2%+3%+1x 1% =17.5%.
Determine the percentage error in the volume of the
EXAMPLE 89. For the estimation of Young's modulus:
block.
Y= 4Mg ~
Solution. Volume of block, V = Ibh 1t d2 . I
The percentage error in the volume is given by for the specimen of a wire, following observations were
recorded: L=2.890,M =3.00,d = 0.082,g =9.81, 1=0.087.
dV x 100 =(dl + db + M)x 100

R
V I b h Calculate the maximum percentage error in the value ofY and
mention which physical quantity causes maximum error.
0.02 0.01 0.01) 100
= ( 12.13 + 8.16 + 3.46 x Solution. Given Y = 4 ~g ~
I 7td
200 100 100 As 4 and 1t are constants and a standard value of' g'
=--+-+-

SI
1213 816 346 is taken, so the percentage error in Y will be
= 0.1649 + 0.1225 + 0.2890 = 0.58%. -dY x 100 = (dM
- + -si. + 2 -M + -dl) x 100
[Rounded off to 2 significant figures 1 Y M L d I
It is obvious from the given data that
EXAMPLE 86. The ~od of oscillation of a simple
pendulum is T = 21t " L/ g. Measured value of Lis 20.0 em =
dM= 0.01, Sl: 0.001, lld = 0.001, dl = 0.001
known to 1 mm accuracy and time Jor 100 oscillations of the .. dY x 100 =(0.01 + 0.001 + 2x 0.001 + 0.001}100
pendulum is Jound to be 90 s using a wrist watch of 1 s Y 3.00 2.890 0.082 0.087
resolution. What is the accuracy in the determination of g ?
= (O.0033+0.0003+0.0244+0.0115)x100
IT
[NCERT]

Solution. As T = 21t Ii = 0D395 x 100 = 3.95%.


Diameter of the wire causes maximum
value of Y.
error in the

or T 2 = 41t2 ~ or EXAMPLE 90. The specific resistance CJof a thin wire of


g radius r em, resistance R n and length L em is given by
H
.. dg x 100 = llL x 100 + 2 x dT x 100 ~R
CJ=--
g L T L
Now L = 20.0 em, dL = 1 mm = 0.1 em, T for 100 If r=0.26±0.02em,R=32±ln
oscillations =90 s, dT = 1 s and L = 78 ± om em,find the percentage error in CJ.
O

.. dg x 100 = 0.1 x 100+ 2x J..-x 100 Solution. The percentage error in specific resis-
g 20.0 90 tance CJis given by
= 0.5 + 2.22 = 2.72% = 3%. dCJx 100 =(2 M+ dR + dL)x 100
CJ r R L
EXAMPLE 87. Find the relative error in Z, if = [2X 0.02 + J..- + 0.01] x 100
Z = A4 B1/3 / CD3/2. [NCERT]
M

0.26 32 78
Solution. The relative error in Z is = [0.15+ 0.03+ 0.0001] x 100
dZ M
-=4x-+-x-+-+--.
1 dB zc 3 si: = 0.1801 x 100 =0.18 x 100 =18%.
Z A 3 B C 2D
EXAMPLE 91. If two resistors of resistances R1 = (4 ± 0.5) n
EXAMPLE 88. A physical quantity X is given by X . = a2~
and ~ = (16 ± 0.5) n are connected (i) in series and (ii) in
evd parallel; find the equivalent resistance in each case with
If the percentage errors of measurement in a,b,c and dare limits of percentage error.
4%, 2%, 3% and 1% respectively, then calculate the Solution. (i) The equivalent resistance of the series
percentage error in X. combination,
a2b3 R = R1 + ~ =(4±0.5)n + (16±0.5)n
Solution. Given: X = G
c..;d = (20 ± 1)n or 20 ± 5%
2.46 PHYSICS-XI

(ii) The equivalent resistance of parallel com- accuracy of 0.2 em. Find the percentage error in the
bination, area of the field. [Himachal07] (Ans.2.16%)
9. The relative density of a material is found by
R' = RIRz = 4x16 =3.20
Rl + Rz 4+16 weighing the body first in air and then in water. If
the weight in air is (10.0± 0.1) N and weight in
111 water is (5.0± 0.1) N, what would be the maximum .
From = R' = Rl + Rz ' we get percentage error in relative density? (Ans.5%)
10. The voltage across a lamp is (6.0± 0.1)volt and the
M' = Ml + ~Rz current passing through it is (4.0± 0.2)ampere. Find
R,2 . Ri Ri the power consumed. [Ans. (24.0 ± 1.6) watt]
11. The radius of a sphere is measured as (2.1± 0.5) em.

R
~R' =(R,2) ~Rl +(R,2) ~Rz Calculate its surface area with error limits.
Ri Ri
[Ans. (55.4 ± 26.4) em2]

= (3~2 r x 0.5 + (~.: r x 0.5


12. The radius of a sphere is 5.3± 0.1 em. Calculate the
percentage error in its volume. [Himachal06]

SI
(Ans. 5.7 %)
= 0.340
13. The measure of the diameter of a cylinder is
Here R' = (3.2 ± 0.34) 0 = 3.2 0 ± 10.625% (1.60± 0.01) em and its length is (5.0± 0.1) em.
Calculate the percentage error in its volume.
(Ans.3.25%)
X PROBLEMS FOR PRACTICE
14. The measured mass and volume of a body are
1. If A = (120 ± 0.1)em and B = (8.5± U5)em, find 2.00 g and 5.0 em3 resrectively. With possible errors
(i) A + Band (ii) A - B. [Delhi 04] of 0.01 g and 0.1 em , what would be the percent
[Ans. (i) 20.5± 0.6em (ii) 3.5± 0.6em] error in density? (Ans.2.5%)
IT
15. A body travels uniformly a distance of (13.8± 0.2)m
2. The temperaturesof two bodies measured by a thermo-
in a time (4.0± 0.3)s. Calculate its velocity with
meter are tl = 20°C ± O.soCand t2 = 50°C ± U5°e.
Calculate the temperature difference and the error error limits. What is percentage error in velocity ?
therein. [NCERT; Central Schools 12] [Ans. (3.5± 0.3) ms - I, 9%]
(Ans. 30°C ± l°q A1/3s4
16. Find the percentage error in 2, if 2 = .....2/3.
H
3. The lengths of two rods are 15.2 ± 0.2 em and . CLF
10.7± 0.2em. Find difference in lengths of the two [Delhi 08]
rods with the limits of error. [Himachal07] [Ans. % error in Z =(f~ + 4fl: + flee+ ~flg)100]
(Ans. 4.5 ± 0.4 em)
17. A physical quantity X is related to three obser-
4. The length and breadth of a rectangle are
vables a, b, c as X = Fa b2 /~. The errors of
O

(5.7 ± 0.1)em and. (3.4 ± 0.2)em. Calculate area of


measurement in a, band care 2%, 1% and 3%
the rectangle with error limits. [Chandigarh04]
respectively. What is the percentage error in the
(Ans. 19.4± 15 em 2) quantity X? (Ans.9%)
. A2 s3/2
5. If displacement of a body, s = (200 ± 5) meters and
18. A physical quantity Q is given by Q = c+ 4 r}/2
time taken by it t = (20 ± 0.2)seconds, then find the
M

percentage error in the calculation of velocity.


The percentage errors in A, B, C and D are 1%, 2%,
[Delhi 05] (Ans.3.5%)
4% and 2% respectively. Find the percentage error
6. If the length and time period of an oscillating in Q. [CentralSchools051 (Ans.22%)
pendulum have errors of 1% and 2% respectively,
19. Two resistors of resistances R,. = 100 ± 3 ohm and
what is the error in the estimate of g ?
~ = 200 ± 4 ohm are connected (a) in series, (b) in
[CentralSchools04] (Ans. 5%) parallel. Find the equivalent resistance of the
7. If ~ = (10.0± 0.1)em and 12 = (9.0 ± 0.1)em, find (a) series combination. (b) parallel combination. Use
their sum, difference and error in each. for (a) the relation R = R,. + ~ and for (b)
[Ans. (19.0± 0.2)em, 1%; (10 ± 0.2)em, 20%] ~ = -.l + -.l and M' = ~ + 11~
R' R,. ~ R,2 R,. 2 ~2 [NCERT]
8. The length and breadth of a field are 22.4 em and
15.8 em respectively and have been measured to an (Ans. 66.7± 180)
UNITS AND MEASUREMENTS 2.47

7td2h
=-c HINTS 13. Volume of a cylinder, V = n?h =--
4
2. t' = t2 - t1 = (50 e ± O.5°q - (20 e ± osq
0 0

t' = 30 e ± 1°e.
0 % Error, -IIV x 100 = [ 2 -lld + -llh] x 100
V d h
4. Here I = 5.7 ± 0.1em and b = 3.4 ± 0.2em
Area = lb = 5.7x3.4= 19.38em2 = 19.4 em2 = [2 x-0.01 + -0.1] x 100= [0.0125 + 0.02] x 100
1.60 5.0
M = ( III + llb ) lb = (0.1 + 0.2) x 19.4
= 0.0325 x 100 = 3.25%.
I b 5.7 3.4
= (0.1017 + 0.059) x 19.4 = 1.47 = 1.5 14. IIp = [ llm + IIV ] x 100
p m V
:. A = (19.4 ± 1.5) cm2•

R
5. As v = s l t = [0.01
-+- 0.1] x 100=2.5%."
2.00 5.0
llv lls M
:. - x 100 = - x 100 + - x 100
v s t 15. Given: s =(13.8± 0.2) m, t =(4.0± 0.3) s
= 25% + 1% = 3.5%. ·
Ve IOCIty,v = -s = -13.8 = 3 .45 ms - 1 = 3.4 ms" 1
4n21 t 4.0

SI
6. As g=-2-
T [Rounded off to first place of decimal]
:. llg x 100 = M x 100 + 2 llT = 1 + 2 x 2 = 5%. llv lls M 0.2 0.3 0.8+ 4.14
g I T -=-+-=-+-=----
v s t 13.8 4.0 13.8 x 4.0
7. Sum = (10.0 + 9.0) ± (0.1 + 0.1) = (19.0 ± 0.2) . 4.94
0.0895
13.8 x 4.0
Percentage error = 0.2 x 100 = 1%
19.0 llv = 0.0895 x v = 0.0895 x 3.45 = 0.3087 = ± 0.3
Difference = (10.0 - 9.0) ± (0.1 + 0.1) = (1.0 ± 0.2) em.
:. Velocity = (3.4 ± 0.3) ms-1 •
0.2
IT
Percentage error = - x 100 = 20%. llv
1.0 :. Error, - x 100 = 0.0895 x 100= 8.95% = 9%.
v
8. Area, A = I x b
18. llQ x 100 = 2 M x 100 + ~ M x 100
% Error = M x 100 = III x 100 + llb Q A 2 B
A I b + 4-
zc l11D
x100+ -- x 100
_--x0.2 100 +-x0.2 100 C 2 D
22.4 15.8
H
3 1
=2x1+-x2+4x4+-x2
= 0.89 + 1.27 = 2.16% 2 2
= 2 + 3 + 16 + 1 = 22%.
I ti
9. Reave density Weight in air
ensi = ~--:--"'-----
Loss of weight in water 19. (a) The equivalent resistance of series combination,
Loss of weight-in water R= R,.+ ~ =(100± 3)0 + (2oo± 4)0
O

= (10.0 - 5.0) ± (0.1 + 0.1) = 5.0 ± 0.2 N =300± 70.


Percentage error in relative density (b) The equivalent resistance of parallel combination,

= Percentage error in weight in air R' = R,.~ = 200 = 66.70


+ Percentage error in loss of weight in water R,.+~ 3
M

=( 0.1 + 0.2) x100=5%.


10.0 5.0

10. Power consumed, P = VI = 6.0 x 4.0 = 24.0 W


llP II V
-=-+-=-+-=-
ta 0.1 0.2 1
P V I 6.0 4.0 15
1 1
M= - x P= - x 24.0= 1.6 W
15 15
Power consumed = (24.0 ± 1.6) W.
11. Surface area of a sphere, A = 4nr2

.. M=2llr.
Hence R' = 66.7 ± 1.8 O.
A r
2.48 PHYSICS-XI

2.33 "f' RULES FOR DETERMINING THE Illustration. 12.9 g-7.06 g= 5.84 g=- 5.8 g, the corr-
UNCERTAINTY OF NUMBERS IN ected difference on being rounded off to first place of
ARITHMATIC OPERATIONS decimal. Both 2.9 g and 7.06 g have three significant
58. State and illustrate the rulesfor determining the figures each but their difference contains two
uncertainty of a number in arithmatic operations. significant figures.
Rules for determining the uncertainty of a 4. Thefractional errors of the value of a number specified
number in arithmatic operations: to n significant figures depends not only on n but also on the
original number.
1. The uncertainty in a measured value is equal to half
the least count of the measuring instrument. Illustration. Accuracy in the measurement of mass
1.02 g = ± 0.005 g
Illustration. Suppose the measured values of the

R
length and breadth of a rectangular sheet are 16.2 em Fractional error in 1.02 g
and 10.1 em respectively. Then its true length and = ± 0.005,)( 100=± 0.5%.
breadth may be expressed as 1.02 '
.' 1 =16.2 ±ix 0.1= 16.2 ±0.05 em Accuracy in the me.asurement of mass 9.89 g

SI
or 1 =16.2-em± 0.05 x 100= 16.2 cm± 0.3%. = ±O.o05 g
16.2
Fractional error in 9.89 g
and b = 10.1 ± i x 0.1 = 10.1 ± 0.05 em.
= ± 0.005 x 100=± 0.05%.
or b =10.1 em± 0.05 x 100= 10.1 em± 0.5%. 9.89
10.1
Both 1.02 8' and 9,89 8 conWn cm.e ligrtificant
2. When two or more measured values are multiplied, the
figures each. Both the numbers have same accuracy
percentage uncertainty of the final result is equal to the
but different fractional errors.
square root of the sum of the squares of the percentage
uncertainties in the original values. 5. The results' in the intermediate steps of a multi-step
IT
Illustration. The true area of the above rectangular calculation should be calculated to one extra significant
sheet can be expressed as figure in each measurement than the number of digits in the
least precise measurement. This avoids building up of errors
1]1 = 16.2x 10.1 em2 ± ~(0.3%l + (0.5%)2
in the process of rounding off the numbers.
= 163.62 em2 ± 0.6% Illustration. Reciprocal of 9.58 after being rounded
lb = 163.62 ± 163.62x 0.6 163.62 ± 1.0 em2. off to 3 significant figures = 0.104.
or
H
100 But the reciprocal of 0.104 calculated to 3 significant
3. If a set of experimental data is specified to n significant figures = 9.62.
figures, then the result obtained by combining this data will However, if we take 1/9.58 = 0.1044, then find the
als« be valid upto n significant figures. But sometimes the reciprocal to three significant figures, we get back the
number of significant figur.es get reduced in subtraction. original number 9.58.
O

Very Short Answer Conceptual Pro1letns

Problem 1. What is the necessity of selecting some Problem 3. In defining the standard of length, (the
M

units as fundamental units ? prototype metre), we have to specify the temperature at


which the measurement should be made. Are we
Solution. The number of physical units required to be
justified in calling length a fundamental quantity, if
measured is very large. If a separate unit is defined for
another physical quantity (temperature) has to be
each of them, then it will become very difficult to specified in choosing a standard ?
remember all of them as they will be quite unrelated to
Solution. Yes, the choice of length as a fundamental
each other. quantity is justified. The modem definition of metre in terms
Problem 2. How is SI a coherent system of units? of wavelength of light radiation is not affectedby temperature,
Solution.In 51, all derived units can be obtained by Problem 4. Do A and AU stand for the same unit of
multiplying and dividing the basic and supplementary length? [Himachal 03, 05C, 08]
units and no numerical factors are required to be Solution. No. 1 A (angstrom) = 10- 10 m
introduced. So 51 is a coherent system of units. 1 AU (astronomical unit) = 1496 x io" m.
UNITS AND MEASUREMENTS 2.49

Problem 5. Why is it convenient to express the Problem 13. If 'slap' times speed equals power, what
distances of stars in terms of light year rather than in will be the dimensional equation for 'slap' ?
metre or kilometre ? Solution. As slap x speed = power
Solution. One light year 2
Slap = Power = ML T - 3 = MLT - 2.
= 9.46 x Hy5 m = 9.46 x uP km. .. Speed LT-1
As the distances of stars are extra-ordinarily large, so
it is convenient to express them in light year rather than in Problem 14. Write the dimensions and SI unit of
metre or kilometre. linear momentum.
Problem 6. Comment on the statement: "To define a Solution. Linear momentum = Mass x velocity
physical quantity for which no method of measurement .. [Linear momentum] = [M][LTl] = [MLTl]

R
is given or known has no meaning." SI unit of linear momentum = kgms -1.
Solution. The given statement is not correct. A Problem 15. What is the basis of the principle of
physical quantity, if it is called so, must have a physical homogeneity of dimensions?
meaning. If it cannot be measured by any direct method,
Solution, The principle of homogeneity of dimensions'
these must be some indirect method for its measurement.
is based on the fact that only the physical quantities of the

SI
Entropy is one such physical quantity. .
same kind can be added, subtracted or compared.
Problem 7. Is the measure of an angle dependent
Problem 16. If x = a + bt + ct 2, where x is in metre
upon the unit of length ? .
and t in second; then what is the unit of c ?
Solution. 8 (radian) = ~ [Himachal 05C]
Radius
As an angle is the ratio of the length of an arc and the 2
Solution. [x] = [ct ] or [C]=[t~J
radius i.e., it is the ratio of two lengths, so the measure of
an angle doe not depend upon the unit of length. :. Unit of c = ms-2.
Problem 8. What is meant by angular diameter of the Problem 17. What are the dimensions of a and b in the
IT
moon ? What is its value? relation: F =a + bx, where F is force and x is distance?
Solution. The angle subtended by the two diametri-
Solution. [a]=[F]=MLT-2
cally opposite ends of the moon at a point on the earth is
2
called angular diameter of the moon. Its valueis about as. [b] = [~] = ML T = Mr2.-
Problem 9. For a given base line, which will show a L
greater parallax-a distant star or a nearby star? Problem 18. Name two physical quantities having
H
Solution. Parallactic angle, 8 = ------
base line b the dimensions [ML2T-2]. .
distance of star 5
Solution. Work and torque.
Thus for a given base line b, parallax of a star is Problem 19. Write three physical quantities having
inversely proportional to its distance S.Hence the nearby dimensions [ML-1T-2].
star will show a greater parallax.
O

Solution. Pressure, stress and Young's modulus of


Problem 10. Why is parallax method not useful for elasticity. '.
measuring the distances of stars more than 100 light
years away? Problem 20. If the units of force and length each are
doubled, then how many times would the unit of energy
Solution. For a star more than 100 light years away,
be affected ?
the parallax angle is so small that it cannot be measured
Solution. Energy = Work done = Force x length
M

accurately.
Problem 11. What is the difference between mN, Nm So when the units of force and length each are
and nm ? [Himachal 03] doubled, the unit of energy will increase four times.
Solution. 1 mN = 1 millinewton = 10- 3 newton (unit of Problem 21. The velocity v of a particle depends on
force). time t as : v = At 2 + Bt + C
1 Nm = 1 Newton metre (unit of work) where v is in m/s and t in second. What are the units of
1 nm = 1 nanometre = 10- 9 metre (unit of distance). A, B andC? ,I
Problem 12. Do all physical quantities have dimen- . . Unit of v ms-I -3
Solution. Unit of A = 2 = --2 - = ms
sions ? If no, name three physical quantities which are Unit of t s
dimensionless. Unit of v ms - 1 2
Solution. No, all physical quantities do not have Unit of B= = -- = ms"
Unitoft s
dimensions. The physical quantities like angle, strain and
relative density are dimensionless. Unit of C = Unit of v = ms-I.
2.50 PHYSICS-XI

Problem 22. Can a quantity have dimensions but still Problem 29. Which, of the following length
has no units ? measurements is (i) most precise and (ii) least precise?
Solution. No, a quantity having dimensions must Give reason (i) I = 5 em (it) I = 5.00 cm (iii) 5.000 em
have some units of its measurement. (iv) 5.000 em (v) 5.00000 em.
Problem 23. Can a quantity have different dimen-
Solution. (i) The last measurement is most precise,
sions in different systems of units ?
because it has been taken with an instrument whose least
Solution. No, a quantity has same dimensions in all
count is equal to 0.00001 ern.
system of units.
(ii) The first measurement is least precise because it
Problem 24. Can a quantity have units but still be
dimensionless ? has been taken with a device having least count equal to

R
1 cm.
Solution. Yes. For example, a plane angle has no
dimensions but has unit like radian for its measurement. Problem 30. Which of the following readings is the
Problem 25. Does the magnitude of a physical most accurate:
quantity depend on the system of units chosen ? (i) 5000 m (ii) 5 x 102 m (iii) 5 x 103 m ?
Solution. No. The magnitude of physical quantity

SI
Solution. (i) 5000 m is most accurate.
remains same in all systems of units.
Problem 26. Justify L + L = Land L - L = L. Problem 31. Which quantity in a given formula
should be measured most accurately?
Solution. When we add a length to another length, we
get length only so L + L = L is justified. Solution. The quantity which has maximum power
When we subtract a length from another length, again (say, n) in the formula should be measured more
we get length. This justifies L - L = L. accurately because an error in measurement is multiplied
Problem 27. Can there be a physical quantity that has n times in the final result.
no units and no dimensions? Problem 32. Which of the following measurements is
IT
Solution. Yes, strain is a physical quantity that has no more accurate and why?
units and no dimensions. (a) 0.0002 g (b) 20.0 g
Problem 28. Can an instrument be called precise with- Solution. The measurement
outbeing accurate?Can it be accuratewithout being precise?
(a) 0.0002 g is more accurate because it is correct
Solution. Yes, an instrument can be precise without upto fourth decimal place while measurement
being accurate but the measurements cannot be accurate
(b) 20.0 g is correct upto first decimal place.
H
without being precise.

Short Answer Conceptual Prohlents


O

Problem 1.Why do we treat length, mass and time as energy in calorie and electrical energy in watt-hour. So
basic or fundamental quantities in mechanics? mks system is not a rational system of units.
Solution. In mechanics, the quantities like length, mass Problem 3. Why it became necessary to redefine
and time are taken as fundamental quantities because: metre on atomic standard?
(i) these quantities represent basic scientific Solution. It became necessary to redefine metre on
M

atomic standard because the prototype metre offered the


notations,
following difficulties:
(ii) there is no other quantity simpler to them,
(i) It is difficult to preserve a metre bar.
(iii) length, mass and time cannot be expressed in
(ii) It is difficult to produce replicas of metre bar
terms of one another, and for their use in different countries.
(iv) all other physical quantities in mechanics can
(iii) The techniques used for producing replicas are
be expressed in terms of these quantities. not of very high accuracy.
Problem 2. SI is a rational system of units while mks Problem 4. What are the advantages of defining
system is not so. Justify. metre in terms of the wavelength of light radiation?
Solution. SI assigns only one unit to a particular Solution. The advantages of defining metre in terms
physical quantity so it is a rational system. For example, -, of wavelength of light radiation are as follows:
all types of energies are measured in joule in S1. But in (i) It can be easily reproduced anywhere and at
mks system, mechanical energy is measured in joule, heat any time.
UNITS AND MEASUREMENTS 2.51

(ii) It is invariant in time and space. Problem 8. What is the basic principle of alpha
(iii) It is unaffectedby environmental conditions particle scattering method for estimating the size of the
like temperature and pressure. nucleus? IPubjab 90]

(iv) It has an high accuracy of 1 part in 109• Solution. Both the a-particle and nucleus are posi-
tively charged. When an a-particle approaches a nucleus,
Problem 5. Give reasons why is platinum iridium
its kinetic energy gradually changes into potential energy'
alloy used in making prototype metre and kilogram.
-due to repulsive forces. At the distance of closest approach
Solution. The reasons for making standard kilogram '0' the entire kinetic energy changes into potential energy.
and metre from platinum-iridium alloy are as follows: This concept can be used to calculate '0 ' which gives the
(i) The alloy is least affected by temperature order of the size of the nucleus.
variations.

R
Problem 9. If the velocity of light is taken as the unit
(ii) It is non-corrosive and so does not wear out
of velocity and one year as the unit of time, what must
easily.
be the unit of length ? What is it called ?
(iii) It is quite hard.
Solution. Unit of length = unit of velocity x unit of time
(iv) It does not change with time.
Problem 6. Suggest a distance corresponding to each =3x10Bms-1 x 1year

SI
of the following order of length : =.3 x lOB ms-1 x 365 x 24 x 60 x 60s
(i) 107 m (ii) 104 m (iii) 102 m (iv) 10-3 m = 9.45 x 1015 ms-1
(v) 10- 6 m (vi) 10-14 m. =1 light year.

Solution. (i) 107 m = Radius of the earth Problem 10. What is common between bar and torr ?
Solution, Both bar and torr are the units of pressure.
(ii) 104 m = Height of Mount Everest
1 bar = 1 atmospheric pressure
(iii) 102 m = Length of Hockey Field
= 760 mm of Hg column
(iv) 10-_3 m = Thickness of a card-board
IT
1 torr > 1 mm of Hg column
(v) 10- 6 m = Mean free path of air molecule
1 bar = 760 torr.
(vI) 10-14 m = Size of atomic nucleus.
Problem 1. Suggest an indirect method for Problem 11. Distinguish between accuracy and '
measuring the height of a tree on a sunny day. _ precision.
Solution. Let AB be the height of a tree, as shown in Solution. By accuracy of a measurement we mean that
Fig. 2.15 and BCbe its shadow cast by the sun. the measured value of a physical quantity is as close to the
H
true value as possible. On the other hand, a measurement
is said to be precise, if same value of the quantity is
obtained in each of the various measurements carried out
,{ . \ \
with the given apparatus.
...- .1~".:"...,
I,
/ ;,~
I I " Problem 12. Which of the following measurements is"
O

I I
I I most accurate and which is most precise:
I ;
I I
(i) 4.00 mm (ii) 4.00 em
I '
II': ~ 1~ (iii) 4.00 m (iv) 40.00 m ?
C~~----------BL,--------~'B' Solution.
Fig. 2.15 Length Least Count Relative Error
M

(l) ~-,;
(lll) (lli/ I)
Let L ACB= 9.
(i) 4.00 mm 0.01 mm 2.5 x 10'- ~
Clearly, tan 9 = AB .-'~
BC 2.5 x 10- 3
(ii) 4.00 em 0.01 em
Take a rod A' B' and fix it at such a point that the tip of -- ------,---
(iii) 4.00 m 0.01 m 2.5 x 10- 3
its shadow coincides with the point C Then
A'B' (iv) 40.00 m 0.01 m 2.5 x 10- 4
tan 9=--.
B'C
AB A'B' Since the relative error in the measurement of 40.00 m
Hence -=-- is minimum, so this measurement is most accurate. The
BC B'C
measurement 4.00 mm is most precise because it is
:. Height of tree, AB = A' B' x BC measured with an instrument of minimum least count.
B'C
2.52 PHYSICS-XI

Problem 13. Two clocks are being tested against a you rule out on the basis of dimensional arguments (m
standard clock located in a national laboratory. At stands for the mass of the body) :
12:00:00 noon by the standard clock, the readings of the (a) K =m2v3 (b) K = (1/ 2)mv2
two clocks are : (c) K =ma (d) K = (3/16)mv2
Day Clock 1 aock 2 (e)K=(1/2)mv2+ma [NCERTJ

Monday 12:00:05 10:15:06 Solution. As SI unit of energy, J = kgm2s-2, so


Tuesday 12:01:15 10:14:59 [Energy] = [ML2r2]
(a) [m2v2] = [M2][Lr1f = [M2er2]
Wednesday 11:59:08 10:15:18
(b) [1/2 mv2] = [M] [Lr1f = [ML2r2]
Thursday 12:01:50 10:15:07
= [MJ[LT-2] = [MLr2]

R
. (c)[ma]
Friday 11:59:15 10:14:53
(d) [3/ 16 mv2] = [M] [Lrl f = [ML2r2]
Saturday 12:01:30 10:15:24
(e) The quantities (1/2) mv2 and ma have different
Sunday 12:01:19 10:15:11 dimensions and hence cannot be added.
If you are doing an experiment that requires Since the kinetic energy K has the dimensions of

SI
precision time interval measurements, which of the two [Mer 2], formulas (a), (c) and (e) are clearly ruled out.
clocks will you prefer? [NCERTJ Dimensional analysis cannot tell which of the two, (b)
Solution. The range of variation over the seven days or (d), is the correct formula. From the actual definition of
of observations is 162 s for clock 1, and 31 s for clock 2. The kinetic energy, only (b) is the correct formula for kinetic
average reading of clock 1 is much closer to the standard energy.
time than the average reading of clock 2. The important Problem. 17. Using the principle of homogeneity of
point is that a clock's zero error is not as significant for dimensions, find which of the following is correct :
precision work as its variation, because a 'zero-error' can 2 3 4 2 3
always be easily corrected. Hence clock 2 is to be (I) T2 = 41t2r2 (ii) T2 = 41t r (iii) T2 =~
IT
G GM
preferred to clock 1.
where T is time period, G is gravitational constant, Mis
Problem 14. For the determination of 'g' using a mass and r is radius of orbit. [Central Schools 05J
simple pendulum, measurements of I and T are 2
Solution. (I) T 2 = 41t r 2
required. Error in the measurement of which of these
will have larger effect on the value of 'g' thus obtained Dimensionally, MOLoT2= MOL2To
and why.? What is done to minimize this error? As LHS"*RHS, the formula is incorrect.
H
[Delhi 04J 2 3
(II..) T 2 41t
=--
r
Solution. Time period of a simple pendulum, G
3
T = 21t fI .. g = 41t~l Dimensionally, MOLoT2= L = M1LoT2
Vg T M-1L3r 2
O

Clearly, the error in the measurement of time period T As LHS"* RHS, the formula is incorrect.
has larger effect on the value of g than the error in the 41t r
2 3
measurement of length l. (iii) T2= __
GM
Reasons (i) T is very small. 3
(ii) In contrast to I, T 2 appears in the formula for g.
Dimensionally, MOLoT2= L = MOLoT2
M-1L3 r2M
M

To minimise the error, time period for a large number As LHS = RHS, the formula is correct.
of oscillations is measured.
Problem 18. The mean value of period of oscil-
Problem. 15. Magnitude of force F experienced by lation of a simple pendulum in an experiment is 2.825 s.
a certain object moving with speed v is given by The arithmatic mean of all the absolute errors is 0.11 s.
F = Kv 2, where K is a constant. Find the dimensions of K. Round off the period of simple pendulum to appro-
[Delhi 04J priate number of significant figures. Give reasons.

Solution. [K] = [~ = [ML~-~] = [ML-1]. [Central School 08J


[v] [Lr] Solution. The absolute error 0.11 s has only two
Problem. 16. The SI unit of energy is J = kgm2s-2, significant figures.
that of speed v is ms-1 and acceleration a is ms-2. Which .. Period of simple pendulum = 2.9 s
of the formulae for kinetic energy (K) given below can [Rounded off upto 2 significant figures]
UNITS AND MEASUREMENTS 2.53

HOTS

Problem 1. If nth division of main scale coincides with Equating the powers of Land T on both sides, we get:
'(n + J)th division of vernier scale, find the least count of the
p + q = I, - 2 P =- 1
vernier. Given one main scale division is equal to 'a' units.
· 1 1
[lIT Mains 03] On so Ivmg, p = 1.' q = 1..
Solution. (n + 1) divisions of vernier scale
Problem 4. A gas bubble, from an explosion under
= n divisions of main scale

R
water, oscillates with a period T proportional to pa db E C
,

1 V.S.D. =_n_ M.S.D. where p is the static pressure, d is the density of water and
n+I
E is the total energy of the explosion. Find the values of a, b
Least count = 1M.S.D.-I V.S.D. and c. [lIT 81 ; MNREC 90]

=1 M.S.D. __ n_ M.S.D. Solution. Let T = K pa db E -,

SI
n+I
where K = a dimensionless constant.
1
=--M.S.D. Putting the dimensions of various quantities,
n+I
T=[ML-1 r2t [ML-3t [ML2 r2r
=-- 1 x . = -- a unl it s.
a uruts or
n+I n+1
Equating the powers of M, L and T on both sides,
Problem 2. If; the velocity of light c, the constant of
we get
gravitation G and Planck's constant h be chosen as
fundamental units, find the dimensions of mass, length and a + b + c=O, - a -3b + 2c=O, -2a -2c=I
IT
time in terms of c, G and h. [lIT 92]
5 1 1
On solving, a = -- b =- C=-.
6' 2' 3
Solution. We have,
Problem 5. A small steel ball of radius r is allowed to
[c] = t.r I, [G] = M-1L3r2, [h] = ML2r1
fall under gravity through a column of a viscous liquid of
[h][c] ML2r 1.r.r 1 2 coefficient of viscosity 11. After some time the velocity of the
--= =M
[G] Ni1L3 r 2 body attains a constant value "r: The terminal velocity
H
depends upon (i) the weight of the ball mg (ii) the coefficient
Hence [M] = hl/2cl/2G-1/2. of viscosity 11 and (iii) the radius of the ball r. By the method
of dimensions, determine the relation expressing terminal
[h] ML2 r1
Again, --,,--=ML velocity. [Chandigarh 07]
[c] Lr1
Solution. Let "r = K (mgt11 br
O

C ,
'h h
[L] = c [M] = ch 1/2 c1/2 G- 1/2 where K = a dimensionless constant.
Putting the dimensions of various quantities,
= hl/2c-3/2Gl/2
Lr1= [MLr2]" [MC1rlt [Lt
1
As [c]=Lr or MOL1T-1 =Ma +bL" -b+cT-2"-b
M

[L] h 1/2 - 3/2 Gl/2


[T] = _ = c = h1l2c- S/2G1I2• Equating the powers of M, Land T on both sides,
C C we get
Problem 3. The velocity of a body which hasfallen freely a + b = 0, a - b + c = I, - 2 a - b = - 1
under gravity varies as gPhq ,where g is the accelerationdue to On solving, a = I, b = -I, c = -1
gravity at the place and h is the height through which the body
has fallen. Determine the values of p and q. [NCERT 83]
.. v = K (mg)I11-1r-l or v ex: mg.
T T 11 r
Solution. Let v = K gP v, Problem 6. Derive dimensionally the relation:
where K = a dimensionless constant.
5 = ut+i at2.
Putting the dimensions of various quantities, we get
Lr1 =[Lr2]p[L]Q Solution. Let 5 = Kux aY tZ ,

or L1r1 = LP+q T-2p where K = a dimensionless constant.


2.54 PHYSICS-XI

Putting dimensions of various quantities, Problem 7. The specific heats of a gas are measured as
L=[LT-1]x'[Lr2]y rrr Cp = (12.28 ± 0.2) units and Cv=(3.97 ± 0.3) units. Find the
or LITo =e+Yrx-2y+z
value of real gas constant R and percentage error in R.
Solution. Gas constant,
Equating the powers of M, L and T, we get
R = Cp -Cv
x+y=1
-x-2y+z=0 =(12.28± 0.2)- (3.97± 0.3)
These two equations cannot be solved for three = (8.31 ± 0.5) units
unknowns x, y and z. The problem is split into two parts.
% Error in R,

R
(i) Suppose the body has no acceleration. Then
S = K1 UX tZ AR
_L.>_ X 100 = [IlC p + IlC 1x 100
v
.. L = [L'I" 'r
[T]Z = LXrx +z R Cp - c,
Equating the powers of Land T, x = I, - x + z'=O
= 0.5 x 100 = 6.016%.
On solving, x = I, z = 1

SI
8.31
Hence S= K1 ut
Problem 8. The heat dissipated in a resistance can be
(ii) Suppose the body has no initial velocity. Then
determined from the relation:
_. S = IS.aYt Z

.. L = [Lr2]y [T]Z = IJIT-2y +.z H= P Rt cal


42
Equating the powers of Land T, y =1, -2y + z =0 If the maximum errors in the measurement of current,
On solving, y = I, Z =2 resistance and time are 2%, 1% and 1% respectively, what
S = IS. at2
IT
Hence would be the maximum error in the dissipated heat ?
. 12 Rt
(iii) Suppose the body has both acceleration and Solution. Given: H =--
initial velocity. Then 42
S= K1 ut + IS. at2 % Error, -sn x 100 = (2 -III + -IlR + -M) x 100
It is found that KI = 1 and IS. = 1/2. Therefore, H I R t
= 2 x 2 + 1 + 1 = 6%.
i at
H
2
S = ut + .

qide~e1> toNCERT Exercises •


O

- \...-

2.1. Fill in the blanks: (iv) Density = Relative density x density of water
(i) The volume of a cube of side 1em is equal to m3. at 4°C
(ii) The surface area of a solid cylinder of radius 2 em = 11.3 x 1gem - 3 = 11.3 gem-3
and height 10 em is equal to (mmf =11.3 x 103 kg m-3 =11300 kgm-3.
M

(iij) A vehicle moving with a speed of 18 kmlh covers 2.2. Fill in the blanks by suitable conversion of units:
...... min 1s. (i) 1kg ~s- 2 = ....g em2s- 2
(iv) The relative density of lead is 11.3. Its density is (il) 1m= lightyear (iii)3ms-2= kmh-2
...... gem - 3 or k-3
gm . 11
(iv) G = 6.67 x 10- Nm kg- 2 =
2 em s- 2g-1
3
Ans.
2 2 3 2
Ans. (I) 1 kg m s- = 1 (10 g) (10 em)2 s-2
(i) V = 13 =(lem)3 =(10-2 m)3 = 10-6 m-3.
=107gcm2s-2.
(ii) r = 2em = 20 mm, h = 10 em = 100 mm
S= 21tT(r + h) = 2 x3.14 x20(20 + 100) (ii) As 1 light year = 9.46 x 1(y.5 m
2'
=15072 mm .
.. 1m = 1 01 light year
(iii km 18x1000m 1
9.46 x l 5
III) v=18-= =5ms- . .".
h 60x60s = 1.053 x 10-16 light year > 10-16 light year.
UNITS AND MEASUREMENTS 2.55

(iii) 3 ms" 2 = 3(10- 3 km) (_1_


60 x60
hJ- 2
large, is meaningless. To correct it, we can say that the
mass of the earth is large in comparison to any object lying
on its surface.
= 3 x 10- 3 x 3600 x 3600 kmh -2 (ii) (a) The size of an atom is much smaller than even
= 3.888 x 104 kmh - 2 !::: 3.9 xl04 kmh -2. the sharp tip of a pin.
(b) A jet planet moves with a speed greater than
(iv) G = 6.67 x 10-11 Nm 2kg - 2
that of a superfast train.
= 6.67 x kg m s-2. m 2 kg-2
10-11
(c) The mass of jupiter is very large compared to
=. 6 67 x 10- 11ms3 -2k g -1 that of the earth.
= 6.67 x 10-11 (102em)3s-2(1000 gr1 (d) The air inside this room contains more number
= 6.67xlO-8cm3s-2g-1. of molecules than in one mole of air.

R
2.3. A calorie is a unit of heat energy and it equals about (e) This is a correct statement.
4.2 J, where 1 J = 1kg m2 s" 2. Suppose we employ a system of if) This is a correct statement.
units in which the unit of mass equals a kg, the unit of length
2.5. A new unit of length is chosen such that the speed of
equals p m, the unit of time is y s. Show that a calorie has a
light in vacuum is unity. What is the distance between the sun
magnitude 4.2 a-I p-2 y2 in terms of the new units.

SI
and the earth in terms of the new unit if light takes 8 min and
Ans. As lcalorie = 4.2 J, where IJ =.1 kg m2s-2 20 s to cover this distance?
Clearly, [Energy] = ML2T - 2 Ans. Speed of light
.. a=l, b=2, c=-2 = 1 new unit of length / s
Time = 8 min 20 s = 8 x 60 + 20 = 500 s
SI New System
Distance between the earth and the sun
1'J = 4.2 11z =?
= Speed of light x time = 1 x 500
M1 = lkg M2 = a kg
= 500 new units of length.
L1 =lm L2 =p m
IT
2.6. Which of the following is the most precise device for

r[~:r
Tl = Is T2 = Y s measuring length:

~:J (a) a vernier calliper with 20 divisions on the sliding

r[~:r[~:r
•. 11z = ~ [ a [~: scale,
(b) a screw gauge of pitch 1 mm and 100 divisions on
the circular scale
= 4.2 [ ~ ~~ 2
H
(c) an optical instrument that can measure length to
= p- 2 Y 2
4.2 a-I within a wavelength of visible light?
Hence 1 calorie =4.2 J =4.2a-1p-2l new units of energy. Ans. The device that has minimum least count will be
more precise for measuring length.
2.4. (i) Explain the statement clearly: To call a dimensional
quantity 'large' or 'small' is meaningless unthout specifying a (a) Least count of vernier callipers
O

standard for comparison. = 1 MSD - 1 VSD = 1 MSD - 19 MSD = ~ MSD


(ii) In view of this, reframe the following statements, 20 20
wherever necessary : 1 1
= - xl mm = - em = 0.005 em.
(a) Atoms are very small objects 20 200
. (b) A jet plane moves with great speed (b) Least count of screw gauge
M

(c) The mass of jupiter is very large Pitch


(d) The air inside this room contains a large number of No. of divisions on circular scale
molecules _ 1.0 mm __ 1_ _ 0 001
- - em-. em
(e) A proton is much more massive than an electron 100 1000
if) The speed of sound is much smaller than the speed (c) Least count of optical instrument
of light.
= Wavelength of visible (red) light
Ans. (i) The given statement is correct. Measurement
= 6000 A = 6000 x 10- 8 cm
is basically a comparison process. Without specifying a
standard of comparison, it is not possible to get an exact = 0.00006 cm.
idea about the magnitude of a dimensional quantity. For Hence the most precise device for measuring length is
example, the statement that the mass of the earth is very the given optical instrument.
2.56 PHYSICS-XI

2.7. A student measures the thickness of a human hair by 2.10. State the number of significant figures in thefollowing:
looking at it through a microscope of magnification 100. He (i) 0.007 m2 (ii) 2.64 x 1024 kg
makes 20 observations and finds that the average width of the
(iii) 0.2370 g em" 3 (iv) 6.320 J
hair in the field of view of the microscope is 3.5 mm. What is the
estimate on the thickness of hair? (v) 6.032 Nm" 2 (vi) 0.0006032 m2
Ans. Average thickness of hair as observed through Ans. (i) One: 7 (ii) Three: 2, 6, 4
microscope = 3.5 mm (iii) Four: 2, 3, 7, 0 (iv) Four: 6, 3, 2, 0
Magnification produced by the microscope = 100 (v) Four: 6, 0, 3, 2 (vi) Four 6, 0, 3, 2
Actual thickness of hair 2.11. The length, breadth and thickness of a rectangular
Observed thickness " 3.5 sheet of metal are 4.234 m, 1.005 m and 2.01 em, respectively.
= = - = 0.03Smm. Give the area and volume of the sheet to correct significant
Magnification 100

R
figures.
2.8. Answer the following:
Ans. Here I = 4.234 m, b = 1.005 rn ,
(a) You are given a thread and a metre scale. How will you
estimate the diameter of the thread? h = 2.01 em = 0.0201 m
(b) A screw gauge has a pitch of 1.0 mm and 200 divisions Area of sheet = 2 (lb +bh + hi)
on the circular scale. Do you think it is possible to increase = 2 (4.234 x 1.005 + 1.005 x 0.0201 + 0.0201 x 4.234) m 2

SI
the accuracy of the screw gauge arbitrarily by increasing = 2 (4.25517 + 0.0202005+ 0.085ID34) m2
the number ..yf divisions on the circular scale? = 2 x 4.3604739 m 2 = 8.7209478 m 2
(c) The mean diameter of a thin brass rod is;to be measured = 8.72 m2 [Rounded off upto 3 significant figures]
by vernier calipers. Why is a set of 100 melisurements of
Volume of the sheet
the diameter expected to yield a more reliable estimate
= lbh = 4.234 x 1.005 x 0.0201 m 3
than a set of 5 measurements only?
= 0.0855289 m 3 = 0.0855 m 3.
Ans. (a) The thread is wound on the metre scale so that
its turns are as close as possible. Thickness' l' of the thread [Rounded off upto three significant figures]
IT
coil is measured and the number of turns 'r( of the thread 2.12. The mass of a box measured by a grocer's balance is 2.3 kg.
coil is counted. Two gold pieces of masses 20.15 g and 20.17 g are added to the
.. Thickness of thread = .!.. cm. box. What is (a) the total mass of the box and (b) the difference in
n the masses of the pieces to correct significant figures?
(b) Least count of a screw gauge Ans. (a) Total mass of the box
Pitch = 2.3 kg + 0.02015 kg + 0.02017 kg
H
Number of divisions on circular scale kg = 2.3 kg.
= 2.34032

Theoretically, it appears that the least count can be The result has been rounded off to first place of
deci eased by increasing the number of divisions on the decimal because mass (2.3 kg) of box has digits upto this
circular scale. Practically, it may not be possible to take place of decimal.
the reading precisely due t910w resolution of human eye. (b) Difference in masses of 2 gold pieces
O

(c) Larger the number. of readings, closer is the = 20.17 g- 20.15 g = 0.02 g.
arithmetic mean to the true value and hence smaller the 2.13. A physical quantity P-is related to four observations:
random error. Hence result with a set of 100 measure- a, b, c and d as follows: P = a3b2j JCd
ments is more reliable than that with a set of 5 measurements. The percentage errors of measurement in a, b, c and dare
2.9. The photograph of a house occupies an area of 1.75 cm2 1%,3%,4% and 2% respectively. What is the percentage error
M

on a 35 mm slide. The slide is projected on to a screen and the in the quantity P ? If the value of P calculated using the above
area of the house-on the screen is 1.55 m2. What is the linear relation turns out to be 3.763, to what value should you round
magnification of the projector-screen arrangement? off the result? [Central Schools 12 ; Delhi 09]
Ans. Size of object = 1.75 cm 2 = 1.75 x 10-4m 2 a3 b2
Ans. Given: P = --
Size of image = 155 m2 JCd
Areal magnification The percentage error in the quantity P is given by
= Size of image = 1.55 = 8857 6P 6a 6b
100 x - = 3 x 100. - + 2 x 100. -
Size of object 1.75 x 10-4 P a b
1,+ 1x 100. 6c + 100 x M
Linear magnification 2 c d
= ~Areal magnification = .)8857 = 94.1. = 3 x I % + 2 x 3% + -t x 4% + 2% = 13%.
UNITS AND MEASUREMENTS 2.57

Since 13% = 0.13, so there are two significant figures in 2.16. The radius of a hydrogen atom is about 0.5 A. What is
the percentage error. Hence P should also be rounded off the total atomic volume in m3 of a mole of hydrogen atoms?
to 2 significant figures. Ans. Radius of a hydrogen atom,
.. P = 3.763 = 3.8. r = o.sA = 0.5 x 10-10 m
2.14. A book with many printing errors contains four
different formulae for the displacement y of a particle Volume of one atom = ~ 1tr3
. 3
undergoing a certain periodic motion :
No. of atoms in 1 mole = 6.023 x 1023
.) . 21tt
(I Y = a sin T ' (ii) Y = a sin vt,
Volume of 1 mole of H-atoms = N x ~ 1tr3
( ...) y= (a).T sm~t
III
= 6.023 x 1023 x~ x 3.14 x (0.5 x 10- 10)3
3

R
3
(.IV ) Y =.J2
(a) ( . 21tt 21tt )
sin T + cos T = 3.154 x 10-7 m3 =3x10-7 m3.

(a = maximum displacement of the particle, V = speed of the 2.17. One mole of an ideal gas at S. T.P. occupies 22.4 L.
particle, T = time-period of motion). Rule out the wrong What is the ratio of molar volume to the atomic volume of a mole
of hydrogen ? Why is this ratio so large ? Take the radius of

SI
formula on dimensional grounds:
Ans. Dimensions of LHS in all the cases (i) to (iv) = L hydrogen molecule to be 1A.

Dimensions of RHS in different cases are as follows Ans. Radius of a hydrogen molecule
. 21tt
(I') [ asm J = L .sm. T = L = 1A = 10-10 m
T T
Atomic volume of a mole of hydrogen
(Ang Ie -21tt. IS diimensio
. nl ess)
= Avogadro's no.
T
x Volume of a hydrogen molecule
This relation is dimensionally correct.
= 6.023 x 1023 x~ x 3.14 x (10- 10)3
IT
(ii) [a sin vt] = L sin (LT -1 T) = L sin (L) 3
(Angle is not dimensionless) = 25.2 x 10- 7 m 3

This relation is dimensionally wrong. Molar volume = 22.4 L = 22.4 x 10- 3 m 3


(1'1'1') [(a)T sm~
. t J =Tsm"L
L. T· . Molar volume 22.4 x 10- 3
------=-- -w 0.89 = 4 4
x 10 . z: 10 .
Atomic volume 25.2 x 10- 7
(Angle is not dimensionless)
H
This ratio is large because the actual size of the gas
This relation is dimensionally wrong. molecules is negligibly small in comparison with the

(iv) [h (Sin 2;t + cos 2;t)] = L [Sin i + cos iJ intermolecular separation.


2.18. Explain why on looking through the window of a fast
(Angle is dimensionless) moving train, the nearby trees and electric poles etc. appear to
O

This relation is dimensionally correct. run in direction opposite to that of motion of the train, while far
Hence formulae (ii) and (iii) are dimensionally wrong. off houses, hilltops, Moon, stars etc. appear stationary.
2.15. A famous relation in physics relates 'moving mass' m Ans. The line joining the object and the eye is called
to the 'rest mass' 1rIo of a particle in terms of its speed v and the the line of slight. The direction of the line of sight of the
speed of light c. (This relation first arose as a consequence of nearby objects like trees, poles etc. ; changes very rapidly
special relatively due to Albert Einstein). A boy recalls the due to fast motion of the train and accordingly they
M

relation almost correctly but forgets where to put the constant c. appear to be moving opposite to the direction of motion of
He writes: m= 1rIo
the train. But the line of sight of a distant object almost
(1_v2)1/2 does not change its direction due to its extremely large
distance from the eye. Hence the distant objects like
Guess where to put the missing c.
hilltops, moon, stars etc. appear stationary ..
Ans. Since quantities of similar nature can only be
2.19. A parsec is a convenient unit of lenph on the
added or subtracted, v2 cannot be subtracted from
astronomical scale. It is the distance of an object that will show a
dimensionless constant 1. It should be divided by c2 so as
parallax of 1" (second) of arc from opposite ends of a baseline
to make it dimensionless. Hence the corrected relation is
equal to the distance from the earth to the sun. How much is
m= 1rIo .
parsec in terms of metres ?
, ( 2
v J1/2 .
1- ? Ans. One parsec is the distance at which an arc of
length 1 AU makes an angle of 1 second of an arc.
2.58 PHYSICS-XI

I 2.22. Just as precise measurements are necessary in


As e(rad)=~=~ r=- •
Radius r o science, it is equally important to be able to make rough
Here I= 1AU = 1.496x 1011 m estiamtes of quantities using rudimentary ideas and
common observations. Think of ways by which you can
e = 1 s of arc = 1t rad
. 60 x 60 x 180 estimate the following (where an estimate is difficult to
obtain, try to get an upper bound on the quantity) :
= 4.85 x 10- 6 rad
(a) the total mass of rain-bearing clouds over India during
1496 x 1011 16
the Monsoon
.. 1parsec = r = . 6 = 3.08 x 10 m.
4.85 x 10 (b) the mass of an elephant
Order of magnitude of parsec = 16. (c) the wind speed during a storm

R
2.20. The nearest star (Alpha Ceniauri) to our solar system (d) the number of strands of hair on your head
is 4.29 light years away. How much is this distance in terms of (e) the number of air molecules in your classroom.
parsec? How much parallax would this star show when viewed
Ans. (a) The average rainfall during the Monsoon in
from two locations of the earth six months apart in its orbit
India is about 100 em or 1 m.
around the sun?
Ans. As 1 light year = 9.46 x 1015 m, Total surface area of India

SI
= 3.3 x 106 km2 = 3.3 x 106 x(103 m)2
1 parsec = 3.08 x 1016m
= 3.3 x 1012 m2.
:. Distance of Alpha Centauri from the earth, Volume of rain water,
S = 4.29 light years = 4.29 x 9.46 x 1015 m V = Ah=3.3x1d2 m2 xl m =3.3x1d2 m3
4.29 x 9.46 x 1015 Density of water, p = 10 kg m-3
3
----n"16,--- parsec = 1.32 parsec
3.08 x to- Hence total mass of rain-bearing clouds over India,
In an orbit around the sun, the distance between the m = Vp = 3.3 x 1012 x 103 = 3.3 x1015 kg.
IT
two locations of the earth six months apart, (b) To estimate the mass of an elephant, consider a
b = Diameter of the earth's orbit = 2 AU boat of base area A in a river. Let Xl be the depth of the
Parallax of the star, boat inside water. Now move the elephant into the boat.
Again measure the depth x2 of the boat inside water.
Arc b 2AU
e = --- =- = = 1.515 s of arc. Volume of water displaced by elephant
Radius S 132 parsec
V=A(x2-x1)
2.21. Precise measurements of physical quantities are a need
H
of science. For example, to ascertain the speed of an aircraft, one According to Archimedes' principle, mass of the
must have an accurate method to find its positions at closely elephant is
separated instants of time. This was the actual motivation m = Mass of water displaced by the elephant
behind the discovery of radar in World War II. Think of different = Vp = A(x2 -x1)p
examples in modern science where precise measurements of Mass of an elephant is about 103 kg.
O

length, time, mass etc. are needed. Also, wherever you can, give
(c) The wind speed during a storm can be measured by
a quantitative idea of the precision needed.
floating a gas filled balloon in air. When there is no wind
Ans. Some of the examples of modem science, where storm, suppose the balloon is at vertical height OA = 11,
precise measurements play an important' role, are as follows: from the ground. Due to the wind storm, suppose the
1. Electron microscope uses an electron beam of balloon moves to position Bin a small time interval l , as
M

wavelength 0.2A to study very minute objects like shown in Fig. 2.16.
viruses, microbes and the crystal structure of
solids.

2. The successful launching of artificial satellites has
been made possible only due to the precise
technique available for accurate measurement of
time-intervals. h

3. The precision with which the distances are o


measured in Michelson-Morley Interferometer o Ground
helped in discarding the idea of hypothetical
medium ether and in developing the Theory of
Relativity by Einstein. Fig. 2.16
UNITS AND MEASUREMENTS 2.59

If S is the angle of drift of the balloon, then from right Angular diameter of Jupiter,
angled !J. OAB,
AB x
S = 35.72" =( 35.72
60 x 60
)0 = 35.72 x .z, rad
3600 180
tan B= -=- [AB=x, say]
OA h
Diameter of Jupiter,
or x = htanS
D = S x S = 824.7 x 106 x 35.72 x ~
Hence the wind speed during the storm, 3600 180
x h tanS =1.428x10S km.
v=-=---.
t t 2.25. A man walking briskly in rain with speed v must slant
(d)First we count the strands of hair of 1 cm2 area of his umbrella forward making an angle S with the vertical. A

R
the head. Then by multiplying it by the total area of the student derives the fOllowing,reiation between Sand v :tan S = v
head, we canestimate the total number of stands of hair and checks that the reiation has a correct limit: as v ~ 0, S ~ 0,
on the head. Its order of magnitude may be as large as lOB. as expected. (We are assuming there is no strong wind and that
(e) The dimensions of a typical classroom are the rain falls vertically for a stationary man). Do you think this
8mx6mx4m relation can be correct ? If not, guess at the correct reiation.
Volume of the class room = 8 x6 x4 = 192m3

SI
Ans. Since trigonometric functions are dimensionless,
Now one mole of air molecules occupy it volume of " [tan S]= 1
22-4 litres or 224x10-3 m3. But [v] = Lr1
:. Number of molecules in 22.4 x 10- 3 m3 :. Dimensions of LHS * Dimensions of RHS
= 6.023 x 1023.
Hence the given relation is dimensionally wrong.
Number of molecules in the classroom
23 This relation can be corrected by dividing RHS by the
= 6.023 x 10 x 192 = 516 x 1026molecules speed 'u' of the rainfall. So the corrected relation is
22.4 x 10- 3
tanS=~.
IT
;.,102Bmolecules. u
2.23. The sun is a hot plasma (ionized matter) with its inner 2.26. It is claimed that two cesium clocks, if allowed to run
core at a temperature exceeding 107 K, and its outer surface at a for 100 years, free from any disturbance, may differ by only
temperature of about 6000 K. At these high temperatures no about 0.02 s. What does this imply for the accuracy of the
substance remains in a solid or liquid phase. In what range do standard cesium clock in measuring a time-interval of 1 s ?
you expect the mass density of the sun to be ? In the range of
Ans. Here !J.t = 0.02 s,
densities of solids and liquids or gases ? Check if your guess is
H
30
correct from thefollowing data: mass of the sun = 2.0 x 10 kg,· t = 100 years = 100 x 365.25 x 86,400 s
radius of the sun = 7.0 x lOB m: Fractional error
Ans. Mass of the sun, M = 2.0 x 1030 kg !J.t 0.02
0.63 x 10-11
Radius of the sun, R = 7.0 x lOB m t 100 x 365.25 x 86400
O

Volume of the sun, So there is an accuracy of 10-11 part in Is or Is irl1011 s.


V = .! nR3 = .! n x (7.0 x lOB)3 2.27. Estimate the average density of a sodium atom
3 3 assuming its radius to be about 25 A. Compare it with the
= 1.437 x 1027 m 3
density of sodium in crystalline phase: 970 kg m- 3. Are the two
Density of the sun, densities of the same order of magnitude? If so, why?
2.0x 1030
M

M
Ans. Radius of a sodium atom,
P= V = 1.437 x 1027 r = 2.5 A = 2.5 x 10- 10m
= 1391.8kgm-3 ;.,1.4 x103 kgm-3.
Volume of a sodium atom,
The density of the sun is in the range of the densities of
the solids and liquids but not gases. The high density is V = .! nr3 =.! n x (2.5 x 10-10)3
due to the invard gravitational attraction on the outer 3 3
layers due to the inward layers of the sun. . = 65.42 x 10- 30 kg
2.24. When the planet Jupiter is at a distance of 824.7 million Mass of a sodium atom
kilometres from the earth, its angular diameter is measured to be Mass number 23
35.72 s of arc. Calculate the diamet~ of Jupiter. = = g
Avogadro's number 6.02 x 1023
Ans. Distance of Jupiter from the earth,
= 3.82 x 10- 23 g = 3.82 x 10- 26 kg
S = 824.7 x 106 km
2.60 PHYSICS-XI

Average density of sodium atom Ans. Here t = 256 S, c=3 x 108 ms - 1

Mass 3.82 x 10- 26


Radius of the lunar orbit around the earth
= Volume = 65.42 x 10- 30
= Distance of the moon from the earth
= 0.58 x 103 kg m " 3 cxt 3 x 10 8
x 2.56 8
----- = 3.84 x 10 m.
Density of sodium in crystalline phase 2 2
= 970 kg m - 3 = 0.970 x 103 kg m- 3 2.30. A SONAR (Sound Navigation and Ranging) uses
Hence the average mass density of sodium atom and ultrasonic waves to detect and locate objects under water. In a
the density of crystalline sodium are of the same order of submarine equipped with a SONAR, the time delay between
magnitude (103). This is because sodium atoms in generation of a probe wave and the reception of its echo after
rejlection from an enemy submarine is found to be 77 s. What is

R
crystalline phase are closely packed.
the distance of the enemy submarine?
2.28. The unit of length convenient on the nuclear scale is a
(Speed of sound in water = 1450 ms-1).
fermi : 1 f = 10-15 m. Nuclear sizes obey roughly the following
empirical relation r = ro A1/3 ; where r is the radius of the Ans. Here t = 77 S, c = 1450 ms-1
nucleus, A its mass number and ro is a constant equal to about Distance of enemy submarine
1.2 f Show that the rule implies that nuclear mass density is

SI
c xt 1450 x 77
nearly constant for different nuclei. Estimate the mass density = 55825 m.
---
2 2
of sodium nucleus. Compare it with the average mass density of
2.31. The farthest objects (known as quasers) in our
a sodium atom obtained in Exercise 2.27.
universe are so distant that light emitted by them takes billion of
Ans. Radius of a nucleus, r = roAl/3 years to reach the earth. What is the distance in km of a quaser
from which light takes 3.0 billion years to reach us ?
Mass number
Mass of a nucleus Ans. Here t = 3.0 billion years
Avogadro's number
= 3.0 x 109 x 365.25 x 24 x 60 x 60 s
N uc Iear mass densi
ensity Mass of a nucleus
= -..,.-----,-----
IT
Volume of a nucleus Speed of light; c = 3 x 105 kms - 1
A A 3 Distance of quaser
or P= -----,,--
N .~ 1tr3 = ct = 3 x 105 x 3.0 x 109 x 365.25 x 24 x 60 x 60
A 3 22
= 2.84 x 10 km.
As p is independent of A, so nuclear mass density is 2.32. It is a well known fact that during a solar eclipse the
same for different nuclei. disc of the moon almost completely covers the disc of the sun.
H
For a kilomole, From this fact and from the information that sun's angular
NA = 6.02 x 1026, 10 = 1.2 f = 1.2 x 10-15 m distance a is measured to be 1920", determine the approximate
diameter of the moon. Given earth-moon distance = 3.8452 x 108 m
3
.. p = -----",------,......." Ans. During total solar eclipse, the disc of the moon
41t x 6.02 x 1026 x (1.2 x 10-15)3
completely covers the disc of the sun, so the angular
O

= 2.3 x 1017 kg m -3 diameters of both the sun and the moon must be equal.
Density of sodium nucleus should also be :. Angular diameter of the moon,
= 2.3x 1017 kg m-3 S = Angular diameter of the sun
From Exercise 2.27, density of sodium atom = 1920" = 1920 x 4.85 x 10- 6 rad

= 0.58 x 103 kg m-3 [',: 1" = 4.85 x 10- 6 rad]


M

Nuclear mass density 2.3 x 1017 n14 Earth-moon distance, S = 3.8452 x 108 m
.. = = 3.96 x hr .
Atomic mass density 0.58 x 103 Diameter of the moon,
D = S x S = 1920 x 4.85 x 10- 6 x 3.8452 x 108
Nuclear density is typically 1015 times atomic density
of matter. = 3.581 x 106 m = 3581 km.
2.33. A great physicist of this century (P.A.M. Dirac) loved
2.29. A laser is a source of very intense, monochromatic,
playing with numerical values of Fundamental constants of
and unidirectional beam of light. These properties of a laser light
nature. This led him to an interesting observation. Dirac found
can be exploited to measure long distances. The distance of the that from the basic constants of atomic physics (c e, mass of
Moon from the Earth has been. already determined very precisely electron, mass of proton) and the gravitational constant G, he
using a laser as a source of light. A laser light beamed at the moon could arrive at a number with the dimension of time. Further, it
takes 2.56 s to return after rejlection at the moon's surface. How was a very large number, its magnitude being close to the
much is the radius of the lunar orbit around the earth? present estimate on the age of the universe ('" 15 billion years).
UNITS AND MEASUREMENTS 2.61

From the table offundamental constants in this book, try to see Clearly, the quantity t has the dimensions of time.
if you too can construct this number (or any other interesting
number you can think oj). If its coincidence with the age of the Put G= 6.67xlO-11
Nm2 kg-2, c= 3x108 ms ",
universe were significant, what would this imply for the e = 1.6 x 10-19 C, me = 9.1 x 10- 31 kg, mp = 1.67 x 10- 27 kg
constancy of fundamental constant? and _1_ = 9 x 109 Nm 2C-2
41tEO'-
Ans. Using basic constants such as speed of light (c),
charge on electron (e), mass of electron (me)' mass of [9 x 109 x(1.6 x 10- 19)2]2
.. t - -----;;;-.;-~--____;"..---..--------c,,_'_;;_----___...
proton (m) and gravitational constant (G), we can - 1.67 x 10- 27 x(9.1 x 10- 31)2 x(3 x 108)3 x 6.67 x 10- 11
construct the quantity,
2 :: 2.13 x 1016 s
e2 1
(
t=--
41tEo J
x 23
mpme c G =
2.13x1d6
7 years = 0.667
9'
x 10 years.

R
3.156 xl0

Now --e ]
[ 41tEo
2
=[ --2r1 e
41tEo r
2
2] =[Fr]2 = 0.667 billion years.
This time is slightly less than the age of the universe
= [ML r2 .L2] = [ML3r2] (~ 15 billion years). It implies that the values of the
32
[ML r f basic constants of physics should change with time

SI
because the age of the universe increases with time.

Text Based Exercises


1 Mark Each
1. What is a physical unit? [Himachal01] 20. Express nanometre in terms of metre and angstrom
IT
2. Write the relationship between numerical value of a units.
quantity and the size of the unit. 21. Express the wavelength of yellow light (5893 A) in
3. Does the magnitude of a physical quantity change terms of nm.
with the change in the system of units. 22. How many nanometres are there in one metre?
4. Define length. [Himachal07C,08C]
5. Define a prototype metre. 23. How many angstrom are there in one metre?
H
6. Define standard metre in terms of the wavelength [Himachal07C,08C]
of light.
24. How many fermi are there in one metre?
7. What is the accuracy of the metre defined in terms [Himachal07C]
of wavelength of light radiation ?
25. How many light years are there in one metre?
8. Define standard metre in terms of velocity of light.
O

[Himachal06C]
9. Why was metre redefined in terms of velocity of
26. Write the full name of the technique used in locating
light in 1983 ?
(a) an under-water obstacle (b) position of an
10. Is light year a unit of time?
aeroplane in space. [Chandigarh07]
11. Define light year and express it in metres.
27. How much far away is the nearest star alpha
[Himachal Zk]
centuri from us ?
M

12. Define one astronomical unit. [Himachal06C]


28. What is the order of size of our galaxy?
13. Name the unit used to express the distances of stars.
29. Give the order of mean free path of an air molecule.
14. Define parsec and express it in metres.
30. What is the shortest distance measured indirectly
[Himachal02,05]
so far?
15. Express parsec in light years.
31. What is the estimated radius of the universe?
16. Arrange in ascending order-astronomical unit,
32. Suggest a distance corresponding to 107 m.
parsec and light year.
33. Name the unit used for measuring nuclear cross-
17. Name two commonly units used to express
wavelength of light. sections.
18. Express one micron in metre. [Delhi 10] 34. Name the device used for measuring directly the
lengths (i) from 10- 3 m to 102 m (ii) to an accuracy
19. Name the unit used to measure size of a nucleus
of 10- 4 m (iii) to an accuracy of 10- 5 m.
and express it in metre.
2.62 PHYSICS-XI

35. Name the device that can be used to measure the 68. Express 0.00000538in powers of 10.
number of wavelengths of light in a given distance. 69. Define dimensions of a physical quantity.
36. What does the word RADAR stand for? 70. State the principle of homogeneity of dimensions.
37. What does the word SONAR stand for? 71. Which of the following has the same dimensions as
38. What does the word LASER stand for? 'Planck's constant' : Torque, gravitational constant,
39. What is a laser? and angular momentum?
40. Express light year in metres. What is its order of 72. What are the dimensions of rate of flow?
magnitude? 73. Give names of a scalar quantity and a vector
41. What types of waves are used in a SONAR? quantity which have same dimensions.
74. What is the difference between the measurements

R
42. Define international standard of mass.
43. Define atomic mass unit. Express it in kg. 4.0 em and 4.000 ern ?
44. Are the inertial and gravitational masses of an 75. What importance is attached to the final zeros in a
object different from one another ? number without any decimal point?
" that common balance compares masses
45. It is said 76. If f = x3, then relative error in f would be how

SI
and spring balance measures weight. If a body is 2 many times the relative error in x ?
kg in common balance, it is 2 kg in spring balance, 77. A junior research fellow takes 100 careful readings
does it mean that mass is equal to weight? .• in an experiment. If he repeats the same experiment
46. How many times larger is kg than a mg ? by taking 500 readings, then by what factor will the
[Himachal03,06C] probable error be reduced ?
47. How many metric tons are there in a teragram ? 78. Give two examples of non-dimensional variables.
48. What is the order of mass of an electron ? 79. Write the names of three dimensional constants.
49. What is the order of mass of universe? 80. Write two examples of non-dimensional constants.
IT
50. What is the smallest mass measured indirectly so 81. If g is the acceleration due to gravity and A is
far? wavelength, then which physical quantity does
51. Define second in terms of casium-133 vibrations. .Jfi represent?
52. Human heart is an in-built clock. Comment. 82. Write the dimensional formula of wavelength and
53. Which is the most accurate atomic clock ? frequency of a wave. [Delhi 09]
54. Name the unit used for measuring very small time 83. If x = a + bt2, where x is in metres and t in seconds,
H
intervals. How is it related to second ? find the units of a and b. [Himachal08]
55. Which technique is used for measuring age of 84. If x = at + bt2, where x is in metres and t in hours
fossils, rocks, etc. (hr), what will be the units of 'a and 'b' ?
56. How many times is millisecond larger than a [CentralSchools08; Delhi 10]
microsecond ? -. 85. Give the no. of significant figures in 6.200x 10°sec.
O

57. Are there more microseconds in a second than the [CentralSchools,07]


number of seconds in a year ? 86. How can a systematic error be eliminated ?
58. What is the order of age of the earth? 87. How can we eliminate constant error?
59. Give the order of average life of a human being. 88. Name the S1 unit used to express the amount of
60. What is the shortest time interval measured substance. [CentralSchools04]
M

indirectly so far? 89. Write the' number of significant figures in each of


61. Name the S1 unit of (I) temperature and (il) electric the following measurements :
current. (a) 1.67x 10- 27 kg ; (b) 0.270em [Delhi 04]
62. Name and define the S1unit of luminous intensity. 90. Add 8.2 and 10.163 and round off the sum to two
63. Define one mole. significant figures. [Delhi 06]
64. What is leap year? 91. Give approximate ratio of 1 AU and 1 light year.
65. Name the year in which there is total solar eclipse. [Delhi 01]
66. The density of wood is 0.5em - 3. Write this value in 92. How many kilograms are there in 1 amu ?
S1units. ' [Himachal2K]
67. The radius of the sun is 696 000 000 m. Express it in 93. What do you understand by absolute error?
scientific notation (in powers of 10). [Himachal04]
UNITS AND MEASUREMENTS 2.63

Answers
1. The standard amount of a physical quantity chosen 20. 1 nanometre or 1 nm = 10- 9 m = 10 A.
to measure the physical quantity of the same kind is 21. As 10 A = 1nm
called a physical unit.
5893
2. The numerical value (n) of a physical quantity is :. 5893 A =- = 589.3 nm.
10
inversely proportional to the size of the unit (u) i.e.,
22. 1 nanometre (nm) = 10-9 m .. 1 m = 109 nm.
1
n oc - or nu = constant. 23. 1 angstrom (A) = 10-10 m .. 1 m = 1010 A
u
15
24. 1 fermi (f) = 10- m .. 1 m = 1015 f
3. No, the magnitude of a physical quantity does not

R
change with the change in the system of units. 25. 1 light year (ly) = 9.46 x 1015 m
4. Length may be defined as distance of separation :. 1 m = 1 nl5 ly = 1.057 x 10-16 ly.
between two points in space. 9.46xhr
5. In 1899, the general conference of weights and 26. (a) Sound navigation and ranging (SONAR)
measures defined metre as the distance between (b) Radio navigation' and ranging (RADAR)

SI
two transverse lines marked on a platinum-iridium 27. Alpha centauri is 4.26 light years away from us.
rod preserved at a constant temperature of 273.16K at 2B. The order of size of our galaxy is 1020m.
1 bar pressure in the International Bureau of Weights
and Measures at Sevres, near Paris in France. 29. The order of mean free path of an air molecule is
10-6 m.
6. Standard metre is defined as the distance which
contains 1650,763.73wavelengths of certain orange- 30. The shortest distance measured indirectly so far is
red radiation emitted by Kr-86 atom in its transition the radius of a proton (""10 -15 m).
between 211.0and 2ds levels. 31. The estimated radius of the universe is 1025m.
7. Standard metre defined in terms of the wavelength 32. Radius of the earth is the order of 107 m,
IT
of light radiation has an accuracy of 1 part in 109• 33. Barn is used to measure nuclear cross-section. •
B. The velocity of light in vacuum, c = 299, 792, 1barn = 10- 28m2.
458 ms - 1. So one metre is defined as the length of
34. (i) Metre-scale (il) Vernier callipers (iii) Screw gange
the path travelled by light in vacuum in
or spherometer.
1/299,792,458 of a second.
35. Optical interferometer.
9. Because of the tremendous accuracy (1 part in 1013}
H
achieved in the measurement of time, metre has 36. The word RADAR stands .for radio detection and
been redefined in terms of velocity of light since 1983. ranging.
10. No, light year is a unit of distance. 37. The word SONAR stands for sound navigation and
ranging.
11. One light year is defined as the distance travelled
by light in one year. 3B. The word LASER stands for light amplification by
O

1 light year = 9.46 x 1015m stimulated emission of radiation.


39. A laser a source of very intense, highly mono-
12. One astronomical unit (AU) is the mean distance of chromatic and highly directional beam of light.
the earth- from the sun.
40. 1 light year = 9.46 x 1015 m = 0.946 x 1016 m.
1AU = 1.496 x 1011 m.
:. Order of magnitude of light year = 1016 m.
13. Parallactic second (parsec).
M

41. Ultrasonic waves.


14. One parsec is defined as the distance at which an
arc of length one astronomical unit subtends an 42. The international standard of mass is kilogram (kg).
angle of one second of an arc. One kilogram is the mass of a platinum iridium
1parsec = 3.08 x 1016 m. (90%Pt and 10% Ir) cylinder of diameter equal to its
height preserved in the International Bureau of
15. 1 parsec = 3.26 light years.
Weight and Measures at Sevres.
16. 1 AU < 1 light year < 1 parsec.
17. nm and A. 43. One atomic mass unit is the mass of ~th of the
12
lB. One micron is one millionth part of a metre. mass of an atom of carbon-12 isotope.
:,!:~, 1 micron = 1!l = 10- 6 m. 1 amu = 1.66 x 10- 27kg
19. The unit used to measure nuclear size is fermi. 44. No, the inertial and gravitational masses of an
1 fermi = 10-15 m. object are equivalent.
2.64 PHYSICS-XI

45. No, mass and weight are two different physical 68. 0.00000538= 5.38 x 10- 6.
quantities. If mass is 2 kg in common balance, it is 2 69. The dimensions of a physical quantity are the
kilogram force (kg f) in spring balance. powers to which the fundamental quantities must
46. 1 kg = 106 mg. be raised to represent that quantity completely.
47. 1 Teragram = 1012g = 109 kg = 106 metric tons. 70. The principle of homogeneity of dimensions states
48. Mass of an electron z: 10- 30 kg that a physical equation will be dimensionally
correct if the dimensions of all the terms occurring
49. Mass of universe =. 1055 kg
on both sides of the equation are the same.
50. The smallest mass measured so far is the mass of an
electron (,., 10- 30 kg). 71. Angular momentum.
3

R
51. One second is defined as the duration of 9192,631, 72. [Rate of flow] = [Volume] = [L ]= [L3r1].
770 vibrations corresponding to the transition [Time] [T].
between two hyperfine levels of cesium-133 atom 73. Speed is a scalar quantity and velocity is a vector
in the ground state. quantity. Both have the same dimensions of [LT~1].
52. Yes, because human heart beats at a regular rate. 74. (i) 4.0 cm has two significant figures while

SI
53. Cesium atomic clock. 4.000 cm has four significant figures.
54. Shake is the unit used for measuring very small (ii) 4.0 em is correct upto first decimal place while
time-intervals. 4.000 em is correct upto third place of decimal.
1 shake = 10- 8 second 75. All such zeros are not significant. For example
55. Radioactive dating. 86,400 has three significant figures.
56. 1 millisecond = 103 microsecond. 3 N /J.x
76. When f =x , - = 3 x-
57. No. As 1 microsecond = 10- 6 s, therefore f x
1 second = 106 microseconds :. Relative error in f
IT
Also, no. of seconds in a year = Three times the relative error in x.
= 365 x 24 x 60 x 60 = 31.536 x 106. 77. The probable error will be reduced by a factor of 1/5.
Clearly, the number of microseconds in a second is 78. Specific gravity and strain.
smaller than the number of seconds in a year. 79. Gravitational constant, Planck's constant and
58. Age of earth = 1017second. Boltzmann's constant.
59. Average life of a human being e 100years = 109 s.
H
80. "It, e, all trigonometric functions.
60. The shortest time interval measured so far is the 81. .Jfi =~L. Lr2 =Lr1
life-span of most unstable nucleus (,., 10- 24s).
61. (i) The SI unit of temperature is kelvin (K). .. .Jfi represents velocity.
82. [A] = [MoL1To]and [v] = [MoLor1]
(it) The SI unit of electric current is ampere (A).
83. Unit of a = unit of x = metre
O

62. The SI unit otluminous intensity is candela (cd).


One candela is defined as the luminous intensity, in Unit of b = unit of (x / t2) = m / s2.
a given direction, of a source that emits a 84. Unit of a = unit of (x / t) = m / h
mochromatic radiation of frequency 540 x 1012 Unit of b = unit of (x / t2) = m / h2.
hertz and has a radiant intensity in that direction of
1/683 watt per steradian. 85. Four: 6, 2, 0, 0
M

63. One mole is that amount of a substance which 86. The errors which tend to occur in one direction,
contains as many elementary entities as there are -positive or negative, are called systematic errors.
atoms iri 12 g of carbon-12 isotope. These entities Such an error can be eliminated by detecting the
may be atoms, molecules, ions, etc. source of error and the rule governing this error.
64.: Leap year is the year which is divisible by four. The 87. The errors which affect each observation by the
February month of a leap year has 29 days. same amount are called constant errors. Such an
65. Tropical year. error can be eliminated measuring the same
. g 0.5x10-3 kg physical quantity by number of different methods,
66. Density = 0.5--3 = 6 3 500 kg m - 3. apparatus or technique.
em 10 m
88. Mole
67. Radius of the sun = 696000 000 m
89. (a) Three : 1, 6, 7
= 6.96 x108 m. (b) Three: 2, 7, 0
.' I
UNITS AND MEASUREMENTS 2.65

90. 8.2 1 AU 1.496 x l(jll m -5


91. = ..15 ~ 10
+ 10.163 1 light year 9.46 x Iv- m
Sum = 18.363 92. 1 amu = 1.66 x 10- 27 kg.
Corrected sum = 18
93. Refer answer to Q.56 on page 2.40.
[Rounded off upto 2 significant figures]

2 or 3 Marks Each

1. Define the following: (i) Light year (ii) Parsecond 16. State the errors other than random and instru-

R
(iii) Astronomical unit. [Himachal 05] mental errors.
2. Explain the triangulation method to measure the 17. Show that the maximum error in the sum of the two
height of an inaccessible object. [Himachal 07} quantities is equal to the sum of the absolute errors
3. Distinguish between mass and weight. in the two individual quantities.
[Himachal 01, 04] 18. Show that the absolute error in the difference of two

SI
4. Distinguish between inertial mass, gravitational quantities is equal to the sum of the absolute errors
mass and weight of a body. in the individual quantities.
5. Which technique is used for measuring large time 19. Show that the maximum fractional error in the
intervals? product of two quantities is equal to the sum of the
b. Define dimensional formula. Give uses of dimen- fractional errors in the individual quantities.
sional analysis. Write down the limitations of 20. Show that the maximum error in the quotient of
dimensional analysis. [Himachal OS} two quantities is equal to the sum of their
7. In What way is the knowledge of the dimensions of individual fractional errors.
IT
a physical quantity useful? 21. Show that the fractional error in the nth power of a
8. How is a dimensional formula different from a quantity is equal to n times the fractional error in
differential equation? the quantity itself. State the general rule for
evaluating the error in a combined calculation.
9. Distinguish between the dimensions and unit of a
physical quantity. 22. What is meant by the term "measurement of a
10. Differentiate between dimensional and non- physical quantity ? How is the result uf
H
measurement of a physical quantity expressed ?
dimensional variables.
[Delhi 03C]
11. Distinguish between dimensional and non-
23. State the principle of homogeneity of dimensions.
dimensional constants.
Test the dimensional homogeneity of the following
12. Name any three physical quantities having the
equation:
same dimensions and also give their dimensions.
t gt
O

2
[Central Schools 07] h = hv + vot + lDelhi 03]
24. Define least count error. What is the value of least
13. What are the dimensional formulae of the following:
count error associated with the scale in your
(i) pressure (ii) power (iii) density (iv) angle?
geometry box? [Central Schools 03]
[Himachal 01}
25. Describe the parallax method to find the distance of
14. Write the dimensional formulae of the following
M

an inferior planet from earth. [Delhi 06}


physical quantities : (i) work (ii) angular velocity
(iii) pressure (iv) Planck's constant. [Himachal 05] 26. What is meant by RADAR and SONAR? How are
long distances measured using these techniques ?
15. Mention the various sources of occurrence of
[Central Schools 04]
errors, while taking measurements.

Answers
1. Refer answer to Q.13 on page 2.5. 4. Inertial mass of a body is the measure of its inertia
2. Refer answer to Q.17 on page 2.8. in translatory motion. It is equal to the force
3. Refer answer to Q.32 on page 2.16. required to produce unit acceleration in the body.
2.66 PHYSICS-XI

The mass of a body which determines the 12. Work, energy and torque. Each quantity has the
gravitational pull acting upon it due tu the earth 1S dimensions [ML2T-2].
called its gravitational mass. It is equal to force
13. (i) [ML-1r2] (ii) [ML2r3]
experienced by the body in a gravitational field of
unit intensity. (iii) [ML-3TO] (iv) dimensionless.

Weight of a body is the force with which a body is 14. (i) [ML2r2] (ii) [MOLorl]
attracted towards the centre of the ·earth. (iii) [ML-1r2] (iv) [ML2rl]
5. The technique of radioactive dating is used to 15. While taking measurements, the sources of
measure long time intervals by finding the ratio of occurrence of errors may be as follows :
the, number of radioactive atoms that have
(i) Instrumental errors due to faulty graduations,
undergone decay to the number of atoms left

R
zero error, etc.
undecayed.
(ii) Personal errors due to personal peculiarities of
6. Refer answer to Q. 44 on page 2.25 and Q.49 on
the experimenter.
page 2.36.
(iii) Error due to imperfection in experimental arran-
7. The dimensions of a physical quantity represent the gement, such as loss of heat due to radiation in

SI
powers of the fundamental units, on which the given calorimetry.
physical quantity depends. So we can express its units
(iv) Errors due to external causes such as expansion
in terms of the fundamental units. Suppose the
of scale due to rise in temperature.
dimensions of a physical quantity are 1 in mass, 1 in
length and - 2 in time, then its'SI unit is kgms-:!. 16. The errors other than random and instrumental
8. Dimensional formula is an expression which shows errors are (i) absolute error (ii) relative error
how and. which of the fundamental units of mass, (iii) percentage error and (iv) gross error. The gross
length and time occur in the derived unit of a error may be due to incorrect reading of the
physical quantity. The equation obtained by instrument or due to incorrect recording of the
IT
equating a physical quantity to its dimensional reading.
formuia is called its dimensional equation. 17. Refer answer to Q. 57 on page 2.42.
9. Dimensions of a physical quantity are the powers to 18. Refer answer to Q. 57 on page 2.42.
which the fundamental units must be raised to 19. Refer answer to Q. 57 on page 2.42.
represent the unit of the given physical quantity.
20. Refer answer to Q. 57 on page 2.42.
The unit is compact mathematical expression
involving fundamental units. The unit may also be 21. Refer answer to Q. 57 on page 2.42.
H
given some name. e.g., [F] = [MLr2] and unit of 22. Refer answer to Q. 3 on page 2.2.
force = kg ms - 2 or newton. 23. For principle of homogeneity of dimensions, refer
10. The quantities which have dimensions but do not answer to Very Short Answer Q. 70 on page 2.64.
possess a constant value are called dimensional
[h] = L, [ho] = L
variables e.g., velocity, force etc. On the other hand,
O

1
the quantities which have neither dimensions nor lUu t] = LT' . T =L
they have a constant value are called non-dimen-
sional variables e.g., relative density, strain, etc. [ .! gt 2 ] = L T" 2 . T2 =L
2
11. The quantities which have dimensions as well as
As all the terms have the same dimensions, so the
a constant value are called dimensional constants
given equation is dimensionally homogeneous.
M

e.g., Planck's constant, Boltzmann's constant. On


the other hand, the quantities which have no 24. Refer answer to Q. 55 on page 2.39.
dimensions but a constant value are called 25. Refer answer tu Q. 22 on page 2.10.
non-dimensional constants t!.g., IT, sin e, cos e etc. 26. Refer answer to Q. 2i and Q. 28 on page 2.11.
Competition
Section

Units and Measurements

R
GLIMPSES

SI
1. Physical quantities. All those quantities which (i) The CGS or the metric system. In this system
can be measured directly or indirectly and in terms the fundamental units of length, mass and time
of which the laws of physics can be expressed are are centimetre, gram and second respectively.
called physical quantities. (ii) The FPS or the British system. In this system
2. (i) Physical unit. The standard amount of a phy- the fundamental units of length, mass and
sical quantity chosen to measure the physical time are foot, pound and second respectively.
IT
quantity of the same kind is calleda physical unit. (iii) The MKS system. In this system the funda-
Measure of a physical quantity = Numerical mental units of length, mass and time are metre,
value of the quantity x size of the unit = nu kilogram and second respectively.
(ji) Relationship between the numerical value (iv) The S1. SI is the abbreviation for Systeme
and the size of the unit. The numerical value Internationale d'Unites, which is the French
(n) of a physical quantity is inversely equivalent for international system of units. In
H
proijOrtional to the size of the unit. this system the fundamental units of length,
1 mass, time, electric current, temperature,
noc- or nu = constant or 1\~ = 1'I:!lI:2.
t.
u luminous intensity and amount of substance
3. fundamental quantities. The physical quantities are metre, kilogram, second, ampere, kelvin,
which can be treated as independent of other candela and mole, respectively.
O

physical quantities and pre not usually defined in 8. The seven basic SI units.
terms of other physical quantities are called funda- (i) Metre (m). One metre is defined as the length
metittil quantities. The seven fundamental quantities of the path travelled by light in vacuum during a
are mass, length, time; electric current, temperature, time interval of 1/299,792,458 of a second (1983).
luminous inte,psity and amount of substance.
(ii) Kilogram (kg). It is the mass of a platinum-
4. Derived quantities. The physical quantities whose iridium cylinder (90% Pt and 10% Ir) whose
M

defining operatiorts are based on other physical height is equal to its diameter (each = 3.9 cm)
quantities are called derived quantities. preserved at International Bureau of Weights
5. Fundamental, units. The physical units which can and Measures at Sevres (1889).
neither be derived from one another, nor they can (iii) Second (5). One second is defined aslhe duration
be further resolved into more simpler units are
of 9,192,631,770periods of the radiation corres-
called fundamental units. The units Qf length, mass,
ponding to the transition between the two
time, etc. are fundamental units.
hyperfine levels of the ground state of the
6. Derived units. All other units which can be expressed cesium-133 atom (1967).
in terms of fundamental units are called derivedunits.
(iv) Ampere (A). One ampere is that constant current
7. System of units. A complete set of units which is which, if maintained in two straight parallel
used for measuring all kinds of fundamental and conductors of infinite length, of negligible
derived quantities is called a system of units. circular cross-section, and placed 1 metre
2.67
2.68 PHYSICS-XI

apart in vacuum, would produce between (xiii) 1 solar year =365.25


average solar days
these conductors a force equal to 2 x 10- 7 366.25 sidereal days
=
newton per metre of length (1948). (xiv) 1 lunar month = 27.3days
(v) Kelvin (K). One kelvin is the fraction 1/273.16 (xv) 1 shake = 10- 8 s
of the thermodynamic temperature of the
(xvi) 1 Torr = 1 mm of Hg
triple point of water (1967).
(xvii) 1bar1atmospheric pressure
=
(vi) Candela (cd). Candela is the luminous intensity,
= 760mm of Hg = 105 Nm-2.
in a given direction, of a source that emits mono-
13. Order of magnitude. The order of magnitude of a
chromatic radiation of frequency 540 x 1012
physical quantity is that power of ten which is
hertz and that has a radiant intensity in that
closest to its magnitude. To determine the order of

R
direction of 1/683 watt per steradian (1979). magnitude of a number N, we express it as
(vii) Mole (mol). One mole is the amount of N = n x 1(1. If 0.5 < n ~ 5, then x will be the order of
substance of a system, which contains as magnitude of N.
many elementary entities as there are atoms in 14. Annual parallax. It is the angle (9) at which the
0.012 kilogram of carbon-12 (1971). semi-major axis of the earth's orbit perpendicular

SI
9. The two supplementary 51 units. . to the star's direction is seen from the star.
(i) Radian (rad). It is defined as the plane angle
Star
subtended at the centre of a circle by an arc
equal in length to the radius of the circle.

o(mra ditarts ) =---=-


Cl' Arc I
. Radius r
(ii) Steradian (sr). It is the solid angle subtended
at the centre of a sphere by a surface of the
IT
sphere equal in area to that of a square, having
each side equal to the radius of the sphere.

u(m. stera dian)


rv Surface area
Ian 2 =
Radius
10. Coherent system. A coherent system of units is a
system based on a certain set of fundamental units
H
from which all derived units are obtained by 15 Indirect methods for measuring long distances.
multiplication or division without introducing (i) Reflection or echo method. The distance (5) of a
any numerical factor. hill can be determined by noting the time interval t
11. Advantages of 51. It is a metric, rational, coherent of sending a sound wave towards the hill and
and internationally accep"able system of units receiving back its echo.
O

ext
which covers all branches of physics. 5=-
2 '
12 Some Practical Units.
where c = Speed of sound waves
(i) 1 fermi = 10-15 m
The same principle is used in radars and sonars,
(ii) 1angstrom (A) = 10- 10 m (ii) Triangulation method. The height (h) of an
M

(iii) 1 nanometre (nm) = 10- 9 m accessible object like a tower or a tree can be found
by measuring the angle 9 subtended by the object
(iv) 1 micron (urn) = 10- 6 m
at the observation point. If x is the distance of the
(v) 1 light year (ly) = 9.46 x 1{y,5 m observation point from the foot of the object, then
(vi) 1 astronomical unit (AU) = 1.496 x to" m h = x tan t:I
(vii) 1 parallactic second (parsec) = 3.08 x 1016 m To find the height of an inaccessible object like a
(viii) 1 barn = 10- 28 m2 hill, its angles of elevation l\ and 92 are measured
at two points separated by distance d.
(ix) 1 tonne = 1000 kg
h= d
(x) 1quintal = 100 kg
cot 92 - cot l\
(xi) 1 slug = 14.57 kg
(iji) Parallax method. By measuring the angle 9
(xii) 1 atomic mass unit (amu) = 1.66 x 10- 27 kg (called parallax angle or parallactic angle) subtended
UNITS AND MEASUREMENTS 2.69

by the astronomical object at two locations on the Thickness of oil film,


earth separated by a large distance b (called basis), Volume of the film
the distance 5 of the object can be determined. t=--------
Area of the film
e=~=~ or 5= Basis b Assuming that the film is of one molecular
Radius 5 Parallactic angle e
thickness, then t is approximately the size of oil
(iv) Size of an astronomical object. The angle e molecule.
(called angular diameter) subtended by the diameter 19. Mass. The mass of a body is the quantity of matter
of the planet is measured at a point on the surface contained in it. Its SI unit is kg.
of the earth. Knowing the distance 5 of the planet 20. Weight. The weight of a body is the force with

R
from the earth, its diameter D can be determined. which a body is attracted towards the centre of
the earth. W = mg.
e=~= D or ti-s ;»
Radius 5 21. Inertial mass. The mass of a body which determines
Linear diameter = Distance x Angular diameter. its inertia in translatory motion is called its inertial
(v) Copernicus method. This method is used to mass. It is measured from Newton's second law of

SI
determine the distance of an inferior planet from motion.
the sun. The angle formed at the earth between the F
m·=-
earth-planet direction and the earth-sun direction I a
is called the planet's elongation (e). 22. Gravitational mass. The mass of a body which
Distance of inferior planet from the sun, determines the gravitational pull upon it due to
the earth is called its gravitational mass. It is
rps = sin e. res = sin I:. AU
measured by using Newton's law of gravitation.
Distance of inferior planet from the earth,
FR2
rpe = cos e. res =cos e AU. m =--
IT
g GM
(vi) By applying Kepler's third law. If 1i and 12 be 23. Dimensions of the derived quantity. These are
the periods of revolution of two planets and the powers to which the fundamental units of
~ and a2 be their semi-major axes, then mass, length and time must be raised in order to
3
represent a derived quantity completely.
a2 _ 12 2 24. Dimensional formula. It is an expression which
~3 -~2
H
shows how and which of the fundamental units of
mass, length and time occur in the derived unit of
or a
2 =[ 12 ]2/3 . ~ a physical quantity.
1i .
25. Dimensional equation. The equation which
16. Molar Volume. One mole of every gas occupies a expresses a physical quantity in terms of the
O

volume of 22.4 Htres at S.T.P. This is called molar fundamental units of mass, length and time, is
volume of the gas. It contains an Avogadro's called dimensional equation.
number (N = 6.023 x 1023 ) of molecules. 26. Principle of homogeneity of dimensions. This
17. Indirect methods for measuring small distances. principle states that a physical equation will be
(i) By using Avogadro's hypothesis. According dimensionally correct if the dimensions of all the
to Avogadro's hypothesis, the actual volume terms occurring on both sides of the equation are
M

occupied by the atoms in one gram of a the same.


substance is two-third of the volume of one 27. Uses of dimensional equations:
gram of the substance. (i) To convert a physical quantity from one
Radius of atom, system of units to another.

r=[ 2n~p r /3 Suppose a physical quantity has dimensional


formula MaLbT c. Let t; and ~ be its numerical
values when the units are ~ and ~. Then
where N, M and p are the Avogadro's number, t;~=~~
molecular weight and density of the substance. t; [MIa L; TIel = ~ [M; L; T~ 1
(ii) Molecular size of oleic acid. A large and very
thin film of oil is formed on water surface and
:. ~ = t; [ ~: ] a [ ~: ] b [ ~ ] C
its area is measured.
COMPETITION \'

2.70 PHYSICS-XI

(ii) To check the correctness of a given physical 33. Accuracy and precision of a measurement.
relation. Accuracy refers to the closeness of a measurement
(iii) To derive a relationship between different to the true value of the physical quantity. Precision
physical quantities. refers to the resolution or the limit to which the
quantity is measured.
28. Significant figures. The significantfigures are nor-
mally those digits in a measured quantity which are 34. Error in a measurement. It is the difference
between the measured value and the true value of
known reliably plus one additional digit that is
a physical quantity. It gives an indication of the
uncertain.
limits within which the true value may lie.
29. Rules for counting the number of significant
figures in a measured quantity: 35. Elimination of random error. The errors which

R
occur irregularly and at random, in magnitude
(i) All non-zero digits are significant. and direction, are called random errors. To
(ii) All zeros between two non-zero digits are eliminate random error, a large number of
significant. readings are taken and their arithmetic mean is
(iii) All zeros to the right of a non-zero digit but to taken as the true value.

SI
left of an understood decimal point are not _ a + a + a + ..... + an 1 n
1 2 3
a = ---.0-_-=-_-=- ""':":"
- - La.
significant. But such zeros are' significant if
n n i=l I
they come from a measurement.
(iv) All zeros to the right of a non-zero digit but to 36. Absolute error. The magnitude of the difference
the left of a decimal point are significant. For between the true value and the measured value is
example, 648700. has six significant figures. called absolute error.
(v) All zeros to the right of a decimal point are Such errors are given by
significant. ~~=ii-~ ~a2=ii-a2
(vi) All zeros to the right of a decimal point but to
IT
the left of a non-zero digit are not significant.
Single zero conventionally placed to the left of 37. Mean absolute error. The arithmetic mean of the
the decimal point is not significant. positive magnitudes of all the absolute errors is
(vii) The number of significant figures does not called mean absolute error
depend on the system of units. _ 1~~I+I~a21+ ..·.. +I~a I 1 n
30. Significant figures in the sum or difference of
~a = n
n = -n ;=1
L I ~a·1
H
I

two numbers. In addition or subtraction the result


38. Relative error. It is the ratio of the mean absolute
should be reported to the same number of decimal
error to the true value.
places as that of the number with minimum
number of decimal places. 8a = ~ii
31. Significant figures in the product or quotient of ii
O

two numbers. In multiplication or division, the 39. Percentage error. The relative error expressed in
result should be reported to the same number of percent is called percentage error.
significant figures as that of the number with ~a
minimum of significant figures. Percentage error = -=- x 100%.
a
32. Rules for rounding off a measurement:
40. Error combination in a sum or a difference. When
M

(i) If the digit to be dropped is less than 5, then two quantities are added or subtracted, the
the preceding digit is left unchanged. absolute error in the final result is the sum of the
(ii) If the digit to be dropped is greater than 5,
absolute errors associated with the individual
then the preceding digit is increased by 1. quantities.

(iii) If the digit to be dropped is 5 followed by ~Z=M+ ~B


non-zero digits, then the preceding digit is 41.' Error combination in a product or a quotient.
increased bv 1. When two quantities are multiplied or divided,
(iv) If the digit to be dropped is 5, then the prece- the fractional error in the final result is the sum of
ding digit is left unchanged if it is even. the fractional errors of the two quantities.
(v) If the digit to be dropped is 5, then the prece- ~Z M M
-=-+-
ding digit is increased by 1 if it is odd. Z A B
SECTION
.'

UNITS AND MEASUREMENTS 2.7,1

Error due to the power of a measured quantity. Opera- Form-


Relative Percentage
Absolute
The fractional error in the nth power of a,quantity error error l00AZ/ Z
tion ula Z error AZ
is equal to n times the' fractional error in. the AZ/ Z
quantity Itself. M+~B M+~B
Sum A+B ~A+ ~B --- ---x 100
AZ, AA' A+B A+ B
If Z =: An, then -'=:n-- M+~B M+~B
.Z, A", Differ- A-B M+~B --- ---x100
" AP 8'1 .'" ence A-B A-B
43. General rule. If Z ;= -' -.r - r then the maximum
C
fractional or relative error 1n.Z will be ,,, , Multi- AxB AM+BM
~A
-+-
~B (~:+~:)x1oo
plication A B
AZ 'M M 'AC ..'

R
--=:l'-*q~"'T-
Z A BC A MA+AM ~A ~B (~:+~:)x1oo
Division - -+-
% Error in Z =: AZ x100 B BZ A B
Z '
, M • i\R'" AE' " nA"-l~A n-
M n ~Ax 100
=J1-. 100 +. q ~B x ]00 x 100 Power A"
,-.X
A .,
+:1--.'
C . A A
1M .!. M x 100

SI
All. .!.A1/.-1M --
44. Propagation or combination of errors, It is'sum-' Root
n n A n A
marised in the adjacent table: '

lIT Entrance Exam


;I MIUJL"1T"~IF'LE
!cHD~!cE QIUJESnlClNS W~TH 5. A quantity X is given by toL AV where EO is the.
t.t
DNE !CDRRE!CT ANSWER
permittivity of the free space, L is a length, AV is a
IT
1. The equation of state for a real gas is given by potential difference and t.t is a time interval. The
dimensional formula for X is the same as that of
( P+ ~2 )(V -b);= RT
(a) resistance (b) charge

The dimensions of the constant a are (t) voltage (d) current [lIT 01]
(a) [ML5r2] (b) r~lL5T2] 6. Which of the following sets have different
H
dimensions?
(c) [ML-5 r1] (d) [ML5rl]
[lIT 97 ; CBSE PMT 96] (a) Pressure, Young's modulus, stress

a
2. Pressure depends on distance as p;= -exp
13
(-az)
ke
-- ,
(b) Emf, potential difference, electric potential
(c) Heat, work done, energy
O

where a, 13 are constants, z is distance, k is Boltzmann's (d) Dipole moment, electric flux, electric field
constant and e is temperature. The dimensions of 13 are [lIT 05]

(a) [MoLoTo] (b) [M-1L-1r1] 7. A cube has a side of length 1.2 x 10-2 m. Calculate
its volume.
(c) [MoL2To] (d) [M-1L-1T::] [lIT 04]
(a) 17x 1O-6m3 (b) 173 x 1O~6m3
}oE
M

2
3. The dimension~ of (~ are 6
(c)l.Ox10- m 3 (d) 1732xlO-6m3 [lIT 03]
8. A student performs an experiment for determi-
(EO : permittivity
(a)[MLrl]
of free space, E : electric field)
(b) [ML2r2] nation of g( =: ~:l). The error in length 1is t:.l and in
(c) [ML-1r2] (d) [ML2rl] [lIT 2K]
time T is t:.T and n is number of times the reading is
4. In the formula : X;= 3YZ2, X and Z have taken. The measurement of K is most accurate for
dimensions of capacitance and magnetic induction Al AT n
respectively. What are the dimensions of Y in MKSQ
(a) 5mm 0.2 see 10
system?
(b) 5mm 0.2 see 20
(a) [M-3L-IT3Q4j (b) [M-"L-2T4Q4j (c) 5mm 0.1 see 10
(c) [M-2L-2T4Q4] (d) [M-"L-2T4Qlj . [lIT 95] (d) Imm 0.1 see 50 [IIT 06]
2.72 PHYSICS-XI

9. A student performs an experiment to determine It moves 0.5 mm on main scale in one rotation. The
the Young's modulus of a wire, exactly 2 m long, by diameter of the ball in figure (ii) is
Searle's method. In a particular reading, the student (a) 2.25 mm (b) 2.20 rom
measures the extension in the length of the wire to be
(c) 1.20 mm (d) 1.25 rom [lIT 06)
0.8 rom with an uncertainty of ± 0.05 mm at a load of
13. A Vernier calipers has 1 rom marks on the main
exactly 1.0 kg. The student also measures the diameter
scale. It has 20 equal divisions on the Vernier scale
of the wire to be 0.4 mm with an uncertainty of
which match with 16 main scale divisions. For this
± O.Olmm. Take g =9.8 m/s? (exact). The Young's
Vernier calipers, the least count is
modulus obtained from the reading is
(a) 0.02 rom (b) 0.05 rom

R
(a) (2.0 ±03)x 1011N/m2
(a) (2.0±02)x 1011N/m2 (c) 0.1 rom (d) 0.2 rom [lIT 2010]

(c) (2.0±O.l)x 1011N/m2 14. The density of a solid ball is to be determined in


an experiment. The diameter of the ball is measured
(d) (2.0± 0.05)x 1011N/m 2 [lIT 07]
with a screw gauge, whose pitch is 0.5 rom and there

SI
10. A wire has a mass 0.3 ± 0.003 g, radius are 50 divisions on the circular scale. The reading on
. 0.5 ± 0.005 rom and length 6 ± 0.06 cm. The maximum the main scale is 2.5 rom and that on the circular scale
percentage error in the measurement of its density is is 20 divisions. If the measured mass of the ball has a
(a) 1 (b) 2 relative error of 2%, the relative percentage error in the
density is
(c) 3 (d) 4 [lIT 04)
(a) 0.9% (b) 2.4%
11. Students I, ITand III perform an experiment for
measuring the acceleration due to gravity (g) using a (c) 3.1% (d) 4.2% [lIT 2011]

simple pendulum. They use different lengths of the


IT
pendulum and/or record time for different number of " MlYb "f8If'1b~ ~IHIQJI~~ ~Y~~'ii"!IQJIf>:II~ WI'ii"il"3
oscillations. The observations are shown in the table. :el ~ Cit1 IQJ~~ H~ ~t>!Il§: jg~~~~C'iF

Least count for length = 0.1 em AiM~ iIl:R


Least count for time = 0.1 s 15. The dimensions of the quantities in one (or
more) of the following pairs are the same. Identify the
Length of No. of Total orne for Time
H
Stu- pair(s)
the pendu- osdlla- (n) osdlla- period
dent
lum (em) tions(n) oon(s) (s) (a) Torque and work
I 64.0 8 128.0 16.0 (b) Angular momentum and work
II 64.0 4 64.0 16.0 (c) Energy and Young's modulus
~ 4
O

III 20.0 36.0 9.0 (d) Light year and wavelength [lIT 86]

16. The pairs of physical quantities that have the


If E1, Ell and Em are the percentage errors in g i.e.,

(7 x 100) for students I, ITand III respectively,


same dimensions are
(a) Reynold number and coefficient of friction
(b) Curie and frequency of a light wave
M

(a) E1 =0 (b) El is maximum


(c) Latent heat and gravitational potential
(c) e, = Ell (d) Ell is maximum [lIT 08)
(d) Planck's constant and torque [lIT 95]
12. In a screw gauge, the zero of main scale
17. The dimensions of length are expressed as
coincides with fifth division of circular scale in
GX d1hZ where G, c and h are the universal gravitational
figure (i). The circular divisions of screw gauge are 50.
constant, speed of light and Planck's constant
respectively, then
(a) x =(1/2), y =(1/2)
(b) x =(1/2), z =(1/2)
(c) y =(-3/2), z =(1/2)
(i) (il) (d)y=(1I2),z=(3/2) [lIT 92]
UNITS AND MEASUREMENTS 2.73

18. The 51 unit of inductance, the henry can be vacuum. If M = mass, L = length, T = time and 1=
written as electrical current, then the dimensional formulae of go
(a) weber/ampere (b) volt-see/amp and Ilo are
(b) joule/(ampere)2 (d) ohm-second [lIT 98] (a) [go] = ~lL-3r21 (b) [go] = ~lL-3r412
19. If L C, R represent inductance, capacitance and
(c)[llo] = MLr2r2 (d)[llo] = ML2r11 [lIT 98]
resistance respectively, the combinations having
dimensions of frequency are 22. A student uses a simple pendulum of exactly 1 m
length to determine g, the acceleration due to gravity.
(a) k (b) ~ He uses a stop watch with the least count of 1second

R
for this and records 40 seconds for 20 oscillations. For
(c) R (d) R this observation, which of the following statement(s) is
L C [lIT 84]
(are) true ?
20. Which of the following combinations have the (a) Error Il T in measuring T, the time period, is
dimensions of time ? L, C, R represent inductance, 0.05 second

SI
capacitance and resistance respectively.
(b) Error Il T in measuring T, the time period, is
(a) RC (b) .J[C 1 second
(c)R/L (d)C/L [IlT86] (c) Percentage error in the determination of g is 5%
21. Let [go] denote the permittivity of the (d) Percentage error in the determination of g is
vacuum, and [Ilo] denote the permeability of the 2.5% [lIT 2010]
IT
23. Match the physical quantities given in column I 24. Column I gives three physical quantities. Select
with dimensions expressed in terms of mass (M), the appropriate units for the choices given in column II.
length (L), time (T), and charge (Q) given in column n. Some of the physical quantities may have more than
[lIT 83] one choice correct. [lIT 90]

1 11 1 11
H
(a) Angular momentum (p) ML2r2
(a) Capacitance (p) ohm-second
(b) Torque (q) ML2r1
(b) Inductance (q) coulomb Ljoule "
(c) Inductance (r) M-1L-2T~2
(c) Magnetic Induction (r) coulomb/volt)"!
(d) Latent heat ~ (s) ML~-2
O

(e) Capacitance (t) ML3r1Q-2 (s) newton (ampere m)-l

if) Resistivity (u) L2r2 (t) volt-second (ampere)-l

25. Some physical quantities are given in column I and some possible 51 units in which these quantities may be
expressed are given in column II. Match the physical quantities in column I with the units in column II. [lIT 07]
M

1 11
(a) GMeMs; G = universalgravitation constant, (p) (volt) (coulomb) (metre)
Me = mass of earth, M. = mass of sun
3RT
(b) -- (kilogram) (metrej ' (secondj
; R = universal gas constant, (q) ?

M
T = absolute temperature M = molar mass
(c) F2 (r) (metre) 2 (second)-2
F = force, q = charge, B = magnetic field
q2W'

GMe.
(d) --, (s) (farad) (volt) 2 (kg)-l
G = universal gravitational constant
Re
Me = mass of earth, Re = radius of earth
" c:
COMPETITION

2.74 PHYSICS-XI

Answers and Explanations


1. (a) As quantities with similar dimensions can be 41t21
8. (d) Given g = -2-
added only, so T

[;2] = [P]
~g = ~l +2 ~T
g 1 T

or [a] =[PHV2] = [ML-1r2HL3f ~l is minimum in option (d),

R
= [ML5r2] ~T is minimum in options (e) and (d),
n is maximum in option (d)
2. (e) Here k = Boltzmann's constant
Hence error will be minimum or the measurement
[k]= [joule] =[ML2r2S-1] of g will be most accurate in option (d).
[kelvin]
9. (b) Young's modulus,

SI
As an exponential factor is dimensionless,
Y _ F L_ 4Mg L
[: J = [~L°r>] - A '[- 7td2 "i
2 1 Y= 4x1.0x9.8x2 =2Dx1011 Nm-2
or [a] = [kzSJ = ML r:S- .s = [MLr2] 1t(0.4x 10-3)2 x 0.8 x 10-3

As the values of L, M and g are exactly given, so


~Y =2~d+~1
Y d 1
IT
or ~Y =(2 xO.D1 + 0.05)X2 x 1011Nm-2
0.4 0.8
=01 x 1011Nm-2
3. (e).!E E2 = Energy
2 0 Volume :. Measured value ofY = (2.0±0.2)x 1011 Nm-2.

:. [.!E E2] = [ML2r2] = [ML-1r2]


H
10. (d) Density = Mass
2 0 [L3] Volume
m
4. (b) [X] = [C] = [M-1L-2T2Q2]
p = 1t?/
[Z] = [B] = [MrlQ-l] The maximum percentage error in density,
O

[Xf [M-IL-2T~2]
[Y] = [Z2] = [MT-1Q-1f ~p x 100 =
p
[~m +2
m
M+
r I
~lJx 100
= ~3L-2T4Q4] = [0003 + 2 x 0005 "-OD6Jx 100
03 0.5 6
5. (d) [EoL] = [capacitance]
= [_1_ + -.3.- + _l_J x 100 = 4.
M

EoL ~ V = C~V = ~q = charge = current. 100 100 100


M M M time
6. (d) Dipole moment = Charge x distance 11. (b) g = 41t2( ;2)
Electric flux = Electric field x area
~g = ~l +2 ~T
:. Dipole moment, electric flux and electric field
g 1 T
have different dimensions.
E = ~l +2 M
7. (a) Volume = (sidej ' =(1.2 x 1O-2m)3
/ t '
=1728x10-6m3 =17x10 6
m3 greater the value of t, lesser the error.
As the side has 2 significant figures only, so the Hence, fractional error in the 1st observation is
expressed volume must have 2 significant figures only. minimum.
SECTION -

UNITS AND MEASUREMENTS 2.75

12. (c) Least count of screw gauge


Pitch O.5mm ~LITJ = ~x+z L3x+y+2z T-2x-y-z
--------=---
No. of circular divisions 50 -x+z=0,3x+y+2z=l,-2x-y-z=O
=0.01 mm 1 3 1
Hence x = -, y = --, z =- .
Initial reading = 0 + 5 x·om = 0.05 mm 2 2 2
Final reading =2 x0.5+25xOm = 1.25 mm 18. (a), (b), (c), (d)
Diameter of ball = 125 -ODS = 1.20 mm.
(a) L= flux ..1H=IWb
13. (d) 20VSD= 16MSD current lA

R
IVSD= 16MSD=iMSD (b) L- E .. IH= 1 V =1 VsA-1
20 5 d[ / dT A/s
LC = 1MSD-l VSD = 1MSD-i MSD
(c) U=!LP or L=2U IJ
5 :.IH=--2
2 [2 lA
. 1 1

SI
= - MSD = - x Irnm = 0.2 mm
5 5 (d) 1 H = 1 V.s = 1 n s.
lA
14. (c) Least count of screw gauge
O.5mm 1 19. (a) and (c)
=---=-mm=O.Ol mm
50 100 [R] = [ML2r3A-2]
:. Diameter D [L] = [ML2r2A -2]
= MSR+CSRx LC [C] = [~lL-2ylA2]
= 2.5 + 20 x 0.01 = 2.70 mm 1 1
,....-;::;
= - = [frequency]
IT
. m v LC [T]
Now, density = 3

~~(~) and [ RJL = ~[T] = [frequency]


:. % error in density 20. (0) and (b)
Sm ~D [RC] = [ML2r3A-2][~lL-2ylA2] =[T]
= -x 100+3-x 100
H
m D [LC] = [ML2r2A-2][~lL-2ylA2] =[T2]
=2%+3x 0.01 x 100 =3.1% .. [.JLC]=[T]
2.70
21. (c) F = _1_qlq2 or I: =_1_qlq2
15. (0), (d) 41tl:o ,2 0 41tF,2
(0) Torque and work both have the dimensions
O

[ML1r2]. [I: ] = [IT][IT] = [M-1L-3T4f]


o [MLr2][L2]
(d) Lightyear and wavelength both have the
dimensions [L]. 1 ~
Also, --=c-
16. (a), (b), (c) !lol:o
(a) Reynold's number and coefficient of friction or 1 1
M

both are dimensionless quantities. [!lo] = [I:o][?] = [~IL-3ylI2][Lrl]2

(b) Curie = Numbe~ of atoms = [rI] = [MLr2r2].


TIme
22. (0), (c) Relative error in measurement of time
Frequency =
. No. of vibrations
. =
rr'i
1second 1
TIme
4Osecond 40
(c) [Latent heat] = [Gravitational poten~,all
Energy Time period = 40 = 2 second
20
Mass
:. Error in measurement of time period
17: (b) and (c). 1 1
Here L= GXcllhz. = 2 x - = - second = 0.05 second
40 20
2.76 PHYSICS-XI

1
Now, goc- [For constant I] 24. (a) Capacitance, C =!L = coulomb volt-1
T2 , V
t:.g 2t:.T 2 xlI 2
-=--=--=-
g T 40 20 or C =-q- =!L =cou]omb2joule-1
W/q W
t:.g x 100 = 5%
(b) Inductance,
g
L=_c_= volt
23. dT / dt ampere / see
(a) [Angular momentum] = [Mer1] = volt sec ampere-1 = ohm see.

R
(d) [Latent heat] = [er2] (c) Magnetic induction,
(b) [Torque] = [ML2r2] B_ £. _ newton
(e) [Capacitance] = [~lL-2T2Q2] n ampere metre

(c) [Inductance] = [ML2Q-2] = newton (ampere metrej "

SI
if) [Resistivity] = [ML3r1Q-2] .. (a) ~ q, r; (b) ~ p, t; (c) ~ S.

:. (a) ~ q; (b) ~ p; (c) ~ 5; 25. (a) ~ p, q; (b) ~ r, S ;

(d) ~ u ; (e) ~ r; (f) ~ t. (c)~r,s; (d)~r,s.

AIEEE
1. Which of the following represents the correct 6. Which of the following units denotes the
IT
dimensions of the coefficient of viscosity ? dimensions ML2Q-2, where Q denotes the electric charge?
(a) [ML-lr2] (b) [ML-2r2] (a) henry (H) (b) weber (Wb)
(c) [ML-1r1] (d) [ML r1] [AIEEE 04) (c) Wb m -2 (d) Hm -2 [AIEEE 06)

2. Dimensions of _1_, where symbols have their 7. The dimensions of magnetic field in M, L, T and
Ilo€o C (Coulomb) are given as
H
usual meanings, are: (a) [MLr1C1] (b) [MT2C2]
(a) [L-1T] (b) [L2T2] (c) [Mr1C1] (d) [MT-2C1] [AIEEE 08)
(c) [L2r2] (d) [Lrl] [AIEEE 03) 8. Two full turns of the circular scale of a screw
3. Identify the pair whose dimensions are equal. gauge cover a distance of 1mm on its main scale. The
O

(a) torque and work (b) stress and energy total number of divisions on the circular scale is 50.
(c) force and stress (d) force and work Further, it is found that the screw gauge has a zero
[AIEEE 02) error of - 0.03 mm. While measuring the diameter of a
4. Out of the following pairs, which one does not thin wire, a student notes the main scale reading of
have identical dimensions 3 mm and the number of circular scale divisions in line
M

with the main scale is 35. The diameter of the wire is


(a) Moment of inertia and moment of a force
(a) 3.32 mm (b) 3.73 mm
(b) Work and torque
(c) Angular momentum and Planck's constant (c) 3.67 mm (d) 3.38 mm [AIEEE 08)

(d) Impulse and momentum [AIEEE 05) 9. In an experiment the angles are required to be
measured using an instrument. 29 divisions of the
5. The physical quantities not having the same main scale exactly coincide with the 30 divisions of the
dimensions are vernier scale. If the smallest division of the main scale
(a) torque and work is half-a-degree (= 0.5"), then the least count of the
(b) momentum and Planck's constant instrument is
(c) stress and Young's modulus (a) one minute (b) half minute

(d) speed and 1/ Jllo€o [AIEEE 03) (c) one degree (d) half degree r \IEEE (9)
UNITS AND MEASUREMENTS 2.77

10. The respective number of significant figures for Given that 1 mm on main scale corresponds to 100
the numbers 23.023,0.0003 and 2.1x 10-3 are divisions of the circular scale.
(a) 4, 4, 2 (b) 5, 1, 2 The diameter of wire from the above data is
(c) 5, 1, 5 (d) 5, 5, 2 [AIEEE 2010] (a) 0.52 em
11. A screw gauge gives the following reading (b) 0.052 em
when used to measure the diameter of a wire. (c) 0.026 em
Main scale reading: 0 rom (d) 0.005 em [AIEEE 2011]
Circular scale reading: 52 divisions

Answers and Explanations

R
Force Distance
1. (c) T\ = --x .. [B] = [MLr2] = [Mr1C-1].
Area Velocity [CT-lUl]

SI
[ ] = [MLr2].~ = [Ml-1r1]. 8. (d) Pitch of screw gauge =.! mm = 0.5 mm
T\ [l2] [LT-1] 2
1 .
2. (c) -- =?, c = speed of light O.5mm
Least count = --- = 0.01 mm
lloEo 50
Observed diameter = 3 mm + 35 x om mm
.. [_1_] = [lrl]2 = [l2r2]
lluEo =3.35 rom
Corrected diameter
3. (a) LTurqueJ = [Wurk] =LML:!r2J
IT
= Observed diameter - Zero error
4. (a) [Moment of inertia] =[ML2~]
= 3.35 - (-0.03) = 3.38 mm.
[Moment of force] = Force x distance
= [ML2r2] 9. (a) 29 MSD= 30 VSD

5. (b) [Momentum] = [MLrt] 1 VSD= 29 MSD


30
H
[Planck's constant] = [ML2rl].
Least count = 1 MSD-1VSD
EMF
6. (a) Inductance = -----
Current / time =(1- 29)MSD =~ MSD
30 30
Work Time
O

Charge Current
= 3~(~r =(6~r =1 minute.
[ML2r2] [T]
[Inductance] = .-- 10. (b) 23.023 --+ 5 (2, 3, 0, 2, 3) ;
[Q] [Q/T]
0.0003 --+ 1 (3) ; 2.1 x 10-3 ~ 2 (2, 1) .
[Henry] = [ML2Q-2]
11. (b) d=MSR+CSR
M

Henry is the SI unit of inductance.


1
7. (c) F = nB or B= £. = 0 + 52 x - = 0.52 rom = 0.052 em
n 100

DeE and G.G.5. Indraprastha University Engineering Entrance Exam ---'

1. Out of the following four dimensional quantities, (b) Surface tension of water
which one qualifies to be called a dimensional constant? (c) Weight of a standard kilogram mass
(a) Acceleration due to gravity (d) The velocity of light in vacuum. [IPUEE 06]
'""': • ~.' • $- • ':~~.: .'. ~~~~:,~ •• ~~.: ~. -

:'jCOM,PETITION:'
. ......,..: ...:
, ~ ~~ - r

2.78 PHYSICS-XI

2. The dimensions of torque are The relation is


(a) [ML2T2] (b) [ML2r2]
3 )1/2
(a) T ec p~ (b) T cc prS
(c) [M2L2r2] (d) [MLrl] [DeE 00] (

3. [ML2r2] are dimensions of 5


(c) T oc pr (d) T oc-
(a) force (b) moment of force 5 pr
(c) momentum (d) power [DeE 04]
10. Which one of the following pairs of quantities
4. The dimensions of Planck's constant are
has the same dimension ?
(a) [M2L2r2] (b) [MLr2]
(a) force and work done

R
(c) [ML2r2] (d) [ML2rl]
(b) momentum and impulse
[DPM 01 ; DeE 07]
5. Dimensions of bulk modulus are (c) pressure and force
(a) [M'lLr2] (b) [ML-lr2] (d) surface tension and stress. [DeE 09]

(c) [ML-2r2]

SI
(d) [M2t2rl] [DeE 07] 11. If L=2.331 em, B=2.1 ern, then L+ B=?

6. The unit of a in van der Waal's gas equation is (a) 4.431 cm (b) 4.43 em

(a) atm L-2mo12 (b) atm L2per mol (c) 4.4 cm (d) 4 em [DeE 03]
l
(c) atm L- mol- 2 (d) atm L2mol-2 [DeE 97] 12. If error in radius is 3%, what is error in volume
7. In the relation, y = rsin( rot + 10:), the dimensional of sphere?

formula for kx or rot is same as (a) 3% (b) 27%


(a) r / co (b) r / y (c) 9% (d) 6% [DeE 06]
IT
(c) rot / r (d) yr / rot [DeE 03] 13. In an experiment, on the measurement of g,
8. Which of the following quantities can be written using a simple pendulum, the time period was
in SI units in kgm2A -2s-3 ?
measured with an accuracy of 0.2% while the length
(a) Resistance (b) Inductance was measured with an accuracy of 0.5%. The
(c) Capacitance (d) Magnetic flux [DeE 07] percentage accuracy in the value of g thus obtained is
9. The time period T of a small drop of liquid (due
H
(a) 0.7% (b) 0.1%
to surface tension) depends on density p, radius r and
(c) 0.25% (d) 0.9% [DeE 07]
surface tension S.

Answers and Explanations


O

1. (d) The velocity of light in vacuum 2


[P]=[~2J or [a]=[P][V ]
(c=3x108m/s)is a dimensional constant.
2. (b) [Torque] = [Force] [Distance] Unit of a = atm L2 per mole.
7. (b) rot and 10: both are dimensionless. Out of the
= [MLr2][L] = [ML2r2]
M

given options, only r / y is dimensionless.


3. (b) [Moment of force] = [Torque] = [ML2r2] 8. (a) [Resistance] = [ML2r3 A -2]

4. (d) [h) = Energy = [ML2r2] = [ML2rl] :. Unit of resistance =kg m2A-2s-3.


Frequency [rl] 9. (a) Let T = KpQrbSc
l2 .. [T] = [ML-3t [L]b[Ml2],
5. (b) [Bulk modulus] = Stre~s = [ML- r ]
Stram 1 or ~LoTl = Ma+cL-3a+bT-2c

= [ML-lr2] .. a + c = 0, -3a + b = 0, -2 c = 1
On solving, a=1 b=~ c = _1
6. (h) Van der Wall's equation is
~)1/2 2' 2.
Hence Toc L .
(P+ ~2 )(V-b) = RT ( s
SECTION

UNITS AND MEASUREMENTS 2.79

10. (b) [Change in momentum] = [Impulse]


13. (d) As T=21tt
= [MLrl].
11. (c) L+ B=2.331+2.1 =4.431 =4.4 em
.. g =41t2~T2
.: Bhas 2 significant figures.
:. L+ B must have only 2 significant figures.
ll.g x100= ll.lx100+2x ll.T x 100
12. (c) V = i 1tr3 g 1 T
3
ll. V t:.r = 0.5% + 2 x 0.2% = 0.9%.
.. -x 100 =3x -x 100 =3x3 =9%.
V r

R
AIIMS Entrance Exam

1. How many wavelengths of Kr86 are there in one 10. What is the dimensional formula of

SI
metre? gravitational constant G ?
(a) 15,53,164.13 (b) 16,50,763.73 (a) [M-1L3r2J (b) [M-2L3r2]
(c) 23,48,123.73 (d) 6,52,189.63 [AllMS 94] (c) [111er2] (d) [111L3rl] '\JJMS 00]

2. One nanometre is equal to 11. The dimension of angular velocity is


(a) 109 mm (b) 10-6 em (a) [MLr2] (b) [M2Lorl]
(c) 10-7 em (d) 10-9 m [AIIMS 94] (c) [MoLorl] (d) [ML2r2] [AItMS 98]

3. Light year is the unit of 12. Which of the following physical quantity has
IT
(a) time (b) distance the dimension of [ML2r3]?
(c) velocity (d) intensity of light (a) work (b) power
[AIIMS 96] (c) pressure (d) impulse [AIIMS 94]

4. Parsec is the unit of 13. The dimension of the modulus of rigidity, is


(a) time (b) distance (a) [MLr2] (b) [ML-1r2]
H
(c) frequency (d) angular momentum (c) [ML-2r2] (d) [ML-1r1] [AIIMS 94]
[AIIMS 96] 14. The dimension of Planck's constant is
5. Length cannot be measured by (a) [ML2rl] (b) [ML-3r1]
(a) fermi (b) debye (c) [ML-2rl] (d) [MlL-1r3] [AIIMS 97]
O

(c) micron (d) light year [AIIMS 02] 15. Dimensions [ML-1r1] are related to
6. The difference in the length of a mean solar day (a) work (b) torque
and a sidereal day is about (c) energy (d) coefficient of viscosity
(a) 1minute (b) 4 minutes [AIIMS 99]
(c) 15 minutes (d) 56 minutes 16. Which of the following is a dimensionless quantity ?
M

[AIIMS 03]
7. Gravitational mass is proportional to gravitational (a) Strain (b) Stress
(a) field (b) force (c) Specific heat (d) Quantity of heat
[AIIMS 94]
(c) intensity (d) all of these [AIIMS 08]
17. Which of the following pair does not have
8. SONAR emits which of the following waves?
similar dimensions ?
(a) radio (b) light
(a) stress and pressure
(c) ultrasound (d) none of these [AIIMS 99]
(b) angle and strin
9. The dimension of torque is
(c) tension and surface tension
(a)[MLr2] (b)[ML2r2]
(d) Planck's constant and angular momentum
(c) [ML-1r1] (d) [ML3r3] [AIIMS 02] [AIIMS 01,07]
2.80 PHYSICS-XI

18. Dimensions of electrical resistance are (c) angular momentum and Planck's constant
(a)[ML2r3A-l] (b)[Mtzr3A-2] (d) work and torque [AIIMS 2010J
3 3 2
(c) [ML T A- ] (d) [ML-1T3A2] [AIIMS 94) 21. If the energy, E = GPhq
d , where G is the
19. The magnetic moment has the dimensions of universal gravitational constant, h is the Planck's
(a)[LA] . (b)[L2A] constant and c is the velocity of light, then the values of.
(c) [Lrl A] (d) [L2rl A] p, q and r are, respectively
[AIIMS 06)
(a) -1/2,1/2 and 5/2
20. Which of the following pairs does not have
same dimensions? (b) 1/2, -1/2 and -5/2
(a) impulse and momentum (c) -1/2,1/2 and3/2

R
(b) moment of inertia and moment of force (d) 1/2, -1/2 and -3/2 [AIlMS 2010)

Answ~rs and fxplanations

SI
1. (b) One metre contains 1650763.73 wavelengths 15. (d) [Coefficient of viscosity] = [ML-1r1l
of certain orange radiation of Kr86. 16. (a) Strain is a dimensionless quantity.
2. (d) 1 nm = 10-9 m. 17. (a) [Tension] = [Force] = [MLr2]
3. (b) One light year is the distance travelled by light . [Force] [MLr2] -2
in on year. [Surface tension] = = = [MT ].
[Length] [L]
4. (b) One parse is the distance at which an arc of
IT
. P.O. Work
length 1 AU subtends an angle of 1 second of arc. 18. (b) Resistance = = ------
Current Chargex Current
5. (b) Debye is the unit of electric dipole moment
not of length or distance.
6. (b) The sidereal day is about 4 minutes (more
precisely, 3 minutes 56 seconds) shorter than a mean 19. (b) [Magnetic moment] = [Current] [Area]
H
solar day (of 24 hours).
= [A][L2] = [l2Al
7. (b) Gravitational mass is proportional to
gravitational force. 20. (b) Moment of inertia,
8. (c) SONAR emtis ultrasound. [I] = Mass x distance/ = [ML2]
Moment of force,
O

9. (b) [Torque] = fMl2r2].


['t] = Force x distance
10. (a) [G] = F"z = [MLr2][L2] = [M-ll3r21 = ML-2T· L = [ML2r2]
"'I111z [M][M]
[J] ;0: ['t] .
11. (c) [co] = [8] =..!..= [MoloT-1] 21. (a) Given E = GPhq c'
M

[t] T
:. [MIL2r2] =[M-1L3r2]p [ML2rl]q [Lr1r
12. (b) [Power] = work = [ML2r2] or M1L2r2 = M- p+q L3p+2q+TT-2p-q-,
time [T]
Equating the powers of M, L and T, we get
= [Ml2r31 -p+ q = 1
1 2
13. (b) [Modulus of rigidity] = Stre~s = [ML- r ] 3p+2q+r=2
Stram 1 -2p-q-r= -2
= [Ml-1r21 On solving, we get

14. (a) [h] = Energy = [ML2r2] = [Ml2rl] 1 5


q=- r=-
Frequency [rl] 2' 2
SECTION

UNITS AND MEASUREMENTS 2.81

CBSE PMT Prelims and Final Exams'


.1. Which of the following has the dimensions of 9. The ratio of the dimensions of Planck constant
pressure? and that of moment of inertia is the dimensions of
(a) [MLr2] (Ii) [ML-Ir2] (a) time (b) frequency
(c)[ML4r2] (d) [~IL-I] '(c) angular momentum (d) velocity l<;BSEPMTOS]
J [CBSEPMT90, 94] 10. Dimensions of re~tance in an electrical circuit,
2. The. d~mensional formula of torque is in terms of dimension of mass M, oflength L, of time T
(a) [ML2r21.u r ,(b) [Merl]
and of current I, would be

R
(cHML-:I:r.:~] i..,. ~3(d){ML-2r2] [CBSEPMT891 (a) [ML2r2] (b) [ML2,rlrl]
(c)[ML2r3r2] (d) [ML2r3rl]
3. The 'diinens{o~S'- of 9'universal qravitational
. [CaSE PM,r 07]
constant are '." ;', .. 0." •
(a) t~IL3r2]'" -, (b) [ML2T-1] 11. Dimensional formula of self ~ductance is

SI
(a) [MLT-2A-2]. (b) [ML2rIA-2].
(c)[M-2e'12] . I - "(d)'[M-2L2rl r 2 2 l
,
(c)[ML2r2A-2] .•(d)[ML r A- ]·.
[CBSE PMT 92, 04]
f ; I,. rl , ) [CBSE PMT 89]
,4. The dimensions .
of impulse are equal to that of
12. If C and R denote capacitance and resistance.,
(a) pressure (b) linear momentum the dimensional formula of CR is
(c) force (d) angular momentum (a) [MoLoTI] (b) [MoLOr?
,. [CBSE PMT 89],
(c) [MoLarl]
~. According to Newton, the viscous force acting
IT
(d) not expressible in terms of MLl' [CaSE PMT 89]
between liquid layers of area A and velocity gradient
flv I Ax is given by 13. The dimension of RC is
, - Ilv (a) square of time (b) square of inverse time
F=-TJA-
/lx' (c) time (d) inverse time.
[CBSE PMT 95]
where TJ is constant called coefficient of viscosity. The
14. \Yhich of the foll()wing dimensions will be the
H
dimensional formula of TJ is
same as that of time?
(a) [ML-2r2] (b) [MoLoTo] .
L " C
(c)[Mer2j '(d)[ML-Irl] [CBSE,PMT90] (a) R -, (b) r: ..
6. Turpentine oil is flowing through a tube of length R
(c) LC (d) -. I
I and radius r. TheJpr~ssure difference between the two L (CBS£,PMT 96]
O

ends of the tube is P. The' viscosity of oil is given by 15. The unit of pemtiftivity of free space EO is
TJ =
p("z-x2) h
, were v
.
IS
th I' f il
e ve ocity 0 0 at a
, 4vl . _ ,c '
(a) coulomb/newton-metre
2
distance x from the axis of the tube. The dimensions of (b) newton-metre /coulomb/
(c) coulombf/newton-metref
M

TJ are • ' f

(a) [MoL°rD] (b) [MLFl] (d) coulombf/mewton-metrejf (CBSE PMT 04]

(c)[ML2r2] (d) [ML-lrl] [CBSE PMT 93] 16. The dimensional formula of magnetic flux is
(a) [MoL-1r2A-2] (b) [MLor2A-2]
, c

7.The dimensional formula of angular momentum is


(a) [ML2r2f' < - (b) [Mt-2rl] . (c)[ML2r2A-I] (d)ML2 rIA'3
[CBSE PMT 99]'
(c)[MLrl] ~ (d)JML2rll [c,BSE PtvIT.881
,-,fl.
1 ~... r
}7.'J!le dimensional.f0rmula of permeability of free
S. The dimensions of Planck; s: constant equals to
space ~o is.
that of -.
(a) [~ILr2A-2] .(b), [WLIT]
(a) energy . , (b) momentum
(c)[WL2rlA2] (d) none of these.
(c) angular momentum (d) power [CBSE PMT 01] lCBS£ PMT 91]
o
COMPETITION

2.12 PHYSICS-XI

18. Which pairs do not have equal dimensions? 25. The frequency of vibration f of a mass m
(a) Energy and torque (b) Force and impulse
suspended from a spring of spring constant k is given
(c) Angular momentum by a relation f = ant kY,
and Planck's constant where a is a dimensionless
. constant. The values of x and y are
~d) Blasttc modulus and pressure. (CBSE PMT 2IC)
1 1 1 1
19. Of the following quantities; which one has -(a) x = 2' ' y = 2' (b) x = -1" y = -2'
dimensions different from the remaining three ? 1 1 1 . 1
(c) x = - , y = -- (d) x = -- , y = -
(a) Energy per unit volume 2 2. 2 2
(b) Force per unit' area (CBSE PMT go)

R
(c) Product of voltage and charge per unit volume 26. P represents radiation pressure, c represents
(d) Angular momentum. (CBSE PMT 89) speed of light and S represents radiation energy
striking per unit area per sec. The non-zero integers
20. If x = at + bt , where x is the distance travelled
2
x, y, z such that pr SY c- is dimensionless, are
by the body in kilometres while t is the time in seconds,
then the unit of b is (a)x=l,y=l,z=l (b)x=-l,y=l,z=l

SI
(a) km/s (b) kms (c) x =1, Y =-1, z '=1 (d) x =l,y =1, z =-1·
. 192)
(c) km/s2 . (d) kms2 (CBSE PMT 89)
rl. If the dimensions of a physical quantity ate
21. An equation is given he~e, (p + ~2 ) = b~ , given by MQ l!'-rc, then the physical quantity. will be
(a) Velocity if a'=l, b =0, c =-1
where P = pressure, V = volume and e = absolute
temperature. If a and b are constants, the dimensions of (b) Acceleration if a = 1, b ~ 1,c =-2
a will be ' (c) Force if a =0, b = -1, c:=-2
IT
5 1
(a) [ML- r ] (b) [ML5Tl] (d) Pressure if a = 1, b = -1, c = -2. (9)
(c) [ML5r2} (d) [~lL5T2] 28. The density of a material in CGSsystem of units
(CBSE PMT 96) is 4 g/ em2. In a system of units in which unit of length
22. The velocity v of a particle at time t is given by is 10em and unit of mass is 100.g, the value of density
b . of material will be
v = at + -- , where a, b and c are constants. The
t+c (a) 400 (b) 0.04
H
dimensions of a, b andc are
(c) 0.4 (d) 40 [CBSE Final 2011)
(a)[L], [LT] and [Lr2]
29. Which of the. following is a dimensional
(b) [Lr2], [L] and [T]
constant?
(c) [L2], [T] and [Lr2]
(a) relative density (b) gravitational constant
O

(d)[Lr2],[LT]~d[L]. (CBSEPMT06]
(c) refractive index (d) poisson ratio.
23. The time dependence of a physical quantity pis [CBSE PMT 95]
2
given by p = Po exp( -at ~ where a is a constant and tis 30. Percentage errors in the measurement of mass
the time ..The constant a and speed are 2%and 3%respectively. The error in the
, i is dimensionless (b) has dimensions [12] estimate of kinetic energy obtained by measuring mass
M

(c) h~s dimensions [T2] and speed will be


. (it) has dimensions of p. [CBSE PMT 93] (a) 8% . (b) 2%
24; Which of the following five physical parameters (c) 12% (d) 10% [CBSE PMT 9S]
have the same dimensions? 31. The density of a cube is measured by measuring
its mass and length of its sides. If the maximum errors
1. energy, density 2. refractive index
in the measurement of mass and lengths are 3% and
3. dielectric constant 4. Young's modulus
2% respectively, the maximum error in the measure-
5. magnetic field ment of density would be
(a) 2 and 4 (b) 3 and 5 (a) 12% (b) 14%
(c) 1and 4 (d) 1 and 5 [c;:BSEPMT 08] (c) 7% (d) 9% [CBSE PMT 89]
SECTION

UNITS AND MEASUREMENTS 2.'~


32. A certain body weighs 22.42 g and has a 33. If the error in the measurement of radius of a
measured 'volume of 4.7 cc. The possible errors in the sphere is 2%~ then the error in the determination of
measurement of mass and volume are 0.01 g and 0.1 cc. volume of the sphere will be
Then maximum error in the density will be (a) 4% (b) 6%
(a) 22% (b) 2% (c) 8% (d) 2% (CBSEPMT (8)
.(c) 0.2% (d) 0.02% [CBSEPMr 91J

Answers .cind Explanations

R
1. (b) [Pressure] = [ML-1r2] 13. (c) Dimension of RC is same as that of time, as
·_Refer to point 11 of Table 2.8 on page 2.20. proved in the above problem. .

2. (a) [Torque] = [ML2r2]


14. (a) [L] ., [ML2r2A-2]

SI
= [T].
Refer to point 12 of Table 2.8 on page 2.20. R . [ML2r3A-2]
3. (a) [G) = [M-1L3T-2] 15. (c) From Coulomb's law,
Refer to point 13 of Table 2.8 on page 2.20. F = _1_ qlq2
4. (b) [Impulse] = [MLl-1] 4m;0 fl
Refer to point 14 of Table 2.8 on page 2.20.
or E = qlq2
o 41tFfl
5 _[F6.x]_[MLr2][L]
IT
. (d) [rt] - A~v - [L2][Lrl] 51 unit. 0 f EO= ~-2
C.C :;: e2/N m2
Nm .
= [ML-1T-1] •
16. (c) [q,] = BA = _F_. - A
2 qvsm9
6. (d) [ ] = p(fl-x ) = [ML-lr2HL~]
. rt 4vl [Lrl][L] ~. [MLr2][L2]
- [AT][Lrl).1
H
= [ML-1r1] ..
= [ML2r2A-1]
7. (d) tQ = [mvr] = [M][Lr1][L]
= [MI!T-1]. 17. (a) [ ] = 41trF= L.MLT;2
1-10 11121 .A2.L
O

8. (c) [h) = Energy


Frequency = [MLT-2A-2]

[ML2r2] = [ML2rl]. 18. (b) [Force] = [MLr2], [Impulse] =[MLT~l]


rr':
19. (d) [~] = ~;2, = [ML-1r2]
h] E [ML2r2]
M

9. (b) [I = vI = [rl][ML2]
[£]
2
= MLr = [ML-1r2]
=[rl] = [Frequency] A L2

10. (c) [R] = V = W = [ML2r2] = [ML2r3:r2] It


[ vo age
qJ = ML2r A-L ·AT 3 1
= [ML-1r2]
I qI [IT][!] v 3

[L] = [ML2rl]
11. (c) [Self inductance] = [ML2r2A-2]
Dimensions of angular momentum L are different
Refer to point 49 of Table 2.8 on page 2.22. from remaining three.
12. (a) [CR] =!L. V =1 = [IT] = [MlLDrl] 20. (c) Unit of b = unit of ~ =kmls2
. V I I [!] t
COMPETITION

2.B~

• 21. 'Cc)
J~.tV
r a;'] = IJ'l- Dimensionless:.constant;
k = pXsY-c
f oc . 'f h '.'
.'
[a] = [p][V2] = [ML-1:rr-2J[l3]2
J '.:.v "·~Ld.fo ::'[~~:r~]7['MF~yr[itl]Z
-
, \ tl) f 1 • ~

= [MLSf2],
"1 ~,

'6 ,), ~L¥ = MX+~Intt~r2x-3y-z r (.

22. (b) V = at+- •• X + Y = O,r::,x.+ Z '70, -2x -3y -z =0'


t+c
On solving, ~ = 1, Y = - L z = 1.
As c is added to t, so
[c] = [T]. [F] MLr2
,27. (rJ Pressure [P] =- =--'
e

R
• ~ ..' [AJ
[at] = [v]

eu.'; fill i '-[a] ,,;,,[Y]~J..rl::itr2] ,


[t] 'l;'~·,,('o " ,h"dr,

ill = [v.] , __t -, ',il~


' __
:-__
I

SI
[T] ~1- ' ~ - .N
; ... ~ fJ}P} ! IJ.-J
or [b]='L1.-l,T~=[L]_
23. (b) Given p = poe-aJ
r : 2 'la'" !rrv,l
-4 [ - 1
-' 7' 100
J[ 1
-,'
16
J':;.
at2 must be dimensionless
=40
[a] = _1_ =~ = [T-2,( -:. " 29. (b) ('Only t:; has "dimensions of [~lL3r2].
o. [t2] T2 _~ 0
Remaining three quantities are pure ratios.
IT
24. (c) Energy density " ' 1 "!.'"
En~rgy '[ML1.-4] 30. (a) K=-mv2
2
= Vol\lme:(' __[LfY , '•.
= [ML-1 r j' 2 .. LV< x 100
K
= 6mx1OO+2~x
m v
100
__

young's modulus,
=2%+2x3%
H
= £}~ = MLr2,~ ;=[ML-lr2]. = 8%.
A I L2 L-<
M M
25. (d) Given : f ="anekY 31. (d) p =V= E
Putting the dimensions'of various quantities,
O

6p x 100 = ¢.Mx 100 +3 M.~ 10<; e- f


p M L
= 1.[Mt [MI2JY -- • '[.; k = ~orce ]
'0

rl
' DIstance = 3o/'l+3x2%,=9%. '. 11
,
~l = M +Yr
x 2y
M
32. (b) p=-
x+y = 0 and -2y =-1 V
M

--
'1 1
y=- and x=-- .. 6p x 100 = 6M x 100 + 6V x 100
2 2- P M V
2, .

26. (c) [P] = Force


Area
= MLr =IML-1r2]
L2' r =( 0.01 + O.1)x 100 = 2%.
i ' 22.42 47
[5] = Energy
Area x Time I ~ 33. (b) Here V =~~ ,

ML2r2, •
= ,=[MI3] 6\1'x 100 =3 6: x 100
e.T ' V r
[c] = [Lrll =3x2%=6%,
SECTION

Delhi P T.an VMM(; Entrance Exam


,Ii (Similar Questi~nsl
- . '

1. Light year is used to 'mcii.ure (a)[MLr3] and [MLr4]


(a) distance, between st~fSl' i (b)[MLr1] apd l¥LTo.J .
. I I.l.l t ~.'" _ t ,
(b) dis~an~e:~meep'~tow~ -. ,J - (c) [MLr3] and [ML~]'
(c) stationary chaJge." I (d) [MLT-4] and [MLT1] [DPMT93]
• 1
(d) none of these. " I' r ; ;-
.J ; , \, 1!
[OPMT 07]
f
9. The dimensional formula ~ith~~onstant a in van
2. Which of the following is tJ;ue for the solid angle? der Waal's gas equation

R
~
(a) 5~= 5rtcosS-
',Ji
~r!

l.tb)_dO)=5ACOS
.' -- 2
e
'> r
(P~;2)W .b)"'::i:RT-is...ll iOI'J .'\
{. ~ t1 .J•..•..
hl11. .=..
•. : ..•£. f

(c) &ro 5Aco,se - ''.·~(d) 5rod O(\Cos


2
e (a)[ML3r2] (b)[ML2~1]
s...-2 ,<: ~ S
-,:3' J it"-! , , ,-3 [DPMT 99] (c) [ML" t J h "=- t (dUML:r:. L r~\; [DPM'!j92]

SI
'.
•3.Dimensions.of coeffcient of viscositY,~I a,re \' 10. The
.equation ( f' +\'1£)
~
"j
(V - b) = Constant. The
-, ; • e-" J •. ~, j l4 i . l.i· )..' >- ~. • J • ,t..
(a) [ML-1r11 (b) [ML-h-4] '!J':1 •
. l V _ j11

units of a are ·1 , • -r
(c) [ML-1r2] " ~;~ (d) [MI;2i~ , lPPMT ~ 04]
(a) dyne x ems (b) dyne x,an4
4. The dimensions of surface tension are
(c) dyne em-3 (d) dyneem-2.; [DPMT 06]
(a) Nm 'r ~- (b) Nm2 _ • _'. J

(c) Nm -1 t.· (d) N-s.'- [DPMT 01]


11. In a system of units, the units of mass, length
and time are 1 quintal, 1 kmand 1 hrespectively. In this
5. Dimensions Of 'Hubble's constant are
IT
system 1 N force will be equal-to r'
(a)[r1]. ' ; (b)[ML-r4] "
(a) 1 new unit" J ,(b) 1~~:?
n~w ,U¢t,§ or
(c) [MlLOr2] (d) [MLT1 1 [D.PMT 97]
(c) 125.7 new units \ ',i.J _
6. The units of Planck's constantare
(d) 103 new units.
(a) J/s (~) J&2
12. What is the dimension of surface tension ?
(d) J8-2
H
(c) Js [DPMT]
(a) [ML1ro] (b) [ML1r1]
7. Which of the following have same dimensions ? (c) [MLOy-2] (d) [M1L1r2) [DPMT 2011]
(a) torque and force
13. The significant figures of the number 6.0023 are
(b) torque and potential energy
(a) 1 (b) 5
O

(c) potential energy and force


(c) 4 (d) 2 [DPMT 95)
(d) Planck's constant and momentum [DPMT 98]
14. The angle subtended by a coin of radius 1 em
8. Force F is given by held at a distance of 80 em from your eyes is
F=at+bt2, (a) 1.430 (b) 0.72 0
where t is time. What are the dimensions of a and b ? (c) 0.0125° (d) 0.0250 [DPMT(9)
M

Answers and Explanations


1. (a) Light year is the distance travelled by light is 3. (a) [coefficient of viscosity] =[ML-1r1~
, .
one year.
, ,/ Force
2. (a) Solid
angle is normal projection of area 4. (c) Surface tension = ---
. Length'
divided by square of radius of the curved surface
:. SI unit of surface tension' = Nm-1.
50)= 5A.cos9
,J
,

COMPETITION

2." PHYSICS-XI

5. (4) According to Hubble's law, speed of recession 11. (b) [Force] = [MLr2]
of a galaxy, .
v=Hr
1 .
n, n,[~:r[~:r[~:r
=

.. [H]=[v]=[Lr ]=[r1]
[r] [L] -1[ lkg
1 quintal
]1[lm]1[ls]-2
lkm Ih
6. (c). h = 'Energy

Unit of h =
.Frequency

~l =Js.
= 1[I~J[I~oJ[3~r2

R
s 3600 x 3600 .
= = 129.6 new umts.
7. (b) [Torque] = [Potential energy] 100 x 1000
12. (c) Surface tension
.=[ML2 r-2]
= Force = MLr2 = [MLOor-2]
8. (a) [a]= [Fl=lMLr2] = [MLr3]

SI
Length L
. [t] [T]
2 13. (b) The number of significant figures in 6.0023 is
[bf=J£l=[MLr ] five (6, 0, 0, 2, 3).
[tf [T]2
14. (a) Here I = 80 ern, 2 r = 2 ern
= [MLr4]

. 9. (c)

..
[Pf=[;2]

[a1 = IP][v2] = [ML-1T-2][L3]2


Eye~r
~T
IT
=[ML5r2] .
10. (b) .unit :of a = Unit of P x Unit of V2
e = -- Arc =-
2r 2 1
= - =- rad
Radius I 80 40
. = dyne ctn- .x (ern
2 3)2
= (180 x ~)O = 1.430..
=.dyne em", 1t 40
H
O
M

You might also like